Участник:Yulya3102/Матан3сем — различия между версиями

Материал из Викиконспекты
Перейти к: навигация, поиск
м (Дифференцирование «произведений»)
(Полиномиальная формула)
 
(не показано 219 промежуточных версий 18 участников)
Строка 1: Строка 1:
 
== Основные вопросы ==
 
== Основные вопросы ==
=== Список теорем ===
 
 
==== Ненаписанные теоремы ====
 
 
Теорема о равномерной сходимости и непрерывности степенного ряда
 
 
Линейные и комплексно линейные отображения. Уравнения Коши--Римана
 
 
Необходимое условие дифференцируемости.
 
 
Достаточное условие дифференцируемости
 
 
Теорема о пространстве линейных отображений
 
 
Теорема о непрерывно дифференцируемых отображениях
 
 
Необходимое условие экстремума. Теорема Ролля — без теоремы Ролля
 
 
Лемма об оценке квадратичной форме и об эквивалентных нормах — не знаю, что хочет Костик, но знаю, что думает Виноградов
 
 
Характеризация потенциальных векторных полей в терминах интегралов
 
 
Потенциальность локально потенциального поля. Следствие о лемме Пуанкаре — проверить текст леммы и та ли она вообще
 
 
Метод Лапласа вычисления асимптотики интегралов — проверить формулировку
 
 
==== Теоремы без доказательств ====
 
Теорема об интегрировании функционального ряда
 
 
Теорема о дифференцировании функционального ряда
 
 
Теорема о почленном предельном переходе в суммах
 
 
Теорема о перестановке пределов
 
 
Признак Дирихле равномерной сходимости функционального ряда
 
 
Метод суммирования Абеля
 
 
Теорема о равномерной сходимости и непрерывности степенного ряда
 
 
Линейные и комплексно линейные отображения. Уравнения Коши--Римана
 
 
Теорема о почленном дифференцировании степенного ряда
 
 
Экспонента, синус, косинус. Свойства.
 
 
Необходимое условие дифференцируемости.
 
 
Достаточное условие дифференцируемости
 
 
Дифференцирование композиции
 
 
Дифференцирование «произведений»
 
 
Теорема Лагранжа для векторнозначных функций
 
 
Экстремальное свойство градиента
 
 
Независимость частных производных от порядка дифференцирования
 
 
Полиномиальная формула
 
 
Лемма о дифференцировании «сдвига»
 
 
Многомерная формула Тейлора (с остатком в форме Лагранжа и Пеано)
 
 
Теорема о пространстве линейных отображений
 
 
Теорема Лагранжа для отображений
 
 
Теорема об обратимости линейного отображения, близкого к обратимому
 
 
Теорема о непрерывно дифференцируемых отображениях
 
 
Необходимое условие экстремума. Теорема Ролля
 
 
Лемма об оценке квадратичной форме и об эквивалентных нормах
 
 
Достаточное условие экстремума
 
 
Лемма о почти локальной инъективности
 
 
Теорема о сохранении области
 
 
Теорема о диффеоморфизме
 
 
Теорема о локальной обратимости
 
 
Теорема о неявном отображении
 
 
Теорема о задании гладкого многообразия системой уравнений
 
 
Необходимое условие относительного локального экстремума
 
 
Вычисление нормы линейного оператора с помощью собственных чисел
 
 
Простейшие свойства интеграла векторного поля по кусочно-гладкому пути
 
 
Обобщенная формула Ньютона--Лебница
 
 
Характеризация потенциальных векторных полей в терминах интегралов
 
 
Лемма о дифференцировании интеграла по параметру
 
 
Необходимое условие потенциальности гладкого поля. Лемма Пуанкаре
 
 
Лемма о гусенице
 
 
Лемма о равенстве интегралов по похожим путям
 
 
Лемма о похожести путей, близких к данному
 
 
Равенство интегралов по гомотопным путям
 
 
Потенциальность локально потенциального поля. Следствие о лемме Пуанкаре
 
 
Асимптотика интеграла $\int_0^{\pi/2}\cos^nx\,dx$, $n\no+\infty$
 
 
Лемма о локализации (в методе Лапласа)
 
 
Метод Лапласа вычисления асимптотики интегралов
 
 
Теоерма Вейерштрасса о приближении функций многочленами
 
 
Формула Стирлинга для Гамма-функции
 
  
 
=== Признак Вейерштрасса ===
 
=== Признак Вейерштрасса ===
Строка 149: Строка 23:
  
 
из 1) и 2) <tex> \Rightarrow S(x) </tex> непрерывна в <tex> (\cdot) x_0 </tex>
 
из 1) и 2) <tex> \Rightarrow S(x) </tex> непрерывна в <tex> (\cdot) x_0 </tex>
 +
 +
Где вы вообще такое доказательство нашли? Тут фигня какая-та. Нормальное доказательство есть в Фихтенгольце.
 
}}
 
}}
  
Строка 170: Строка 46:
  
 
=== Теорема о дифференцировании функционального ряда ===
 
=== Теорема о дифференцировании функционального ряда ===
Проверить пункты про сходимость
 
 
{{Теорема
 
{{Теорема
 
|statement=
 
|statement=
Пусть <tex> u_n \in C'[a; b] </tex> (<tex> C' </tex> — множество непрерывно дифференцируемых функций). <tex> \sum u_n(x) </tex> поточечно сходится на <tex> [a; b] </tex>, <tex> S(x) = \sum u_n(x) </tex>. <tex> \sum u'_n(x) = \varphi(x)</tex> при <tex> x \in [a, b] </tex>,<tex> \sum u'_n(x) </tex> — равномерно сходится на <tex> [a; b] </tex> к <tex> \varphi(x) </tex>. Тогда <tex> S(x) \in C'([a, b]) </tex> и <tex> S'(x) = \varphi(x) </tex>.
+
Пусть <tex> u_n \in C'[a; b] </tex> (<tex> C' </tex> — множество непрерывно дифференцируемых функций).  
 +
 
 +
1) <tex> \sum_{n = 1}^{+ \infty} u_n(x)  = S(x) </tex> поточечно сходится на <tex> [a; b] </tex>
 +
 
 +
2) <tex> \sum_{n = 1}^{+ \infty} u'_n(x) = \varphi(x)</tex> равномерно сходится при <tex> x \in [a, b] </tex>
 +
 
 +
Тогда <tex> S(x) \in C'[a, b] </tex> и <tex> S'(x) = \varphi(x) </tex>.
 +
|proof=
 +
Следует из т. о предельном переходе под знаком производной (прошлый семестр).
 +
* <tex> (\lim_{n \to +\infty} f_n) = \lim_{n \to +\infty}(f{'}_n); \ f_n \in C^1[a, b] </tex>
 +
 
 +
* <tex> f_n \to f </tex> — поточечно на <tex> [a, b]. \ f{'}_n \rightrightarrows \varphi </tex> при <tex> n \to +\infty, x \in [a, b] </tex>
 +
 
 +
* Тогда <tex> f </tex> — дифф. на <tex> [a, b] \ \forall x \in [a, b] : f{'}(x) = \varphi(x) </tex>.
 +
 
 +
<tex> \begin{matrix} S_n \rightarrow S \\ S_{n}' \rightrightarrows \Phi \end{matrix} </tex> Тогда <tex> S' = \Phi </tex>
 
}}
 
}}
  
Строка 191: Строка 81:
  
 
2) <tex> \sum a_n = \lim_{x \to x_0} (\sum_{n=1}^{+\infty} u_n(x) ) </tex>
 
2) <tex> \sum a_n = \lim_{x \to x_0} (\sum_{n=1}^{+\infty} u_n(x) ) </tex>
 +
|proof=
 +
1) <tex> S_N = \sum_{n = 1}^{N} u_n(x); S_N^{(a)} = \sum_{n = 1}^{N} a_n ? S_N^{(a)} </tex> — имеет предел
  
 +
* Критерий Больцано-Коши <tex> \lim S_n^{(a)} = S^{(a)} </tex>
 +
* <tex> \forall \epsilon > 0 \ \exists N \ \forall n > N \ \forall p : |S_n^{(a)} - S_{n + p}^{(a)}| < \epsilon </tex>
 +
 +
<tex> |S_n^{(a)} - S_{n + p}^{(a)}| \le |S_n^{(a)} - S_n(x)| + |S_n(x) - S_{n + p}(x)| + |S_{n + p}(x) - S_{n + p}^{(a)}| </tex>
 +
 +
Берём <tex> \forall \epsilon > 0 </tex> из р. сх-ти
 +
 +
<tex> \exists N \ \forall n > N \ \forall p \ \forall x : |S_n(x) - S_{n + p}(x)| < \frac{\epsilon}{3} </tex>
 +
 +
<tex> |S_n(x) - S(x)| < \frac{\epsilon}{6} </tex>
 +
 +
<tex> |S_{n + p}(x) - S(x)| < \frac{\epsilon}{6} </tex>
 +
 +
При данном <tex>n : S_n(x) = u_1(x) + \ldots + u_n(x) \xrightarrow[x \rightarrow x_0]{} a_1 + \ldots + a_n = S_n^{(a)} </tex>
 +
 +
Выберем <tex> x </tex> так близко к <tex> x_0 </tex>, чтобы <tex> \begin{matrix} |S_n^{(a)} - S_n(x)| < \frac{\epsilon}{3} \\ |S_{n + p}(x) - S_{n + p}^{(a)}| < \frac{\epsilon}{3} \end{matrix} </tex>
 +
 +
<tex>u_n(x); \hat{u}_n(x) := \begin{Bmatrix} u_n(x) & x \ne x_0 \\ a_n & x = x_0 \end{Bmatrix}</tex> — непр. равномерно в <tex> (\cdot) x_0 </tex>
 +
 +
<tex> \sum \hat{u}_n(x) </tex> — р. сх. на <tex> \langle a, b \rangle </tex>
 +
 +
Утв. 2 следует из [[Участник:Yulya3102/Матан3сем#Теорема Стокса--Зайдля для рядов|т. 1. Стокса-Зайдля для рядов]]
 +
 +
<tex> M_n = \sup |\sum_{n = N + 1}^{+ \infty} \hat{u}_n(x)| \le \sup |\sum_{n = n + 1}^{+ \infty} u_n(x)| + |\sum_{n = N + 1}^{+ \infty} a_n| \xrightarrow[N \rightarrow +\infty]{} 0 </tex>
 
}}
 
}}
  
Строка 244: Строка 160:
 
Пусть есть ряд <tex> \sum a_n(x) b_n(x) </tex>, <tex> x \in X </tex>
 
Пусть есть ряд <tex> \sum a_n(x) b_n(x) </tex>, <tex> x \in X </tex>
  
1) частичные суммы ряда равномерно ограничены, т.е. <tex> \exists c_a \ \forall x | \sum_{k = 1}^{n} a_k(x) | \leqslant c_a </tex>
+
1) частичные суммы ряда <tex>a_n(x)</tex> равномерно ограничены, т.е. <tex> \exists c_a \ \forall x | \sum_{k = 1}^{n} a_k(x) | \leqslant c_a </tex>
  
 
2) <tex> b_n(x) </tex> монотонна по <tex> n </tex> и равномерно сходится к <tex> 0 </tex>
 
2) <tex> b_n(x) </tex> монотонна по <tex> n </tex> и равномерно сходится к <tex> 0 </tex>
  
 
Тогда <tex> \sum a_n(x) b_n(x) </tex> равномерно сходится на <tex> X </tex>.
 
Тогда <tex> \sum a_n(x) b_n(x) </tex> равномерно сходится на <tex> X </tex>.
 +
 +
|proof=
 +
Применяя преобразование Абеля
 +
 +
<tex>\sum_{k=n+1}^{n+p}b_k(x)a_k(x) = b_{n+p}(x)\sum_{k = 1}^{n + p}a_k(x)-\sum_{k=n+1}^{n+p-1}(b_{k+1}(x)-b_k(x))\sum_{j=1}^{k}a_j(x)</tex>
 +
 +
В силу равномерной ограниченности частичных сумм ряда <tex>\sum a_k(x)</tex> при некотором <tex>M</tex>
 +
 +
<tex>|\sum_{k = 1}^{n}a_k(x)| \le M \ \forall n \in N, \forall x \in X</tex>
 +
 +
Тогда, используя монотонность <tex>b_k(x)</tex> (по <tex>k</tex>), имеем
 +
 +
<tex>|\sum_{k=n+1}^{n+p}b_k(x)a_k(x)| \le M|b_{n+p}(x)|+M \sum_{k = n + 1}^{n+p-1}|b_{k+1}(x)-b_k(x)|= 2M|b_{n+p}(x)|+M|b_{n+1}(x)|</tex>
 +
 +
Из этого неравенства в силу <tex>b_k \rightrightarrows 0</tex> получаем, что
 +
 +
<tex>\forall \varepsilon > 0 \ \exists n(\varepsilon ) :
 +
|\sum_{k=n+1}^{n+p}b_k(x)a_k(x)| < \varepsilon \ \forall n \ge n(\varepsilon), \forall p \in N, \forall x \in X</tex>
 +
 +
Применяя критерий Коши, получаем, что ряд сходится равномерно на <tex>X</tex>.
 
}}
 
}}
  
Строка 256: Строка 192:
 
Пусть <tex> \sum a_n </tex> сходится. Рассмотрим функцию <tex> f(x) = \sum a_n x^n </tex>. Тогда <tex> \sum a_n = \lim_{x \to 1 - 0} f(x) </tex>.
 
Пусть <tex> \sum a_n </tex> сходится. Рассмотрим функцию <tex> f(x) = \sum a_n x^n </tex>. Тогда <tex> \sum a_n = \lim_{x \to 1 - 0} f(x) </tex>.
 
|proof=
 
|proof=
<tex>a_n, b_n = x^n ?; \ X = [0, 1]</tex>
+
<tex>a_n, b_n = x^n; \ X = [0, 1]</tex>
  
<tex> \sum a_n x^n </tex> — по пр. Абеля равномерно сх-ся <tex>[0, 1]</tex>
+
<tex> \sum a_n b_n </tex> — [[Участник:Yulya3102/Матан3сем#Признак Абеля равномерной сходимости|по признаку Абеля]] равномерно сх-ся <tex>[0, 1]</tex>
  
 
<tex>lim \ a_n x^n \xrightarrow[x \rightarrow 1 - 0]{} a_n </tex>
 
<tex>lim \ a_n x^n \xrightarrow[x \rightarrow 1 - 0]{} a_n </tex>
Строка 266: Строка 202:
 
{{Теорема
 
{{Теорема
 
|statement=
 
|statement=
<tex> \sum_{k=0}^{+ \infty} a_k(z-z_0)^k </tex> — произв. ст. ряд <tex> [ a_k \in \mathfrak{C}, z </tex> — комплексная переменная <tex> ] </tex> или <tex> [ a_k \in \Re; z, z_0 \in \Re ] </tex>
+
Пусть <tex> (A) </tex> <tex> \sum_{k=0}^{+ \infty} a_k(z-z_0)^k </tex> — произвольный степенной ряд <tex> [ a_k \in \mathbb{C}, z </tex> — комплексная переменная <tex> ] </tex> или <tex> [ a_k \in \mathbb{R}; z, z_0 \in \mathbb{R} ] </tex>
  
Возьмём три случая:
+
Возможны три случая:
  
1) <tex> \forall z \in \mathfrak{C} </tex> ряд <tex> (A) </tex> сходится
+
1) <tex> \forall z \in \mathbb{C} </tex> ряд <tex> (A) </tex> сходится
  
2) <tex> (A) </tex> сходится только при <tex> z = z_0 </tex>
+
2) <tex> (A) </tex> сходится только при <tex> z = z_0 </tex>
  
 
3) <tex> \exists R </tex>  <tex> 0 < R < + \infty </tex> при
 
3) <tex> \exists R </tex>  <tex> 0 < R < + \infty </tex> при
Строка 286: Строка 222:
 
<tex> \sum |a_k| \cdot |z - z_0|^k </tex>
 
<tex> \sum |a_k| \cdot |z - z_0|^k </tex>
  
Признак Коши:  
+
* Признак Коши: <tex> \overline{lim}_{n \to + \infty} \sqrt[n]{|a_n| \cdot |z - z_0|^n} = \overline{lim}_{n \to + \infty} \sqrt[n]{|a_n|} \cdot |z - z_0| = |z - z_0| \cdot\overline{lim}_{n \to + \infty} \sqrt[n]{|a_n|} </tex>
<tex> \overline{lim}_{n \to + \infty} \sqrt[n]{|a_n| \cdot |z - z_0|^n} = \overline{lim}_{n \to + \infty} \sqrt[n]{|a_n|} \cdot |z - z_0| = |z - z_0| \cdot\overline{lim}_{n \to + \infty} \sqrt[n]{|a_n|} </tex>
 
  
 
1) <tex> \overline{lim} = 0 </tex> при всех <tex> z </tex> ряд <tex> (A) </tex> сходится абсолютно
 
1) <tex> \overline{lim} = 0 </tex> при всех <tex> z </tex> ряд <tex> (A) </tex> сходится абсолютно
Строка 305: Строка 240:
 
{{Теорема
 
{{Теорема
 
|statement=
 
|statement=
Ряд <tex> (A) = \sum a_n(z - z_0)^n, 0 < R \le + \infty </tex> — р. сх-ся
+
Пусть ряд <tex> (A) = \sum a_n(z - z_0)^n, 0 < R \le + \infty </tex> — радиус сходимости. Тогда:
 
 
Тогда:
 
  
1) Для <tex> r : 0 < r < R </tex> ряд <tex> (A) </tex> р. сх-ся в круге <tex> \overline{B(z_0, r)} </tex>
+
1) Для <tex> r : 0 < r < R </tex> ряд <tex> (A) </tex> равномерно сходится в круге <tex> \overline{B(z_0, r)} </tex>
  
 
2) В круге <tex> B(z_0, R) </tex> сумма ряда <tex> (A) </tex> — непрерывна.
 
2) В круге <tex> B(z_0, R) </tex> сумма ряда <tex> (A) </tex> — непрерывна.
 
|proof=
 
|proof=
(1) Признак Вейерштрасса
+
(1) [[Участник:Yulya3102/Матан3сем#Признак Вейерштрасса|Признак Вейерштрасса]]
  
 
<tex> z \in \overline{B(z_0, r)} </tex>
 
<tex> z \in \overline{B(z_0, r)} </tex>
Строка 329: Строка 262:
  
 
=== Линейные и комплексно линейные отображения. Уравнения Коши--Римана ===
 
=== Линейные и комплексно линейные отображения. Уравнения Коши--Римана ===
 +
{{Лемма
 +
|statement=
 +
Пусть <tex> f: E \subset \mathbb{C} \to \mathbb{C}, \ z_0 \in \operatorname{Int} E, \ f </tex> — комплексно дифференцируема в точке <tex> z_0 </tex>. Тогда, если <tex> f \leftrightarrow F: \mathbb{R}^2 \to \mathbb{R}^2, \ (x, y) \mapsto (\operatorname{Re}{f(x + iy)}, \operatorname{Im}{f(x + iy)} ) </tex>, отображение <tex> F </tex> дифференцируемо в <tex> (x_0, y_0) </tex> и выполнены соотношения:
 +
 +
<tex> \frac{\partial F_1}{\partial x} (x_0, y_0) = \frac{\partial F_2}{\partial y} (x_0, y_0) </tex>
 +
 +
<tex> \frac{\partial F_1}{\partial y} (x_0, y_0) = - \frac{\partial F_2}{\partial x} (x_0, y_0) </tex>
 +
 +
(уравнения Коши-Римана)
 +
 +
|proof=
 +
Википедия [http://ru.wikipedia.org/wiki/%D0%A3%D1%81%D0%BB%D0%BE%D0%B2%D0%B8%D1%8F_%D0%9A%D0%BE%D1%88%D0%B8_%E2%80%94_%D0%A0%D0%B8%D0%BC%D0%B0%D0%BD%D0%B0]
 +
}}
 +
 
=== Теорема о почленном дифференцировании степенного ряда ===
 
=== Теорема о почленном дифференцировании степенного ряда ===
 
{{Теорема
 
{{Теорема
 
|statement=
 
|statement=
Рассмотрим ряды <tex> \sum_{n=0}^{+ \infty} a_n (z - z_0)^n = f(z), \ R \in [0; + \infty], \ |z - z_0| < R </tex> и <tex> (\sum_{n=1}^{+ \infty} n a_n(z - z_0)^{n-1} </tex> Тогда:
+
Ряд <tex> (A) = \sum a_n(z - z_0)^n = f(z), R \in [0, + \infty], |z - z_0| < R </tex>
 +
 
 +
Ряд <tex> (A)' = \sum_{n = 1}^{+ \infty} n a_n (z - z_0)^{n - 1} </tex>
  
1) радиус сходимости второго ряда равен <tex> R </tex>
+
Тогда: 1) радиус сх-ти <tex> (A') = R </tex>. 2) при <tex> |z - z_0| < R; f'(z) = \sum n a_n (z - z_0)^{n - 1} </tex>
  
2) при <tex> |z - z_0| < R \ f'(z) = \sum n a_n (z - z_0)^{n - 1} </tex>
+
[Тогда <tex>f</tex> — дифф. при <tex> |z - z_0| < r </tex> и <tex> f'(z) = \sum n a_n (z - z_0)^{n - 1} </tex> ]
 +
|proof=
 +
<tex>R = \frac{1}{\overline{\lim}\sqrt[n]{|a_n|}}; R_A = \frac{1}{\overline{\lim}\sqrt[n]{(n + 1)|a_{n + 1}|}} = R</tex>
 +
 
 +
<tex> \frac{f(z + h) - f(z)}{h} = \sum \frac{a_n (z + h - z_0)^n - a_n (z - z0)^n }{h} = \sum a_n \frac{(z + h - z_0) - (z - z_0)^n}{h} </tex>
 +
 
 +
Проверим р. сх. <tex> z \in B(z_0, r), r < R </tex>; <tex> ]h : |h| \le r - |z - z_0| </tex>
 +
 
 +
Тогда: <tex> z + h \in \overline{B(z_0, r)}; |z + h - z_0| \le r; |z - z_0| \le r </tex>
 +
 
 +
<tex> |a_n \frac{(z + h - z_0)^n - (z - z_0)^n}{h}| \le \frac{|a_n|}{|h|} n r^{n - 1} |h| = |a_n| n r^{n - 1} </tex>
 +
 
 +
<tex> \sum h|a_n|r^{n - 1} </tex> — сх. <tex>\Rightarrow</tex> по [[Участник:Yulya3102/Матан3сем#Признак Вейерштрасса|признаку Вейерштрасса]] р. сх. при <tex> |h| < r - |z - z_0| </tex>
 +
 
 +
<tex> f(z) = \lim_{h \rightarrow 0} \frac{f(z + h) - f(z)}{h} = \sum \lim a_n \frac{(z + h - z_0)^n - (z - z_0)^n}{h} = \sum n(z - z_0)^{n - 1} a_n </tex>
 
}}
 
}}
  
 
=== Экспонента, синус, косинус. Свойства. ===
 
=== Экспонента, синус, косинус. Свойства. ===
<tex> \mathrm{exp}(0) = 1 </tex>
+
1.1) <tex> \mathrm{exp}(0) = 1 </tex>
  
<tex> \mathrm{exp}(\overline{z}) = \overline{\mathrm{exp}(z)} </tex>
+
1.2) <tex> \mathrm{exp}(\overline{z}) = \overline{\mathrm{exp}(z)}; \ /S_n(\overline{z}) = \overline{S_n(x)})/</tex>
  
<tex> (\mathrm{exp}(z))' = \mathrm{exp}(z) </tex>
+
1.3) <tex> (\mathrm{exp}(z))' = \mathrm{exp}(z); \ /\sum_{n = 1}^{+ \infty} (\frac{z^n}{n!})' = \sum_{n = 1}^{+ \infty} \frac{z^{n - 1}}{(n - 1)!} = \sum_{n = 0}^{+ \infty} \frac{z^n}{n!}/ </tex>
  
<tex> \mathrm{exp}(z + w) = \mathrm{exp}(z) ⋅ \mathrm{exp}(w) </tex>
+
1.4) <tex> (\mathrm{exp}(x))'|_{x = 0} = 1 </tex>
  
<tex> \mathrm{exp}(z) 0, \ \forall z \in \mathbb{C} </tex>
+
{{Теорема
 +
|statement=
 +
<tex> \forall z, w \in \mathbb{C} : \mathrm{exp}(z + w) = \mathrm{exp}(z) ⋅ \mathrm{exp}(w) </tex>
 +
|proof=
 +
<tex> \sum \frac{z^n}{n!} \cdot \sum \frac{w^k}{k!} </tex>
 +
 
 +
<tex> \sum_{k = 0}^{+ \infty} \frac{(z + w)^k}{k!} = \sum_{k = 0}^{+ \infty} \sum_{l = 0}^{k} \frac{z^l}{l!} \cdot \frac{w^{k - l}}{(k - l)!} = \sum_{l = 0}^{+ \infty} \sum_{k = l}^{+ \infty} \frac{z^l}{l!} \cdot \frac{w^{k - l}}{(k - l)!} = </tex>
 +
 
 +
<tex> = \sum_{l = 0}^{+ \infty} \sum_{n = 0}^{+ \infty} \frac{z^l}{l!} \cdot \frac{w^n}{n!} = \sum_{l = 0}^{+ \infty}(\frac{z^l}{l!} \cdot \sum_{n = 0}^{+ \infty} \frac{w^n}{n!}) = (\sum \frac{w^n}{n!})(\sum \frac{z^l}{l!}) </tex>
 +
}}
  
<tex> \sin x = \frac{\mathrm{exp}(ix) - \mathrm{exp}(-ix)}{2i} </tex>
+
* Следствие: <tex> \mathrm{exp}(z) \ne 0 </tex> — ни при каких <tex> z </tex>
  
<tex> \cos x = \frac{\mathrm{exp}(ix) + \mathrm{exp}(-ix)}{2} </tex>
+
2.1) <tex> \sin x = \frac{\mathrm{exp}(ix) - \mathrm{exp}(-ix)}{2i} </tex>
  
<tex> \overline{\mathrm{exp}(iz)} = \mathrm{exp}(\overline{iz}) = \mathrm{exp}(-i\overline{z}) </tex>
+
2.2) <tex> \cos x = \frac{\mathrm{exp}(ix) + \mathrm{exp}(-ix)}{2} </tex>
  
<tex> \cos(z) = \sum_{n=0}^{+\infty} (-1)^n \frac{z^{2n}}{(2n)!} </tex>
+
2.3) <tex> \cos(z) = \sum_{n=0}^{+\infty} (-1)^n \frac{z^{2n}}{(2n)!} </tex>
  
<tex> \sin(z) = \sum_{n=0}^{+\infty} (-1)^n \frac{z^{2n - 1}}{(2n - 1)!} </tex>
+
2.4) <tex> \sin(z) = \sum_{n=0}^{+\infty} (-1)^n \frac{z^{2n - 1}}{(2n - 1)!} </tex>
  
Пусть <tex> T(x) = \mathrm{exp}(ix) </tex>
+
2.5) Пусть <tex> T(x) = \mathrm{exp}(ix) </tex>
  
 
<tex> T(x+y) = T(x)T(y) </tex>
 
<tex> T(x+y) = T(x)T(y) </tex>
Строка 368: Строка 340:
 
<tex> \sin(x + y) = \cos(x)\sin(y) + \cos(y)\sin(x) </tex>
 
<tex> \sin(x + y) = \cos(x)\sin(y) + \cos(y)\sin(x) </tex>
  
<tex> |T(x)| = 1 </tex>
+
2.6) <tex> |T(x)| = 1; \ \cos^2(x) + \sin^2(x) = 1 </tex>
 
 
<tex> \cos^2(x) + \sin^2(x) = 1 </tex>
 
 
 
<tex> \lim_{x \to 0} \frac{1-cos(x)}{x^2} = \frac{1}{2} </tex>
 
 
 
<tex> \lim_{x \to 0} \frac{1-cos(x)}{x} = 0 </tex>
 
  
<tex> e^x = 1 + x + \frac{x^2}{2} + ... </tex>
+
<tex> (\frac{T(x) + T(-x)}{2})^2 + (\frac{T(x) - T(-x)}{2i})^2 = T(x)T(-x) = T(0) = \mathrm{exp}(i0) = 1 </tex>
  
<tex> \sin(x) = x + \frac{x^3}{3} + ... </tex>
+
2.7) <tex> \lim_{x \to 0} \frac{\sin(x)}{x} = 1; \ \lim_{x \to 0} \frac{1 - \cos(x)}{x^2} = \frac{1}{2}</tex>
  
<tex> \cos(x) = 1 - \frac{x^2}{2} + ...</tex>
+
<tex> \lim_{x \to 0} (\frac{\mathrm{exp}(ix) - 1}{ix}) = \lim_{x \to 0} (\frac{\cos(x) - 1}{ix} + \frac{i \sin(x)}{ix}) </tex>
  
<tex> |x| < 1: \ (1 + x)^\alpha = 1 + \alpha x + \frac{\alpha (\alpha - 1)}{2} x^2 + ... </tex>
+
-----
 +
<tex> x \in \mathbb{C} \begin{cases} e^x = 1 + x + \frac{x^2}{2} + \ldots \\ \sin(x) = x + \frac{x^3}{3} + \ldots \\ \cos(x) = 1 - \frac{x^2}{2} + \ldots \end{cases} </tex>
  
<tex> |x| < 1: \ \frac{1}{1-x} = 1 + x + x^2 + ... </tex>
+
<tex> |x| < 1 \begin{cases} (1 + x)^\alpha = 1 + \alpha x + \frac{\alpha (\alpha - 1)}{2} x^2 + \ldots \\ \frac{1}{1-x} = 1 + x + x^2 + \ldots \\ \ln(1 + x) = x - \frac{x^2}{2} + \frac{x^3}{3} - \ldots \end{cases}</tex>
  
<tex> |x| < 1: \ \ln(1 + x) = x - \frac{x^2}{2} + \frac{x^3}{3} - ... </tex>
+
<tex> \sum a_k \to </tex> [[Участник:Yulya3102/Матан3сем#Признак Абеля равномерной сходимости|Абель]] <tex> \to \sum a_k \cdot x^k = f(x); \lim_{x \to 1- 0}f(x) = S </tex>
  
 
=== Единственность производной ===
 
=== Единственность производной ===
Строка 420: Строка 387:
 
{{Теорема
 
{{Теорема
 
|statement=
 
|statement=
<tex> f : E \subset \Re^m \rightarrow \Re </tex> — дифф. <tex> a \in Int(E) </tex>
+
Пусть <tex> f : E \subset \mathbb{R}^m \rightarrow \mathbb{R} </tex> — дифференцируемо в точке <tex> a \in \operatorname{Int}(E) </tex>
  
Тогда <tex> \forall x \exists {\partial f\over\partial x_k}(a) </tex> и матрица Якоби <tex> f'(a) = ({\partial f\over\partial x_1}(a), \ldots, {\partial f\over\partial x_m}(a)) </tex>
+
Тогда <tex> \forall x \ \exists {\partial f\over\partial x_k}(a) </tex> и матрица Якоби <tex> f'(a) = ({\partial f\over\partial x_1}(a), \ldots, {\partial f\over\partial x_m}(a)) </tex>
  
Замечание: Для <tex> F : E \rightarrow \Re^l </tex> — дифф. <tex>(a)</tex>; <tex>F'(a) = ({\partial f_i\over\partial x_j})_{i = 1 \ldots l; j = 1 \ldots m} </tex>
+
Замечание: Для <tex> F : E \rightarrow \mathbb{R}^l </tex> — дифференцируемо в точке <tex> a </tex>; <tex>F'(a) = ({\partial f_i\over\partial x_j})_{i = 1 \ldots l; j = 1 \ldots m} </tex>
 
|proof=
 
|proof=
<tex>f(a + h) = f(a) = f'(a) \cdot h + o(h)</tex>
+
<tex>f(a + h) = f(a) + f'(a) \cdot h + o(h)</tex>
  
 
<tex> h := (0, \ldots, 0, t, 0, \ldots, 0) </tex>
 
<tex> h := (0, \ldots, 0, t, 0, \ldots, 0) </tex>
  
<tex> f(a_1, \ldots, a_k + t, \ldots, a_m) = f(a_1 \ldots a_m) + (f'(a))_k \cdot t + o(t) </tex> — это св-во дифф-ти <tex> \varphi_k </tex> в <tex> \cdot (a) </tex> из опр. частн. производных
+
<tex> f(a_1, \ldots, a_k + t, \ldots, a_m) = f(a_1 \ldots a_m) + (f'(a))_k \cdot t + o(t) </tex> — это св-во дифф-ти <tex> \varphi_k </tex> в <tex> \cdot (a) </tex> из [[Участник:Yulya3102/Матан3сем#Частные производные|опр. частн. производных]].
  
 
<tex> {o(h)\over ||L||} \rightarrow 0 </tex>
 
<tex> {o(h)\over ||L||} \rightarrow 0 </tex>
Строка 438: Строка 405:
 
{{Теорема
 
{{Теорема
 
|statement=
 
|statement=
<tex> f : E \subset \Re^m \rightarrow \Re; \ \exists r \ B(a, r) \subset E </tex>
+
Пусть <tex> f : E \subset \mathbb{R}^m \rightarrow \mathbb{R}; \ \exists r \ B(a, r) \subset E </tex>, в шаре <tex>B(a, r) </tex> существуют все <tex> f'x_k, k = {1..m} </tex> и все производные непрерывны в точке <tex> a</tex>. Тогда <tex> f </tex> дифференцируема в точке <tex> a</tex>
 +
|proof=
 +
<tex> m = 2 </tex>
  
В шаре <tex>B(a, r) </tex> существуют все <tex> f'x_k, k = {1..m} </tex> и все производные непрерывны в <tex> (\cdot) a</tex>
+
<tex> f(x_1, x_2) - f(a_1, a_2) = (f(x_1, x_2) - f(x_1, a_2)) + (f(x_1, a_2) - f(a_1, a_2)) =^* </tex> // <tex> =^* </tex> — По теореме Лагранжа
  
Тогда <tex> f </tex> дифф. в <tex> (\cdot) \ a</tex>
+
// <tex> \varphi_2(t) = f(x, t); \varphi_2(x_2) - \varphi_2(a_2) = \varphi'_2(t) \cdot (x_2 - a_2) </tex> // <tex> t </tex> — средняя точка
 +
 
 +
<tex> =^* \frac{\partial f}{\partial x_2}(x_1, \bar x_2)(x_2 - a_2) + \frac{\partial f}{\partial x_1}(\bar x_1, a_2)(x_1 - a_1) = </tex><tex> \frac{\partial f}{\partial x_2}(a_1, a_2)(x_2 - a_2) + \frac{\partial f}{\partial x_1}(a_1, a_2)(x_1 - a_1) + </tex>
 +
 
 +
<tex> o(\begin{bmatrix} x_1 - a_1 \\ x_2 - a_2 \end{bmatrix}) \to ||\ldots|| = \sqrt{(x_1 - a_1)^2 + (x_2 - a_2)^2} \begin{cases} + [\frac{\partial f}{\partial x_2}(x_1, \bar x_2) - \frac{\partial f}{\partial x_2}(a_1, a_2)](x_2 - a_2) + \\ [\frac{\partial f}{\partial x_1}(\bar x_1, a_2) - \frac{\partial f}{\partial x_1}(a_1, a_2)](x_1 - a_1) \end{cases}</tex>
 +
 
 +
<tex>[\ldots] \cdot \frac{x - a}{\sqrt{(x_1 - a_1)^2 + (x_2 - a_2)^2}} \ \</tex> где: <tex> \frac{x - a}{\sqrt{(x_1 - a_1)^2 + (x_2 - a_2)^2}} \le 1 </tex> по модулю; <tex> [\ldots] \to 0 </tex> при <tex> (x_1, x_2) \to (a_1, a_2) </tex>  
 
}}
 
}}
  
Строка 448: Строка 423:
 
{{Лемма
 
{{Лемма
 
|statement=
 
|statement=
Пусть <tex> A: \mathbb{R}^m \to \mathbb{R}^l </tex> — линейный оператор. Тогда <tex> \forall x \in \mathbb{R}^m \ ||Ax|| = C_A || x || </tex>, где <tex> C_A = \sqrt{\sum_{i, j} a_{i, j}^2} </tex> (<tex> a_{i, j} </tex> — элементы его матрицы)
+
Пусть <tex> A: \mathbb{R}^m \to \mathbb{R}^l </tex> — линейный оператор. Тогда <tex> ||Ax|| \le C_A||x|| </tex>, где <tex> C_A = \sqrt{\sum_{i, j} a_{i, j}^2} </tex> (<tex> a_{i, j} </tex> — элементы его матрицы)
 
|proof=
 
|proof=
 
<tex> ||x|| = 0 </tex>, т.е. если <tex> x = 0 </tex>, то тривиально
 
<tex> ||x|| = 0 </tex>, т.е. если <tex> x = 0 </tex>, то тривиально
Строка 466: Строка 441:
 
{{Теорема
 
{{Теорема
 
|statement=
 
|statement=
Пусть <tex> F: E \subset \mathbb{R}^m \to \mathbb{R}^l, \ a \in \operatorname{Int} E, \ F(E) \subset I </tex>, <tex> G: I \subset \mathbb{R}^l \to \mathbb{R}^n, \ b = F(a) \in \operatorname{Int} I </tex>, <tex> F </tex> дифференцируемо в <tex> a </tex>, <tex> G </tex> дифференцируемо в <tex> b </tex>. Тогда <tex> G \circ F </tex> дифференцируемо в <tex> a </tex>, и при этом <tex> (G \circ F)'(a) = G'(F(a)) F'(a) </tex>
+
<tex> F : E \subset \mathbb{R}^m \to \mathbb{R}^l; \ a \in IntE, F(E) \subset I </tex>
 +
 
 +
<tex> G : I \subset \mathbb{R}^l \to \mathbb{R}^n; \ b = F(a) \in IntI </tex>
 +
 
 +
<tex> F </tex> — дифф. в <tex> (\cdot) a, G </tex> — дифф. в <tex> (\cdot) b </tex>;
 +
 
 +
<tex> H = G \circ F \ // H(x) = G(F(x)) </tex>
 +
 
 +
Тогда: <tex> H </tex> — дифф. в <tex> (\cdot) a; H'(a) = G'(F(a)) \cdot F'(a) </tex>
 +
|proof=
 +
<tex> F(a + h) = F(a) + F'(a)h + \alpha(h)||h||; \ // \alpha(h) \xrightarrow[h \to 0]{} 0 </tex>
 +
 
 +
<tex> G(b + k) = G(b) + G'(b)k + \beta(k)||k||; \ // \beta(k) \xrightarrow[k \to 0]{} 0 </tex>
 +
 
 +
<tex> H(a + h) = G(F(a + h)) = G(\overbrace{F(a)}^{b} + \overbrace{F'(a)h + \alpha(h)||h||}^{k}) = </tex><tex> G(b) + G'(b)(F'(a)h + \alpha(h)||h||) + \beta(k)||k|| = </tex>
 +
 
 +
<tex> = \overbrace{G(F(a)) + G'(F(a) \cdot F'(a)h)}^{H(a)} + \overbrace{G'(b)\alpha(h)||h|| + \beta(k)||k||}^{? o(h) \leftarrow \text{proverim}} </tex>
 +
 
 +
1. <tex> ||\ G'(b)\alpha(h)\|h\| \ || = \|h\| \cdot ||G'(b)\alpha(h)|| \le \|h\|\cdot C_{G(b)} \cdot ||\alpha(h)|| = o(h) </tex>
 +
 
 +
2. <tex> \beta(k)||k|| </tex>
 +
 
 +
<tex> \|k\| = || \ F'(a)h + \alpha(h)\|h\| \ || \le \overbrace{||F'(a)h||}^{C_{F'(a)} \cdot \|h\|} + \|\alpha(h)\|\cdot\|h\| \le (C_{F'(a)} + \|\alpha(h)\|\cdot \|h\|) </tex>
 +
 
 +
<tex> ||\ \beta(k)\cdot \|k\| \ || \le \overbrace{||\beta{k}||}^{\to 0, h \to 0} \cdot \overbrace{(C_{F'(a)} + ||\alpha(h)||)}^{ogr. pri: \ h \to 0} \cdot \|h\| = o(h)</tex>
 +
 
 +
<tex> F = (f_1(x_1 \ldots x_m), f_2(x_1 \ldots x_m), \ldots, f_l(x_1 \ldots x_m)) </tex>
 +
 
 +
<tex> G = (g_1(y_1 \ldots y_l), \ldots, g_n(y_1 \ldots y_l)) </tex>
 +
<tex> H = \overbrace{g_1}^{h_1}(f_1(x_1 \ldots x_n), \ldots, f_l(x_1 \ldots x_n)), \ldots, \overbrace{g_n}^{h_n}(f \ldots)) </tex>
 +
 
 +
<tex> \frac{\partial h_i}{\partial x_j}(a) = \frac{\partial g_i}{\partial y_1}(b) \cdot \frac{\partial f_1}{\partial x_j}(a) + \frac{\partial g_i}{\partial y_2}(b) \cdot \frac{\partial f_2}{\partial x_j}(a) + \ldots + \frac{\partial g_i}{\partial y_l}(b) \cdot \frac{\partial f_l}{\partial x_j}(a) </tex>
 
}}
 
}}
  
Строка 474: Строка 480:
 
Пусть <tex> F, G: \ E \subset \mathbb{R}^m \to \mathbb{R}^l </tex>, <tex> \lambda: E \to \mathbb{R} </tex>, <tex> a \in \operatorname{Int} E </tex>; <tex> F, G, \lambda </tex> — дифференцируемые в <tex> a </tex>. тогда:
 
Пусть <tex> F, G: \ E \subset \mathbb{R}^m \to \mathbb{R}^l </tex>, <tex> \lambda: E \to \mathbb{R} </tex>, <tex> a \in \operatorname{Int} E </tex>; <tex> F, G, \lambda </tex> — дифференцируемые в <tex> a </tex>. тогда:
  
1) <tex> (\lambda F)' (a) h = ( \lambda'(a), h ) F(a) + \lambda(a) (F'(a) h) </tex>
+
1) <tex> (\lambda F)' (a) h = ( \lambda'(a) h ) F(a) + \lambda(a) (F'(a) h) </tex>
  
 
2) <tex> \left \langle F, G \right \rangle ' (a) h = \left \langle F'(a) h, G(a) \right \rangle + \left \langle F(a), G'(a) h \right \rangle </tex>
 
2) <tex> \left \langle F, G \right \rangle ' (a) h = \left \langle F'(a) h, G(a) \right \rangle + \left \langle F(a), G'(a) h \right \rangle </tex>
Строка 484: Строка 490:
 
<tex> (\lambda f_i)'(a)h = (\lambda'(a)(h))f_i(a) + \lambda(a)(f'_i(a)h) </tex> — <tex>i</tex>-ая коорд. док. ф-лы; <tex> ]f_i \leftrightarrow f </tex>
 
<tex> (\lambda f_i)'(a)h = (\lambda'(a)(h))f_i(a) + \lambda(a)(f'_i(a)h) </tex> — <tex>i</tex>-ая коорд. док. ф-лы; <tex> ]f_i \leftrightarrow f </tex>
  
<tex> \lambda(a + h)f(a + h) - \lambda(a)f(a) = (\lambda(a + h) - \lambda(a))f(a + h) + \lambda(a)f(a + b) - f(a)) =  
+
<tex> \lambda(a + h)f(a + h) - \lambda(a)f(a) = (\lambda(a + h) - \lambda(a))f(a + h) + \lambda(a)(f(a + h) - f(a)) =  
(\lambda'(a)h + o(h))f(a + h) + \lambda(a)(f'(a)h + o(h)) =  
+
(\lambda'(a)h + o(h))f(a + h) + \lambda(a)(f'(a)h + o(h)) = </tex>
(\lambda'(a)h) \cdot f(a) + \lambda(a)f'(a)h + (\lambda'(a)h)(f(a + h) - f(a)) + o(h)f(a + h) + \lambda(a) \cdot o(h) </tex>
+
 
 +
<tex> = (\lambda'(a)h) \cdot f(a) + \lambda(a)f'(a)h + (\lambda'(a)h)(f(a + h) - f(a)) + o(h)f(a + h) + \lambda(a) \cdot o(h) </tex>
  
 
<tex> || \frac{1 slag.}{||h||} || = \frac{|\lambda'(a)h|\cdot||f(a + h) - f(a)||}{||h||} \le \frac{||\lambda'(a)||\cdot||h||\cdot||f(a + h) - f(a)||}{||h||} \rightarrow 0 </tex>
 
<tex> || \frac{1 slag.}{||h||} || = \frac{|\lambda'(a)h|\cdot||f(a + h) - f(a)||}{||h||} \le \frac{||\lambda'(a)||\cdot||h||\cdot||f(a + h) - f(a)||}{||h||} \rightarrow 0 </tex>
Строка 496: Строка 503:
 
2. <tex> \left \langle F, G \right \rangle ' (a)h = (\sum_{i = 1}^{l}f_i g_i)'(a)h = </tex> лин. дифф. <tex> \sum(f_i g_i)'(a)h = \sum(f'_i(a)h)g_i(a) </tex><tex> + f_i(a)(g'_i(a)h) = \left \langle F'(a)h, G(a) \right \rangle + \left \langle F(a), G'(a)h \right \rangle </tex>
 
2. <tex> \left \langle F, G \right \rangle ' (a)h = (\sum_{i = 1}^{l}f_i g_i)'(a)h = </tex> лин. дифф. <tex> \sum(f_i g_i)'(a)h = \sum(f'_i(a)h)g_i(a) </tex><tex> + f_i(a)(g'_i(a)h) = \left \langle F'(a)h, G(a) \right \rangle + \left \langle F(a), G'(a)h \right \rangle </tex>
  
Замечание: <tex>m = 1; \ F, G : \Re \rightarrow \Re^l </tex>
+
Замечание: <tex>m = 1; \ F, G : \mathbb{R} \rightarrow \mathbb{R}^l </tex>
  
 
<tex> \left \langle F, G \right \rangle ' (a) = \left \langle F'(a), G(a) \right \rangle + \left \langle F(a), G'(a) \right \rangle </tex>
 
<tex> \left \langle F, G \right \rangle ' (a) = \left \langle F'(a), G(a) \right \rangle + \left \langle F(a), G'(a) \right \rangle </tex>
Строка 504: Строка 511:
 
{{Теорема
 
{{Теорема
 
|statement=
 
|statement=
Пусть <tex> a, b \in \mathbb{R} </tex>, <tex> a < b </tex>, вектор-функция <tex> f: [a, b] \to \mathbb{R}^m </tex> непрерывна на <tex> [a, b] </tex> и дифференцируема на <tex> (a, b) </tex>. Тогда найдётся такая точка <tex> c \in (a, b) </tex>, что <tex> || f(b) - f(a) || \leqslant || f'(c) || \cdot |b - a| </tex>.
+
<tex> F : [a, b] \rightarrow \mathbb{R}^l; F </tex> — непр. на <tex> [a, b] </tex> и дифф. на <tex> [a, b] </tex>
 +
 
 +
Тогда: <tex> \exists c_{G(a, b)} : ||F(b) - F(a)|| \le ||F'(c)|| \cdot |b - a| </tex>
 +
|proof=
 +
<tex>\varphi (t) := \langle F(b) - F(a), F(t) \rangle; t \in [a, b]; (\varphi : [a, b] \rightarrow \mathbb{R}) </tex>
 +
 
 +
<tex> \varphi(b) - \varphi(a) = \langle F(b) - F(a), F(b) - F(a) \rangle = ||F(b) - F(a)||^2 </tex>
 +
 
 +
<tex> \begin{matrix} \varphi'(t) = \langle F(b) - F(a), F'(t) \rangle \\
 +
\varphi(b) - \varphi(a) = \varphi'(c)(b - a) \end{matrix} </tex>
 +
 
 +
<tex> ||F(b) - F(a)|| \le ||F'(c)||(b - a) </tex>
 +
 
 +
// Если ехать быстро и криво
 +
 
 +
<tex> F : \mathbb{R} \rightarrow \mathbb{R}^2; t \rightarrow (\cos t, \sin t) </tex>
 +
 
 +
<tex> F' = (-\sin t, \cos t); ||F'(t)|| = 1 </tex> при <tex> \forall t </tex>
 +
 
 +
<tex> ||F(b) - F(a)|| \ne ||F'(c)|| \cdot (b - a) </tex>
 +
 
 +
// <tex>||F'(x)|| = 1; (b - a) </tex> — длина дуги; <tex> ||F(b) - F(a)|| </tex> — длина хорды
 
}}
 
}}
  
Строка 510: Строка 538:
 
{{Теорема
 
{{Теорема
 
|statement=
 
|statement=
Пусть функция <tex> f: D \subset \mathbb{R}^n \to \mathbb{R} </tex> дифференцируема в точке <tex> x \in \operatorname{Int} D, \ \operatorname{grad} f(x) \neq \mathbb{O}_n </tex>. Тогда для любого <tex> h \in \mathbb{R}^n: |h| = 1</tex> верно <tex> \ -|\operatorname{grad} f(x)| \leqslant D_h f(x) \leqslant | \operatorname{grad} f(x)| </tex>.
+
<tex> f : E \subset \mathbb{R}^m \rightarrow \mathbb{R}; f </tex> — дифф. в <tex> (\cdot) a, \nabla f(a) \ne 0 </tex>
 +
 
 +
<tex> l = \frac{\nabla f(a)}{||\nabla f(a)||} </tex> — направление
 +
 
 +
Тогда <tex> l </tex> указывает напр-е наискорейшего возр. ф-и, а <tex> -l </tex> самого быстрого убывания.
 +
 
 +
Более того: <tex> \forall </tex> напр. <tex> u : -||\nabla f(a)|| \le \frac{\partial f}{\partial u}(a) \le ||\nabla f(a)|| </tex> равенство достижимо для <tex> u = \pm l </tex>
 +
|proof=
 +
<tex> -||\nabla f(a)|| \cdot ||u|| \le \frac{\partial f}{\partial u}(a) \le ||\nabla f(a)|| \cdot ||u|| </tex> // <tex> u = 1 </tex>
 +
 
 +
// <tex> \frac{\partial f}{\partial u}(a) = \langle \nabla f(a), u \rangle </tex>
 
}}
 
}}
  
Строка 516: Строка 554:
 
{{Теорема
 
{{Теорема
 
|statement=
 
|statement=
Пусть <tex> r - 1 \in \mathbb{N} </tex>, <tex> D </tex> открыто в <tex> \mathbb{R}^n </tex>, <tex> f \in C^{r} (D), \ i_1, ... , i_r \in [1 : n] </tex>, набор <tex> (j_1, ..., j_r) </tex> получен из набора <tex> (i_1, ... , i_r) </tex> перестановкой. Тогда для всех <tex> x \in D </tex> верно <tex> D_{i_1, ..., i_r}^r f(x) = D_{j_1, ..., j_r}^r f(x) </tex>.
+
<tex> f : E \subset \mathbb{R}^2 \to \mathbb{R}; \ a \in IntE </tex>
 +
 
 +
<tex> \frac{\partial f}{\partial x_1}, \frac{\partial f}{\partial x_2} </tex> — опр. в окр. <tex> (\cdot) a </tex>, дифф. в окр. <tex> (\cdot) a </tex>
 +
 
 +
<tex> \frac{\partial^2 f}{\partial x_1 \partial x_2} </tex> и <tex> \frac{\partial^2 f}{\partial x_2 \partial x_1} </tex> — непр. в <tex> (\cdot) a </tex>
 +
 
 +
Тогда эти две частные производные равны.
 +
|proof=
 +
<tex> \vartriangle^2 f(h, k) = f(a_1 + h, a_2 + k) - f(a_1 + h, a_2) - f(a_1, a_2 + k) + f(a_1, a_2) </tex> — задано при <tex> |h|, |k| < r; V(a) = B(a, 2r) </tex>
 +
 
 +
фикс. <tex>k: \varphi(h) = f(a_1 + h, a_2 + k) - f(a_1 + h, a_2) </tex>
 +
 
 +
<tex> \vartriangle^2 f(h, k) = \varphi(h) - \varphi(0) \overbrace{=}^{t. Lagrange} \varphi'(\bar h)h = </tex><tex> (f'_{x_1}(a_1 + \bar h, a_2 + k) - f'_{x_1}(a + \bar h, a_2) )h  \overbrace{=}^{t. Lagrange} f''_{x_1 x_2}(a_1 + \bar h, a_2 + \bar k)hk </tex>
 +
 
 +
<tex> \bar h, \bar k </tex> — средние точки
 +
 
 +
<tex> \psi(k) = f(a_1 + h, a_2 + k) - f(a_1, a_2 + k) </tex>
 +
 
 +
<tex> \vartriangle^2 f(h, k) = f''_{x_2 x_1}(a_1 + \hat h, a_2 + \hat k)hk </tex>
 +
 
 +
<tex> f''_{x_2 x_1}(a_1 + \hat h, a_2 + \hat k) = f''_{x_1 x_2}(a_1 + \bar h, a_2 + \bar k) \Rightarrow f''_{x_2 x_1} = f''_{x_1 x_2} </tex>
 
}}
 
}}
 +
* Замечание 1:
 +
 +
Аналогично: <tex> i, j : 1 \le i, j \le m; i \ne j </tex>
 +
 +
<tex> \frac{\partial f}{\partial x_i}, \frac{\partial f}{\partial x_j} </tex> — опр. в окр. <tex> (\cdot) a; \frac{\partial^2 f}{\partial x_i \partial x_j}, \frac{\partial^2 f}{\partial x_j \partial x_i} </tex> — непр. в <tex> (\cdot) a </tex>
 +
 +
* Замечание 2:
 +
 +
Если <tex> f </tex> сущ. част. пр. <tex>k</tex>-того порядка в окр. <tex>(\cdot)a</tex> и все они непр. в <tex>(\cdot)a</tex>
 +
 +
Для <tex> \forall i_1 \ldots i_k </tex> — индексы <tex> \in \{ 1 \ldots m \} </tex>
 +
 +
и <tex> \forall j_1 \ldots \j_k </tex> — которые получаются из набора <tex> i_1 \ldots i_k </tex> перестановка
 +
 +
Верно: <tex> \frac{\partial^k f}{\partial x_{i_1} \ldots \partial x_{i_k}}(a) = \frac{\partial^k f}{\partial x_{j_1} \ldots \partial x_{j_k}}(a) </tex>
  
 
=== Полиномиальная формула ===
 
=== Полиномиальная формула ===
 
{{Лемма
 
{{Лемма
 
|statement=
 
|statement=
Если <tex> r \in \mathbb{Z}_+ </tex>, <tex> a </tex> — мультииндекс, то <tex> (a_1 + ... + a_m)^r = \sum_{\alpha: (\alpha) = r} \frac{r!}{\alpha!} a^{\alpha} </tex>
+
Если <tex> r \in \mathbb{Z}_+ </tex>, <tex> k </tex> — мультииндекс, <tex> a </tex> - вектор, то <tex> (a_1 + ... + a_m)^r = \sum_{k: (k) = r} \frac{r!}{k!} a^{k} </tex>
 +
|proof=
 +
Индукция по <tex>r</tex>
 +
 
 +
<tex> r = 1 </tex>
 +
 
 +
<tex> k = (0, 0, \ldots, \overbrace{1}^{k}, 0, \ldots); a_k \cdot \frac{1!}{0!0! \ldots 1!0! ...} = 1 </tex>
 +
 
 +
<tex> r = r + 1 </tex>
 +
 
 +
<tex> (a_1 + ... + a_m)^{r + 1} = (a_1 + ... + a_m) \cdot \sum \frac{r!}{k_1! ... k_m!} \cdot a_1^{k_{1}} ... a_m^{k_{m}} = </tex>
 +
 
 +
<tex> = \sum \frac{r!}{k_1! ... k_m!} \cdot a_1^{k_{1}+1} ... a_m^{k_{m}} + \sum \frac{r!}{k_1! ... k_m!} \cdot a_1^{k_{1}} a_2^{k_2 + 1} ... a_m^{k_{m}} + </tex><tex> \sum \frac{r!}{k_1! ... k_m!} \cdot a_1^{k_{1}} ... a_{m-1}^{k_{m - 1}} a_m^{k_{m} + 1} = </tex>
 +
 
 +
<tex> = \sum_{\beta : |\beta| = r + 1; \beta_1 \ge 1} \frac{r! \beta_1}{\beta_1!\beta_2!...\beta_m!} \cdot a_1^{\beta_1}...a_m^{\beta_m} + \sum_{\beta : |\beta| = r + 1; \beta_2 \ge 1} \frac{r! \beta_2}{\beta_1!\beta_2!...\beta_m!} \cdot a_1^{\beta_1}...a_m^{\beta_m} + </tex> <ещё <tex> m - k </tex> суммы> = <tex> \sum_{|b| = r + 1} \frac{r! (b_1 + ... + b_m)}{b_1! ... b_m!} \cdot a_1^{\beta_1}...a_m^{\beta_m} </tex>;
 +
 
 +
<tex> \beta_1 \ge 1 .. </tex> — это ограничение можно убрать, т.к. все слагаемые с <tex> \beta_1 = 0 </tex> имеют нулевой индекс
 +
 
 +
<tex> (k_1 + 1, k_2 ... k_m) \to (\beta_1 ... \beta_m) </tex>
 
}}
 
}}
 +
* Замечание 1
 +
 +
<tex> \sum_{(k_1...k_m); k_i \ge 0; k_1 + ... + k_m = r} \frac{r!}{k_1! ... k_m!} \cdot a_1^{k_{1}} ... a_m^{k_{m}} = </tex><tex> \sum_{i_1 = 1}^m \sum_{i_2 = 1}^m ... \sum_{i_r = 1}^m a_{i_1} a_{i_2} ... a_{i_r} </tex>
 +
 +
* Замечание 2
 +
 +
<tex> m = 2; k_1, k_2 = r - k_1 </tex>
 +
 +
<tex> \sum_{k_1 = 0}^{r} \frac{r!}{k_1!(r - k_1)!} \cdot a_1^{k_1} a_2^{r - k_1} = (a_1 + a_2)^r </tex>
  
 
=== Лемма о дифференцировании «сдвига» ===
 
=== Лемма о дифференцировании «сдвига» ===
Строка 529: Строка 629:
 
|statement=
 
|statement=
 
Пусть <tex> f: E \subset \mathbb{R}^m \to \mathbb{R} </tex>, <tex> E </tex> открыто в <tex> \mathbb{R}^m </tex>, <tex> \ a \in E, \ h \in \mathbb{R}^m </tex>, так, что <tex> \forall t \in [-1; 1] \ a + th \in E </tex>. Также <tex> f \in C^r(E) </tex>. Пусть <tex> \varphi (t) = f(a + th) </tex>. Тогда <tex> \forall t_0 \in (-1; 1) </tex> верно <tex> \varphi^{r} (t_0) = \sum_{\alpha: (\alpha) = r} \frac{r!}{\alpha!} f^{(\alpha)} (a + t_0 h) h^{\alpha} </tex>.
 
Пусть <tex> f: E \subset \mathbb{R}^m \to \mathbb{R} </tex>, <tex> E </tex> открыто в <tex> \mathbb{R}^m </tex>, <tex> \ a \in E, \ h \in \mathbb{R}^m </tex>, так, что <tex> \forall t \in [-1; 1] \ a + th \in E </tex>. Также <tex> f \in C^r(E) </tex>. Пусть <tex> \varphi (t) = f(a + th) </tex>. Тогда <tex> \forall t_0 \in (-1; 1) </tex> верно <tex> \varphi^{r} (t_0) = \sum_{\alpha: (\alpha) = r} \frac{r!}{\alpha!} f^{(\alpha)} (a + t_0 h) h^{\alpha} </tex>.
 +
|proof=
 +
Доказательства нет, есть пример, из которого можно придумать доказательство по индукции, наверное.
 
}}
 
}}
  
Строка 536: Строка 638:
 
|statement=
 
|statement=
 
Пусть <tex> r \in \mathbb{R}_+ </tex>, <tex> D </tex> открыто в <tex> \mathbb{R}^n </tex>, <tex> f \in C^{(r + 1)} (D), \ a, x \in \mathbb{R}^n, \ \overline{a, x} \subset D </tex>. Тогда существует такое <tex> \theta \in (0, 1) </tex>, что <tex dpi="150"> f(x) = \sum_{(k) \leqslant r} \frac{f^{(k)} (a) }{k!} (x - a)^k + \sum_{(k) = r + 1} \frac{f^{(k)} (a + \theta(x - a))}{k!} (x - a)^k </tex>.
 
Пусть <tex> r \in \mathbb{R}_+ </tex>, <tex> D </tex> открыто в <tex> \mathbb{R}^n </tex>, <tex> f \in C^{(r + 1)} (D), \ a, x \in \mathbb{R}^n, \ \overline{a, x} \subset D </tex>. Тогда существует такое <tex> \theta \in (0, 1) </tex>, что <tex dpi="150"> f(x) = \sum_{(k) \leqslant r} \frac{f^{(k)} (a) }{k!} (x - a)^k + \sum_{(k) = r + 1} \frac{f^{(k)} (a + \theta(x - a))}{k!} (x - a)^k </tex>.
 +
|proof=
 +
 +
<tex>\phi(t)=f(a+th), t\in{[-1;1]}</tex>
 +
 +
<tex>f(a+h) = \phi(1)</tex>
 +
 +
Разложили <tex>\phi(1)</tex> по одномерной формуле Тейлора в точке 0, используя лемму о дифференцировании сдвига, — получили то, что нужно.
 +
 
}}
 
}}
  
Строка 543: Строка 653:
 
{{Теорема
 
{{Теорема
 
|statement=
 
|statement=
Пусть <tex> r \in \mathbb{N} </tex>, <tex> D </tex> открыто в <tex> \mathbb{R}^n </tex>, <tex> f \in C^{(r)} (D), \ x \in D </tex>. Тогда <tex dpi="150"> f(x + h) = \sum_{(k) \leqslant r} \frac{f^{(k)} (x)}{k!} h^k + o(|h|^r), \ h \to \mathbb{O}_n </tex>.
+
Пусть <tex> r \in \mathbb{N} </tex>, <tex> D </tex> открыто в <tex> \mathbb{R}^n </tex>, <tex> f \in C^{(r + 1)} (D), \ x \in D </tex>. Тогда <tex dpi="150"> f(x + h) = \sum_{(k) \leqslant r} \frac{f^{(k)} (x)}{k!} h^k + o(|h|^r), \ h \to \mathbb{O}_n </tex>.
 
}}
 
}}
  
 
=== Теорема о пространстве линейных отображений ===
 
=== Теорема о пространстве линейных отображений ===
 +
{{Теорема
 +
|statement=
 +
<tex>(1) ||\ldots||_{m, n} </tex> — норма в пр-ве <tex> \mathcal{L}_{m, n} </tex>, то есть
 +
 +
<tex> 1. ||A|| \ge 0, ||A|| = 0 \Leftrightarrow A = \mathbb{O}_{m, n} </tex>
 +
 +
<tex> 2. \forall \lambda \in \mathbb{R} : ||\lambda A|| = |\lambda|\cdot||A|| </tex>
 +
 +
<tex> 3. ||A + B|| \leqslant ||A|| + ||B|| </tex>
 +
 +
<tex> (2) A \in \mathcal{L}_{m, n}, B \in \mathcal{L}_{n, k}: ||BA||_{m, k} \leqslant ||B||_{n, k} \cdot ||A||_{m, n} </tex>
 +
|proof=
 +
<tex>(1)</tex>
 +
 +
1. очевидно <tex>||A|| = 0; sup_{|x| \le 1}|Ax| = 0 \Rightarrow Ax \equiv 0 \Rightarrow A = \mathbb{O} </tex> // для <tex> x \in B(0, 1) </tex>
 +
 +
2. очевидно, св-ва <tex> sup </tex>. Википедия[http://ru.wikipedia.org/wiki/%D2%EE%F7%ED%E0%FF_%E2%E5%F0%F5%ED%FF%FF_%E8_%ED%E8%E6%ED%FF%FF_%E3%F0%E0%ED%E8%F6%FB_%EC%ED%EE%E6%E5%F1%F2%E2]
 +
 +
3. <tex> \forall x : |(A + B)x| = |Ax + Bx| \le |Ax| + |Bx| \le ||A||\cdot|x| + ||B||\cdot|x| </tex><tex> = (||A|| + ||B||)|x| \Rightarrow ||A + B|| \le C </tex> \\ <tex> ||A|| + ||B|| = C </tex>
 +
 +
<tex>(2)</tex>
 +
 +
<tex> |B(Ax)| \le ||B||\cdot|Ax| \le ||B||\cdot||A||\cdot|x| \Rightarrow ||BA|| \le C </tex> \\ <tex> ||B|| \cdot ||A|| = C </tex>
 +
}}
 +
 
=== Теорема Лагранжа для отображений ===
 
=== Теорема Лагранжа для отображений ===
 
{{Теорема
 
{{Теорема
 
|statement=
 
|statement=
Пусть <tex> D </tex> открыто в <tex> \mathbb{R}^n </tex>, отображение <tex> f: D \to \mathbb{R}^m </tex> дифференцируемо на <tex> D </tex>, <tex> \overline{a, b} \subset D </tex> (<tex> \overline{a, b} = \{a + t(b - a): t \in [0, 1]\} </tex> называется отрезком с концами <tex> a </tex> и <tex< b </tex>). Тогда найдётся такое <tex> \theta \in (0, 1) </tex>, что <tex> |f(b) - f(a)| \leqslant || f'(a + \theta(b - a)) || \cdot |b - a| </tex>.
+
<tex> F : E </tex> откр. <tex> \subset \mathbb{R}^m \rightarrow \mathbb{R}^n; </tex> дифф. <tex> E; a, b \in E </tex>
 +
 
 +
<tex> [a, b] = \{ c = a + t(b - a), t \in [0, 1] \} \subset E </tex>
 +
 
 +
Тогда: <tex> \exists c \in [a, b] : |F(b) - F(a)| \le ||F'(c)||\cdot|b - a| </tex>
 +
|proof=
 +
<tex> g(t) = F(a + t(b - a)), t \in [0, 1] \\ g'(t) = F'(a + t(b - a))\cdot(b - a) </tex> // <tex> |g(b) - g(a)| \le |g'(c)|\cdot|b - a| </tex>
 +
 
 +
<tex> ||F(b) - F(a)|| = |g(1) - g(0)| \le |F'(c)(b - a)| \le ||F'(c)||\cdot|b - a| </tex>
 
}}
 
}}
  
Строка 563: Строка 706:
  
 
3) <tex> ||B^{-1} - A^{-1}|| \leqslant \frac{||A^{-1}||}{||A^{-1}||^{-1} - ||B - A||} ||B - A|| </tex>.
 
3) <tex> ||B^{-1} - A^{-1}|| \leqslant \frac{||A^{-1}||}{||A^{-1}||^{-1} - ||B - A||} ||B - A|| </tex>.
 +
|proof=
 +
 +
Лемма: пусть <tex>\exists{c > 0} : \forall{x} |Bx| \ge c|x|</tex>
 +
 +
Тогда <tex>B</tex> — обратим, <tex>||B^{-1}|| \le \frac{1}{c}</tex>
 +
 +
Это правда, потому что <tex>\operatorname{Ker}{B} = \{0\}</tex>, значит, <tex>B</tex> — биекция(пусть <tex>B(x_1)=B(x_2): B(x_1)-B(x_2)=0 \Leftrightarrow B(x_1 - x_2) = 0 \Rightarrow x_1 = x_2</tex>)
 +
 +
Неравенство получается из <tex>|Bx| \ge c|x|</tex> заменой <tex>Bx=y, x = B^{-1}y</tex>
 +
 +
Само доказательство:
 +
 +
<tex>|Bx| = |Ax + (B-A)x| \ge |Ax| - |(B-A)x| \ge \frac{1}{||A^{-1}||}|x| - ||B-A|| \cdot |x| = (\frac{1}{||A^{-1}||} - ||B-A||) \cdot |x|</tex>
 +
 +
По условию теоремы множитель в последней части больше нуля, поэтому по лемме <tex>B</tex> обратим, по этой же лемме выполнено 2).
 +
 +
 +
<tex>||B^{-1} - A^{-1}|| = ||B^{-1}\cdot (A-B) \cdot A^{-1}|| \le ||B^{-1}||\cdot ||A-B|| \cdot ||A^{-1}|| \le \frac{||A^{-1}||}{||A^{-1}||^{-1} - ||B - A||} ||B - A||</tex>
 
}}
 
}}
  
 
=== Теорема о непрерывно дифференцируемых отображениях ===
 
=== Теорема о непрерывно дифференцируемых отображениях ===
 +
{{Теорема
 +
|statement=
 +
Пусть <tex> F : E \subset \mathbb{R}^m \rightarrow \mathbb{R}^n </tex>, где <tex> E </tex> открыто, дифференцируемо на <tex> E </tex>. Тогда эквивалентны утверждения:
 +
 +
<tex> I) F \in C^{1}(E) </tex>
 +
 +
<tex> II) F' : E \rightarrow \mathcal{L}_{m, n} </tex> — непрерывна.
 +
|proof=
 +
<tex> I \Rightarrow II </tex>
 +
 +
<tex> ||A|| \le \sqrt{\sum a_i^2}; A = (a_{ij}); </tex>
 +
 +
? <tex> F' </tex> непр. в <tex> (\cdot) \overline{X} </tex>
 +
 +
<tex> \forall \epsilon > 0 \exists \delta > 0 : \forall x : |x - \overline{x}| < \delta </tex>
 +
 +
<tex> ||F'(x) - F'(\overline{x})|| < \epsilon </tex>
 +
 +
<tex> ||F'(x) - F'(\overline{x})|| \le \sqrt{\sum(\frac{\partial f_i}{\partial x_j}(x) - \frac{\partial f_i}{\partial x_j}(\overline{x}))^2} </tex>
 +
 +
<tex> \forall \epsilon > 0 </tex> выберем <tex> \delta : |\frac{\partial f_i}{\partial x_j}(x) - \frac{\partial f_i}{\partial x_j}(\overline{x})| < \frac{\epsilon}{\sqrt{mn}}</tex>; при <tex> |x - \overline{x}| < \delta; i = 1 \ldots n; j = 1 \ldots m </tex>
 +
 +
<tex> II \Rightarrow I </tex>
 +
 +
<tex> F' </tex> — непрерывна. <tex> e_1 \ldots e_m </tex> — нормированный базис <tex>\mathbb{R}^m</tex>
 +
 +
<tex> F'(x)e_i =  \begin{pmatrix} \frac{\partial f_i}{\partial x_1}(x) \\  \ldots \\ \frac{\partial f_i}{\partial x_n}(x) \end{pmatrix}; </tex>
 +
 +
<tex> \begin{matrix} |F'(x)e_i| \le ||F'(x)|| \cdot 1 \\ |\frac{\partial f_i}{\partial x_j}(x)| \le |F'(x)e_i| \le ||F'(x)|| \end{matrix} </tex>
 +
 +
Точно также: <tex> |\frac{\partial f_i}{\partial x_j}(x) - \frac{\partial f_i}{\partial x_j}(\overline{x})| \le ||F'(x) - F'(\overline{x})|| </tex>
 +
}}
 +
 
=== Необходимое условие экстремума. Теорема Ролля ===
 
=== Необходимое условие экстремума. Теорема Ролля ===
 +
'''Необходимое условие экстремума:'''
 +
{{Теорема
 +
|statement=
 +
Пусть <tex> f: E </tex> открыто <tex> \subset \mathbb{R}^m \to \mathbb{R}; \ \ a </tex> — точка лок. экстремума. <tex> f </tex> — дифф. на <tex> E </tex>.
 +
 +
Тогда <tex> \nabla_a f = 0 </tex> (т.е. <tex> f'_{x_1}(a) = 0, \ldots, f'_{x_m}(a) = 0 </tex>)
 +
 +
|proof=
 +
Меняем <tex>f(a+l)</tex> на <tex>g(t)=f(a+tl)</tex>, по теореме Ферма из первого семестра <tex>g'(0)=0</tex>. Из этого следует, что все частные производные в точке a равны нулю, что нам и было нужно.
 +
}}
 +
'''Теорема Ролля:'''
 
{{Теорема
 
{{Теорема
 
|statement=
 
|statement=
Пусть <tex> f: D \subset \mathbb{R}^n \to \mathbb{R}, \ x_0 \in \operatorname{Int} D </tex> — точка экстремума <tex> f, \ k \in [1 : n] </tex>. Тогда если <tex> D_k f(x_0) </tex> существует, то <tex> D_k f(x_0) = 0 </tex>.
+
Пусть <tex> f: K </tex> компакт <tex> \subset \mathbb{R}^m \to \mathbb{R} </tex>, дифференцируемо на <tex> \operatorname{Int} K \ne 0 </tex>, <tex> f \equiv \operatorname{const} </tex> на <tex> \partial K </tex> (граница <tex> K </tex>), <tex> f </tex> — непр. на <tex> K </tex>.  
 +
 
 +
Тогда существует <tex> a \in \operatorname{Int} K: \ \nabla f(a) = 0 </tex>.
 +
|proof=
 +
Если <tex>f</tex> постоянна на <tex>K</tex>, то утверждение очевидно.
 +
Если нет, то по [[Участник:Katyatitkova/Матан#Теорема Вейерштрасса о непрерывном образе компакта. Следствия|теореме Вейерштрасса]] <tex>f</tex> на компакте достигает наибольшего или наименьшего значения в какой-то точке, а по необходимому условию экстремума в этой точке градиент равен нулю.
 
}}
 
}}
Теорема Ролля — ???
 
  
=== Лемма об оценке квадратичной форме и об эквивалентных нормах ===
+
=== Лемма об оценке квадратичной формы и об эквивалентных нормах ===
Наверное, это не совсем то
 
 
{{Утверждение
 
{{Утверждение
 
|statement=
 
|statement=
Если форма <tex> K </tex> положительно определена, то существует такое <tex> \gamma > 0 </tex>, что <tex> K(h) \geqslant \gamma |h|^2 </tex> для всех <tex> h \in \mathbb{R}^n </tex>.
+
1) Если квадратичная форма <tex> h </tex> положительно определена, то существует такое <tex> \gamma_h </tex>, что <tex> h(x) \ge \gamma_h |x|^2 </tex> для всех <tex> x \in \mathbb{R}^m </tex> <br>
 +
2) Пусть <tex> p : \mathbb{R}^m \to \mathbb{R}_+ </tex> — норма. Тогда <tex> \exists c_1, c_2 > 0 \ \forall x \ c_1 |x| \leqslant p(x) \leqslant c_2 |x| </tex>.
 +
|proof=
 +
1) <tex> \gamma_h = min_{|x| = 1}h(x) </tex>
 +
 
 +
(Сфера <tex> \{ x : |x| = 1 \} </tex> — компакт по [[Участник:Katyatitkova/Матан#Теорема Вейерштрасса о непрерывном образе компакта. Следствия|теореме Вейерштрасса]] <tex> \exists min </tex>)
 +
 
 +
<tex> x = 0 : \text{ok} </tex>
 +
 
 +
<tex> x \ne 0 : h(x) = h(|x| \cdot \frac{x}{|x|}) = |x|^2 \cdot h(\frac{x}{|x|}) \ge \gamma_h |x|^2 </tex>
 +
 
 +
<tex> h(tx) = t^2 h(x) </tex>
 +
 
 +
2) <tex> c_1 := min_{|x| = 1} p(x); c_2 := max_{|x| = 1} p(x); </tex> — по т. Вейерштрасса (т.к. <tex>p(x)</tex> — непр.)
 +
 
 +
<tex> x = 0 : \text{triv} </tex>
 +
 
 +
<tex> x \ne 0 : p(x) = p(|x| \cdot \frac{x}{|x|}) = |x| \cdot p(\frac{x}{|x|}) \begin{matrix} \le c_2|x| \\ \ge c_1|x| \end{matrix} </tex>
 
}}
 
}}
  
Строка 583: Строка 808:
 
{{Теорема
 
{{Теорема
 
|statement=
 
|statement=
Пусть <tex> D </tex> открыто в <tex> \mathbb{R}^n </tex>, <tex> f \in C^{(2)}(D), \ x_0 \in D </tex> — стационарная точка <tex> f </tex> (то есть <tex> \nabla f(x_0) = \mathbb{O}_n </tex>). Тогда справедливы следующие утверждения:
+
Пусть <tex> f = Е </tex> открыто в <tex> \mathbb{R}^m \to \mathbb{R} </tex>, дифф. на <tex> Е, a \in E </tex> — стационарная точка <tex> f </tex> (то есть <tex> \nabla f(a) = \mathbb{O}_m </tex>). <tex> d^2 f(a, h) = Q(h) </tex> — кв. форма.
 +
 
 +
Тогда справедливы следующие утверждения:
 +
 
 +
1) Если <tex> Q(h) </tex> положительно определённая, то <tex> a </tex> — точка минимума (локального).
 +
 
 +
2) Если <tex> Q(h) </tex> отрицательно определённая, то <tex> a </tex> — точка максимума (локального).
 +
 
 +
3) Если <tex> Q(h) </tex> не знакоопределённая, то <tex> a </tex> — не точка экстремума.
  
1) Если форма <tex> d^2 f(x_0) </tex> положительно определённая, то <tex> x_0 </tex> — точка строгого минимума <tex> f </tex>.
+
4) Если <tex> Q(h) </tex> положительно/отрицально опр. вырожденное, то (?) может быть макс., мин. требуется исследование
 +
|proof=
 +
<tex>(1) : f(a + h) = f(a) + \sum_{i = 1}^{m} f'_{x_i}(a) \cdot h_i + \frac{1}{2} \sum f''_{x_i x_j}(a + \theta h)h_i h_j </tex>
  
2) Если форма <tex> d^2 f(x_0) </tex> отрицательно определённая, то <tex> x_0 </tex> — точка строгого максимума <tex> f </tex>.
+
<tex> 2(f(a + h) - f(a)) = \sum_{i, j = 1}^{m}f''_{x_i x_j}(a)h_i h_j + \sum_{i, j = 1}^{m}(f''_{x_i x_j}(a + \theta h) - f''_{x_i x_j}(a))h_i h_j </tex> // <tex> |h_i| < |h| </tex>
  
3) Если форма <tex> d^2 f(x_0) </tex> неопределённая, то <tex> x_0 </tex> — не точка экстремума <tex> f </tex>.
+
Выберем <tex> U(a) </tex> так, чтобы при <tex> a + h \in U(a) </tex>
 +
 
 +
<tex> \sum |f''_{x_i x_j}(a + \theta h) - f(a)| \le \frac{\gamma}{2} </tex>
 +
 
 +
<tex> 2(f(a + h) - f(a)) \ge \gamma_Q |h|^2 - \frac{\gamma_Q}{2} |h|^2 > 0 </tex>
 +
 
 +
Таким образом <tex>a</tex> точка локального минимума
 +
 
 +
<tex>(3) : Q(h) </tex> — не знакоопределён. <tex> \begin{matrix} h \ne 0 & Q(h) \ge 0 \\ \bar h \ne 0 & Q(\bar h) < 0 \end{matrix} </tex>
 +
 
 +
<tex> 2(f(a + th) - f(a)) = Q(th) + \sum(f''_{x_i x_j}(a + \theta th) - f''_{x_i x_j}(a))th_i th_j = </tex>
 +
 
 +
<tex> = t^2 Q(h) + t^2 \sum(f''_{x_i x_j}(a + \theta th) - f''_{x_i x_j}(a))h_i h_j </tex>
 +
 
 +
<tex>Q(h) > 0; t^2 \sum(f''_{x_i x_j}(a + \theta th) - f''_{x_i x_j}(a))h_i h_j </tex> — при <tex> t \to 0 </tex> эта сумма из '?' б.м по модулю <tex> \le Q(h) </tex> при малых <tex> t </tex>
 
}}
 
}}
  
Строка 596: Строка 845:
 
|statement=
 
|statement=
 
Пусть <tex> F: O \subset \mathbb{R}^m \to \mathbb{R}^m </tex> — диффеоморфизм, <tex> x_0 \in \mathbb{R}^m , \ \det F'(x_0) \neq 0 </tex>. Тогда <tex> \exists c, \delta > 0 \ \forall h: |h| < \delta \  | F(x_0 + h) - F(x_0) | \geqslant c|h| </tex>
 
Пусть <tex> F: O \subset \mathbb{R}^m \to \mathbb{R}^m </tex> — диффеоморфизм, <tex> x_0 \in \mathbb{R}^m , \ \det F'(x_0) \neq 0 </tex>. Тогда <tex> \exists c, \delta > 0 \ \forall h: |h| < \delta \  | F(x_0 + h) - F(x_0) | \geqslant c|h| </tex>
 +
|proof=
 +
1) <tex> F </tex> — линейное. <tex> \exists (F'(x_0))^{-1} </tex>
 +
 +
<tex> F(x_0 + h) - F(x_0) = F(h); F'(x_0) \equiv F </tex>
 +
 +
<tex> |h| = |F^{-1} Fh| \le ||F^{-1}|| \cdot |Fh| </tex>
 +
 +
<tex> |Fh| \ge \frac{1}{||F^{-1}||} \cdot |h|; c := \frac{1}{||F^{-1}||} </tex>
 +
 +
2) <tex> F(x_0 + h) - F(x_0) = F'(x_0)h + \alpha(h)\cdot|h|; c = \frac{1}{||F'(x_0)^{-1}||} </tex>
 +
 +
<tex> |F(x_0 + h) - F(x_0)| \ge |F'(x_0)h| - |\alpha(h)|\cdot|h| \ge c|h| - |\alpha(h)|\cdot|h| </tex><tex> = (c - (\alpha(h))) \cdot |h| \ge^* \frac{c}{2}\cdot|h| </tex>
 +
 +
// <tex> \ge^*: \exists \delta > 0: </tex> при <tex> |h| < \delta: |\alpha(h)| < \frac{c}{2} </tex>
 
}}
 
}}
  
Строка 602: Строка 865:
 
|statement=
 
|statement=
 
Пусть <tex> F: O \subset \mathbb{R}^m \to \mathbb{R}^m </tex>, где <tex> O </tex> открыто — диффеоморфизм в <tex> O </tex>, <tex> \forall x \in O \ \det(F'(x)) \neq 0 </tex>. Тогда <tex> F(O) </tex> открыто.
 
Пусть <tex> F: O \subset \mathbb{R}^m \to \mathbb{R}^m </tex>, где <tex> O </tex> открыто — диффеоморфизм в <tex> O </tex>, <tex> \forall x \in O \ \det(F'(x)) \neq 0 </tex>. Тогда <tex> F(O) </tex> открыто.
 +
 +
* Замечание
 +
 +
1. Если <tex> O </tex> — лин. связное и <tex> F </tex> — непр. <tex> \Rightarrow F(O) </tex> — лин. связное
 +
 +
2. Непрерывность <tex> F : \forall A \subset \mathbb{R}^m : F^{-1}(A) </tex> — откр. [в <tex> O </tex>]
 +
|proof=
 +
<tex> x_0 \in O; y_0 = F(x_0) </tex> — внутрення точка <tex> F(O) </tex>?
 +
 +
<tex> \exists c, \delta : \forall |h| \le \delta \ |F(x_0 + h) - F(x_0)| \ge c|h| </tex>
 +
 +
при <tex> |h| = \delta \ F(x_0 + h) \ne F(x_0) = y_0 </tex>
 +
 +
<tex> dist(y_0, A) = inf_{a \in A} \rho (y_0, c)</tex>
 +
 +
Возьмем <tex> r = \frac{1}{2} dist(y_0, F(S(x_0, \delta))) </tex>(S — сфера, т. е. граница шара)
 +
 +
Утверждение: <tex> B(y_0, r) \subset F(O) </tex>
 +
 +
Т.е.: <tex> \forall y \in B(y_0, r) \ \exists x \in B(x_0, \delta) \ F(x) = y </tex>
 +
 +
<tex> \varphi(x) = |F(x) - y|^2 = (F_1(x_1...x_m) - y_1)^2 + (F_2 - y_2)^2 + \ldots + (F_m - y_m)^2; </tex> <tex> x \in B(x_0, \delta</tex>
 +
 +
<tex> min \varphi </tex> — внутри <tex> B(x_0, \delta) </tex>
 +
 +
В точке <tex>x_0: \varphi(x_0) = |y_0 - y|^2 < r^2 </tex>.
 +
 +
На сфере <tex> S(x_0, \delta) </tex>: <tex> \varphi(x) = |F(x) - y|^2 \ge (\overbrace{|F(x) - y_0|}^{ \ge 2r} - \overbrace{|y - y_0|}^{ < r })^2 \ge r^2 </tex>
 +
 +
<tex> \varphi </tex> — имеет <tex> (\cdot) min </tex> внутри шара <tex> B(x_0, \delta) </tex> по [[Участник:Katyatitkova/Матан#Теорема Вейерштрасса о непрерывном образе компакта. Следствия|теореме Вейерштрасса]]
 +
 +
<tex> \begin{cases}  2(F_1(x_1...x_m) - y_1)\frac{\partial F_1}{\partial x_1} + 2(F_2(x_1...x_m) - y_2)\frac{\partial F_2}{\partial x_1} + \ldots + 2(F_m() - y_m)\frac{\partial F_m}{\partial x_1} = 0 \\ \ldots \\ 2(F_1(x_1...x_m) - y_1)\frac{\partial F_1}{\partial x_m} + \ldots + 2(F_m() - y_m)\frac{\partial F_m}{\partial x_m} = 0 \end{cases} </tex>
 +
 +
<tex> det(\frac{\partial F_i}{\partial x_j}) \ne 0 \Rightarrow </tex> в точке минимума <tex> \begin{matrix} F_1(x_1...x_m) = y_1 \\ \ldots \\F_m(x_1..x_m) = y_m \end{matrix} </tex>(у системы есть только тривиальное решение)
 
}}
 
}}
  
Строка 607: Строка 904:
 
{{Теорема
 
{{Теорема
 
|statement=
 
|statement=
Пусть <tex> F: O \subset \mathbb{R}^m \to \mathbb{R}^m, \ F \in C^r(O) </tex>, <tex> F </tex> — обратима и невырождена, <tex> \forall x \in O \det(F'(x)) \neq 0 </tex>. Тогда:
+
Пусть <tex> F: O \subset \mathbb{R}^m \to \mathbb{R}^m, \ F \in C^r(O) </tex>, <tex> F </tex> — обратима и её производная невырождена, <tex> (\forall x \in O \ \det(F'(x))) \neq 0 </tex>.
 +
 
 +
Тогда:
  
 
1) <tex> F^{-1} \in C^r </tex>
 
1) <tex> F^{-1} \in C^r </tex>
  
2) <tex> y_0 = F(x_0), \ (F^{-1})' (y_0) = (F'(x_0))^{-1} (y_0) </tex>
+
2) <tex> y_0 = F(x_0), \ (F^{-1})' (y_0) = (F'(x_0))^{-1} </tex>
 +
|proof=
 +
 
 +
1) <tex> r = 1 </tex>
 +
 
 +
<tex>F(O) = O' </tex> — открытое
 +
 
 +
Пусть <tex> S = F^{-1}, S : O' \to O</tex>
 +
 
 +
Пусть <tex> U \subset O</tex> — открытое, тогда <tex> S^{-1}(U) </tex> — открытое.
 +
 
 +
* <tex> T : X \to Y</tex> — непрерывное отображение <tex> \Leftrightarrow \forall U \subset Y : T^{-1}(U) </tex> — открыто. // Мне кажется, из определения диффеоморфизма и предыдущей теоремы следует, что обратное отображение тоже диффеоморфизм и предыдущие строчки и так очевидны.
 +
 
 +
<tex> y_0 = F(x_0); x_0 = S(y_0) </tex>
 +
 
 +
<tex> y - y_0 = F(x) - F(x_0) = A(x - x_0) + o(x - x_0) </tex>
 +
 
 +
<tex> S(y) - S(y_0) = x - x_0 = A^{-1}(y - y_0) - A^{-1} o(x - x_0) </tex>
 +
 
 +
* <tex> T </tex> — диффеоморфизм, матрица <tex>T'(x_0)</tex> невырождена <tex>\Rightarrow</tex> <tex> \exists c, \delta \ \forall x \in B(x_0, \delta) \ |T(x) - T(x_0)| > c|x - x_0| </tex> // По лемме о почти локальной инъективности
 +
 
 +
Возьмём <tex> c, \delta </tex> из леммы.
 +
 
 +
Пусть <tex> T = F'(x_0) </tex>
 +
 
 +
<tex> y - y_0 = T(x - x_0) + \alpha(x)|x - x_0| </tex>
 +
 
 +
<tex> S(y) - S(y_0) = T^{-1}(y - y_0) - \overbrace{T^{-1} \alpha(x) |S(y) - S(y_0)|}^{? o(y - y_0)} </tex>
 +
 
 +
Можно считать, что <tex> y </tex> близко к <tex> y_0 </tex>, так что <tex> |x - x_0| = |S(y) - S(y_0)| < \delta </tex>
 +
 
 +
<tex> | \ T^{-1} \alpha(x) \cdot |x - x_0| \ | = |T^{-1}(\alpha(x))|\cdot|x - x_0| \le </tex><tex> \| T^{-1} \| \cdot |\alpha(x)| \cdot \frac{1}{c} |F(x) - F(x_0)| \le \frac{\| T^{-1} \|}{c}|y - y_0|\cdot|\alpha(x)| </tex>
 +
 
 +
<tex>// y \to y_0; x \to x_0; \alpha(x) \to 0 </tex>
 +
 
 +
<tex> y \mapsto S(y) = x \mapsto F'(x) = T \mapsto T^{-1} = S'(y) </tex>
 +
 
 +
2) <tex> r </tex> — любое. (без доказательства)
 
}}
 
}}
  
Строка 617: Строка 953:
 
{{Теорема
 
{{Теорема
 
|statement=
 
|statement=
Пусть <tex> F: O \subset \mathbb{R}^m \to \mathbb{R}^m </tex>, где <tex> O </tex> открыто, <tex> F \in C^1(O, \mathbb{R}^m) </tex> (т.е. <tex> F </tex> 1 раз непрерывно дифференцируемо на <tex> O </tex>, а его первая производная непрерывна на <tex> D </tex>), <tex> x_0 \in O, \ \det F'(x_0) \neq 0 </tex>. Тогда <tex> \exists U(x_0): \ F | _U </tex> — диффеоморфизм (<tex> F |_U </tex> или <tex> F|U </tex> — сужение отображения <tex> F </tex> на множество <tex> U </tex>).
+
Пусть <tex> F: O \subset \mathbb{R}^m \to \mathbb{R}^m </tex>, где <tex> O </tex> открыто; <tex> F \in C^1(O, \mathbb{R}^m); x_0 \in O; \det F'(x_0) \ne 0 </tex>
 +
 
 +
Тогда <tex> \exists U(x_0): \ F |_U </tex> — диффеоморфизм (<tex> F |_U </tex> или <tex> F|U </tex> — сужение отображения <tex> F </tex> на множество <tex> U </tex>).
 +
|proof=
 +
Нужно проверить лишь: <tex> \exists U(x_0) : F|_U </tex> — обратима
 +
 
 +
[так как можно считать что <tex> \det F'(x) \ne 0 </tex> на <tex> U(x_0) \Rightarrow F(U(x_0)) </tex> открыто и <tex> F^{-1} </tex> определено на открытом множестве и дифференцируемо по предыдущим теоремам]
 +
 
 +
<tex> |F(x) - F(y)| \ge^{?} |x - y| </tex> // Это какая-то хрень, к тому же она в конце не доказана. Надо проверить, что <tex>\forall{x \neq y} |F(x) - F(y)| > 0</tex>, тогда отображение будет биекцией.
 +
 
 +
<tex> \exists c \ \forall h \in \mathbb{R}^m : |F'(x_0)h| \ge c|h|; \ U = B(x_0, r) \subset O </tex>
 +
 
 +
<tex> \begin{matrix} 1: \forall x \in U & \det F'(x) \ne 0 \\ 2: \forall x \in U & \| F'(x) - F'(x_0) \| < \frac{c}{4} \end{matrix} </tex>
 +
 
 +
<tex> x, y \in B(x_0, r); y = x + h </tex>
 +
 
 +
<tex> F(y) - F(x) = ( F(x + h) - F(x) - F'(x)h ) + ( F'(x) - F'(x_0) )h + F'(x_0)h </tex>
 +
 
 +
<tex> |F(y) - F(x)| \ge |F'(x_0)h| - |F(x + h) - F(x) - F'(x)h| - |(F'(x) - F'(x_0))h| \ge </tex>
 +
 
 +
<tex> \ge c|h| - sup_{t \in [x, x + h]} \| F'(t) - F'(x) \| \cdot |h| - \| F'(x) - F'(x_0) \| \cdot |h| \ge c|h| - \frac{c}{4}|h| - \frac{c}{4}|h| = \frac{c}{2}|h| > 0</tex>
 
}}
 
}}
 +
* Замечание
 +
 +
<tex> \det F' \ne 0 </tex> — нужно для дифференцируемости.
 +
 +
<tex> F : \mathbb{R} \to \mathbb{R}; x \mapsto x^3; F^{-1} </tex> — не дифференцируемо в нуле
  
 
=== Теорема о неявном отображении ===
 
=== Теорема о неявном отображении ===
Строка 627: Строка 988:
 
1) существуют открытые <tex> P \subset \mathbb{R}^m, \ Q \subset \mathbb{R}^n, \ a \in P, \ b \in Q </tex>, и существует единственное <tex> \varphi: P \to Q, \varphi \in C^r </tex>, что <tex> \forall x \in P \ F(x, \varphi(x) ) = 0 </tex>
 
1) существуют открытые <tex> P \subset \mathbb{R}^m, \ Q \subset \mathbb{R}^n, \ a \in P, \ b \in Q </tex>, и существует единственное <tex> \varphi: P \to Q, \varphi \in C^r </tex>, что <tex> \forall x \in P \ F(x, \varphi(x) ) = 0 </tex>
  
2) <tex> \varphi'(x) = [F'_y (x, \varphi(x) ) ]^{-1} \cdot F'_x(x, \varphi(x)) </tex>
+
'''Раньше тут был забыт минус!'''
 +
2) <tex> \varphi'(x) = -[F'_y (x, \varphi(x) ) ]^{-1} \cdot F'_x(x, \varphi(x)) </tex>
 +
 
 +
|proof=
 +
 
 +
Пусть <tex>\Phi(x, y) = (x, F(x,y))</tex>.
 +
 
 +
<tex>\Phi(a, b) = (a, 0)</tex>
 +
 
 +
<tex>\Phi{'} = \begin{pmatrix} E_n & O \\ F'_x & F'_y \end{pmatrix}</tex>.
 +
 
 +
<tex>\det{\Phi'} = \det{F'_y} \neq 0</tex>
 +
 
 +
По теореме о локальной обратимости <tex>\exists{U(a,b)}</tex> — такая, что <tex>\Phi</tex> — диффеоморфизм в данной окрестности.
 +
 
 +
Тогда существует обратное отображение <tex>\Psi(u, v) = (u, H(u, v))</tex>.
 +
 
 +
Почти очевидно, что <tex>\varphi(x) = H(x, 0)</tex>.
 +
 
 +
Берем производную — получаем 2): <tex>F'(x, \varphi(x)) = F'_x + F'_{y}\varphi{'} = 0</tex>
 
}}
 
}}
  
Строка 644: Строка 1024:
  
 
2.2) <tex> \nabla f_1, ... , \nabla f_{m - k} </tex> — линейно независимые
 
2.2) <tex> \nabla f_1, ... , \nabla f_{m - k} </tex> — линейно независимые
 +
|proof=
 +
<tex> 1 \Rightarrow 2 </tex>
 +
 +
<tex> \Phi : \Omega \to \mathbb{R}^m </tex> — параметризация <tex> C^r; \ p = \Phi(t_0); \ \Phi'(t_0) </tex> — матрица <tex> m \times k </tex>
 +
 +
<tex> Rg \Phi'(t_0) = k </tex> — реализуется на первых <tex> k </tex> степенях
 +
 +
<tex> \det( \frac{\partial \Phi_i}{\partial U_j} (t_0) ) \ne 0; \ L : \mathbb{R}^m \mapsto \mathbb{R}^k; \ (x_1 ... x_m) \mapsto (x_1 ... x_k) </tex>
 +
 +
<tex> 2 \Rightarrow 1 </tex>
 +
 +
Очевидно: <tex> (L \circ \Phi)'(p) </tex> — невырожденно.
 +
 +
<tex> \Phi = (\Phi_1 ... \Phi_m); L \circ \Phi = (\Phi_1 ... \Phi_k) </tex>
 +
 +
<tex> \exists W(t_0) : L \circ \Phi </tex> — диффеоморфизм на <tex> W(t_0) </tex>
 +
 +
<tex> V = (L \circ \Phi)(W) \Rightarrow L </tex> взаимно однозначное отображение <tex> \Phi(W) </tex> на <tex> V </tex>
 +
 +
<tex> \Psi_1 = (L \circ \Phi)^{-1}; \ H : V \to \mathbb{R}^{m - k}; \ \Phi(\Psi(V)) = (V, H(V)) </tex>
 +
 +
<tex> \Phi(W) </tex> — открыто в <tex> M \Rightarrow \Phi(W) </tex> — реал. как <tex> G \cap M, \ G </tex> — откр. в <tex> \mathbb{R}^m </tex>
 +
 +
<tex> G := V \times \mathbb{R}^{m - k}; \ \tilde{U} = G \cap G_1 </tex>
 +
 +
<tex> \begin{cases} f_1 = H_1 - X_{k + 1} \\ \ldots \\ f_{m - k} = H_{m - k} - X_m \end{cases} </tex>
 +
 +
<tex> \begin{matrix} \nabla f_1 = (\frac{\partial H_1}{\partial x_1} \cdots \frac{\partial H_1}{\partial x_k}, 1, 0, \ldots, 0 ) \\ \cdots \\ \nabla f_{m - k} = ( \frac{\partial H_{m - k}}{\partial x_1} \cdots \frac{\partial H_{m - k}}{\partial x_k}, 0, \ldots, 0, 1 ) \end{matrix} </tex>
 
}}
 
}}
  
Строка 654: Строка 1062:
 
\Phi(a) = \mathbb{O}_n
 
\Phi(a) = \mathbb{O}_n
 
\end{cases} </tex>
 
\end{cases} </tex>
 +
|proof=
 +
Пусть ранг реализуется на столбцах <tex> x_{m + 1}, \ldots, x_{m + n} </tex>. Переобозначим <tex> y_1 = x_{m + 1}; \ldots; y_n = x_{m + n} </tex>.
 +
 +
По теореме о неявном отображении: <tex> \exists \Psi: U(a_x) \rightarrow W(a_0) \\ \forall x \in U(a_x) \ \Phi(x, \Psi(x)) = 0  </tex>
 +
 +
<tex> x \mapsto (x, \Psi(x)) </tex> — гл. параметризация
 +
 +
<tex> g(x) = f(x, \Psi(x)) </tex>; Точка <tex> a_x </tex> — лок. экстремум <tex> g' </tex>.
 +
 +
<tex> f'_x(a) + f'_y(a) \cdot \Psi'(a_x) = 0 </tex> — необходимое усл. экстремума в матр. форме.
 +
 +
<tex> \Phi'_x(a) + \Phi'_y(a) \cdot \Psi'(a_x) = 0 </tex>
 +
 +
<tex> \forall \lambda \in \mathbb{R}^n : \ \lambda \Phi'_x(a) + \lambda \Phi'_y(a) \cdot \Psi'(a_x) = 0 </tex>
 +
 +
<tex> (f'_x(a) + \lambda \Phi'_x(a)) + (f'_y(a) + \lambda \Phi'_y(a)) \cdot \Psi'(a_x) = 0 </tex>
 +
 +
<tex> \lambda := -(f'_y(a))(\Phi'_y(a))^{-1} </tex>
 +
 +
При таком <tex> \lambda : </tex>
 +
 +
<tex> \begin{cases} f'_x(a) + \lambda \Phi'_x(a) = 0 \\ f'_y(a) + \lambda \Phi'_y(a) = 0 \\ \Phi(a) = 0 \end{cases} </tex>
 
}}
 
}}
  
Строка 660: Строка 1090:
 
|statement=
 
|statement=
 
Пусть <tex> A \in \mathcal{L}_{m, n} </tex>. Тогда <tex> || A || = \max \{\sqrt{\lambda}, \lambda </tex> — собственное число <tex> A^T \cdot A \} </tex>.
 
Пусть <tex> A \in \mathcal{L}_{m, n} </tex>. Тогда <tex> || A || = \max \{\sqrt{\lambda}, \lambda </tex> — собственное число <tex> A^T \cdot A \} </tex>.
 +
|proof=
 +
<tex> ||A||^2 = max_{|x| = 1}|Ax|^2 = max_{|x| = 1} \langle Ax, Ax \rangle = max_{|x| = 1}\langle A^tAx, x \rangle </tex>
 
}}
 
}}
  
 
=== Простейшие свойства интеграла векторного поля по кусочно-гладкому пути ===
 
=== Простейшие свойства интеграла векторного поля по кусочно-гладкому пути ===
{{Теорема
 
|statement=
 
 
1) Линейность по векторному полю: <tex> I(\alpha V_1 + \beta V_2, \gamma) = \alpha I(V_1, \gamma) + \beta I(V_2, \gamma) </tex>.
 
1) Линейность по векторному полю: <tex> I(\alpha V_1 + \beta V_2, \gamma) = \alpha I(V_1, \gamma) + \beta I(V_2, \gamma) </tex>.
  
2) Аддитивность при дроблении пути: если раздробили путь <tex> \gamma </tex> на <tex> \gamma_1 </tex> и <tex> \gamma_2 </tex>, то <tex> I(V, \gamma) = I(V, \gamma_1) + I(V, \gamma_2) </tex>.
+
<tex> \int_{a}^{b} \langle (\alpha V_1 + \beta V_2), \gamma{'} \rangle dt </tex> — по линейному скалярному произведению
  
3) Замена параметра: если <tex> \varphi: [p; q] \to [a; b] </tex> — гладкая, <tex> \varphi(p) = a, \ \varphi(q) = b </tex>, <tex> \gamma: [a; b] \to \mathbb{R}^m </tex>, <tex> \tilde{\gamma} = \gamma \circ \varphi: [p; q] \to \mathbb{R}^m </tex>, то <tex> I(V, \gamma) = I(V, \tilde{\gamma}) </tex>.
+
2) Аддитивность при дроблении пути:
 +
 
 +
<tex> \gamma : [a, b] \to \mathbb{R}^m; \ c \in [a, b] </tex>
 +
 
 +
<tex> \gamma_1 : [a, c] \to \mathbb{R}^m; \ t \mapsto \gamma(t); \ \gamma_2 : [c, b] \to \mathbb{R}^m </tex>
 +
 
 +
<tex> I(V, \gamma) = I(V, \gamma_1) + I(V, \gamma_2) </tex>.
 +
 
 +
<tex> \int_{a}^{b} ... = \int_a^c + \int_c^b </tex>
 +
 
 +
3) Замена параметра: если <tex> \varphi: [p; q] \to [a; b] </tex> — гладкая, <tex> \varphi(p) = a, \ \varphi(q) = b </tex>, <tex> \gamma: [a; b] \to \mathbb{R}^m </tex>, <tex> \tilde{\gamma} = \gamma \circ \varphi: [p; q] \to \mathbb{R}^m </tex> <tex> s \mapsto \gamma(\varphi(s)) </tex>
 +
 
 +
Тогда <tex> I(V, \gamma) = I(V, \tilde{\gamma}) </tex>.
 +
 
 +
<tex> I(V, \gamma) = \int_a^b \langle V(\gamma(t)), \gamma{'}(t) \rangle dt =_{t = \varphi(s)} </tex><tex> \int_a^b \langle V (\gamma(\varphi (s))), \gamma{'}(\varphi (s)) \varphi'(s) \rangle ds = \int_p^q \langle V(\tilde{\gamma}(s)), \tilde{\gamma}'(s) \rangle ds </tex>
  
 
4) Пусть <tex> \gamma_1: [a; b] \to \mathbb{R}^m, \ \gamma_2: [c; d] \to \mathbb{R}^m, \ \gamma_1(b) = \gamma_2(c), \ \gamma = \gamma_2 \gamma_1 </tex> — произведение путей:
 
4) Пусть <tex> \gamma_1: [a; b] \to \mathbb{R}^m, \ \gamma_2: [c; d] \to \mathbb{R}^m, \ \gamma_1(b) = \gamma_2(c), \ \gamma = \gamma_2 \gamma_1 </tex> — произведение путей:
Строка 675: Строка 1119:
 
<tex> \gamma: [a; b + d - c] \to \mathbb{R}^m = \begin{cases}
 
<tex> \gamma: [a; b + d - c] \to \mathbb{R}^m = \begin{cases}
 
\gamma_1(t), \ t \in [a; b] \\
 
\gamma_1(t), \ t \in [a; b] \\
\gamma_2(t - b + c), \ t \in [a; b + d - c]
+
\gamma_2(t - b + c), \ t \in [b; b + d - c]
\end{cases} </tex>,
+
\end{cases} </tex>
 +
 
 +
то <tex> I(V, \gamma_2 \gamma_1) = I(V, \gamma_1) + I(V, \gamma_2) </tex>.
 +
 
 +
<tex> \int_a^{b + d - c} \langle V(\gamma(t)), \gamma{'}t \rangle dt = \int_a^b + \int_b^{b + d - c} </tex> \\ заменить параметр <tex> s = t - b + c; s \in [c, d] </tex>
 +
 
 +
<tex> \gamma : [a, b] \to \mathbb{R}^m; \ \gamma_- </tex> — противоположный путь (в обратную сторону)
 +
 
 +
<tex> \gamma_-(t) = \gamma(b + a - t), t \in [a, b] </tex>
 +
 
 +
<tex> I(V, \gamma_-) = -I(V, \gamma) </tex>
 +
 
 +
<tex> \int_a^b \langle V(\gamma(b - a - t)), \gamma_-(t) \rangle dt = \int \langle V (\gamma(s)), \gamma{'}(s) \rangle ds </tex>
  
то <tex> I(V, \gamma) = I(V, \gamma_1) + I(V, \gamma_2) </tex>.
+
5) Оценка интеграла:
 +
{{
 +
Теорема
 +
|statement=
 +
<tex> | \int\limits_{a}^{b} (V_1 dx_1 + ... + V_m dx_m) | \leqslant \max_{x \in t_{\gamma}} |V(x)| \cdot L(\gamma) </tex>, где <tex> L(\gamma) </tex> — длина пути.
  
5) Оценка интеграла: <tex> | \int\limits_{a}^{b} (V_1 dx_1 + ... + V_m dx_m) | \leqslant \max |V(x)| \cdot L(\gamma) </tex>, где <tex> L(\gamma) </tex> — длина пути.
+
<tex> \gamma : [a, b] \to \mathbb{R}^m; L_{\gamma} = \gamma [a, b] \subset \mathbb{R}^m </tex>
 +
|proof=
 +
<tex> | \int_a^b \sum V_i (\gamma(t)) \cdot \gamma{'}_i(t) dt | \le \int_a^b |...| dt \le \int_a^b \sqrt{\sum V_i^2(\gamma(t))} \sqrt{\sum \gamma_i^{'2}(t)} dt = \int_a^b |V(\gamma(t))| \cdot |\gamma{'}(t)|  \le max_{x \in L_{\gamma}} (V(x)) \cdot \int_a^b |\gamma{'}(t) dt| </tex>
 
}}
 
}}
  
Строка 686: Строка 1148:
 
{{Теорема
 
{{Теорема
 
|statement=
 
|statement=
Пусть <tex> V: O \to \mathbb{R}^m </tex> потенциально, <tex> f </tex> — потенциал <tex> V </tex>, <tex> \gamma[a;b] \to 0 </tex>. Тогда <tex> \int\limits_{\gamma} (V_1 dx_1 + ... V_m dx_m) = f(\gamma(b)) - f(\gamma(a)) </tex>.
+
Пусть <tex> V: O \to \mathbb{R}^m </tex> потенциально, <tex> f </tex> — потенциал <tex> V </tex>, <tex> \gamma[a;b] \to O </tex> — кусочно гладкий.
 +
 
 +
Тогда <tex> \int\limits_{\gamma} (V_1 dx_1 + ... V_m dx_m) = f(\gamma(b)) - f(\gamma(a)) </tex>.
 +
|proof=
 +
1) <tex> \int\limits_{\gamma} \sum V_k d x_k = \int\limits_{a}^{b} (V_1(\gamma(t))\cdot\gamma'_1 + \ldots + V_m(\gamma(t))\cdot\gamma'_m) = f(\gamma(t))|_a^b </tex> — доказано для гладкого пути
 +
 
 +
\\ <tex> V_1(\gamma(t))\cdot\gamma'_1 + \ldots + V_m(\gamma(t))\cdot\gamma'_m = f(\gamma(t))' </tex> <tex> = f(\gamma_1(t)\ldots\gamma_m(t))' = \frac{\partial f}{\partial x_1}\cdot\gamma'_1 + \ldots + \frac{\partial f}{\partial x_m}\cdot\gamma'_m </tex>
 +
 
 +
\\ <tex> \frac{\partial f}{\partial x_1} = V_1; \ldots; \frac{\partial f}{\partial x_m} = V_m </tex>
 +
 
 +
2) <tex> a = t_0 < t_1 < \ldots < t_n = b </tex>
 +
 
 +
<tex> \gamma|_{[t_{k-1}, t_{k}]} </tex> — гладкий
 +
 
 +
<tex> \int\limits_{\gamma}\sum_k V_k d x_k = \sum_k (\int\limits_{t_k-1}^{t_k} \sum_i V_i d \gamma_i) = </tex><tex> \sum(f(\gamma(t_k)) - f(\gamma(t_{k-1}))) = f(\gamma(b)) - f(\gamma(a)) </tex>
 
}}
 
}}
  
 
=== Характеризация потенциальных векторных полей в терминах интегралов ===
 
=== Характеризация потенциальных векторных полей в терминах интегралов ===
=== Лемма о дифференцировании интеграла по параметру ===
+
{{
{{Лемма
+
Теорема
 
|statement=
 
|statement=
Пусть <tex> f: [a; b] \times [c; d] \to \mathbb{R}, \ f(x, y) </tex> — непрерывна, дифференцируема по <tex> y </tex> при любых <tex> x </tex> и <tex> f'_y </tex> непрерывна на промежутке. Пусть <tex> \Phi(y) = \int\limits_a^b f(x, y) dx, \ y \in [c, d] </tex>. Тогда <tex> \Phi(y) </tex> дифференцируема и <tex> \Phi'(y) = \int\limits_a^b f'_y(x, y) dx </tex>.
+
Если <tex> V : O \to \mathbb{R}^m </tex> тогда эквиваленты следующие утверждение:
}}
+
 
 +
1) V потенциально в <tex> O </tex>
 +
 
 +
2) Интеграл <tex> V </tex> не зависит от пути (в обл. <tex> O </tex>)
 +
 
 +
3) <tex> \forall \gamma : [a, b] \to O, \ \gamma(a) = \gamma(b); \ \int_{\gamma} \sum V_i dx_i = 0 </tex>
 +
|proof=
 +
<tex> 1 \Rightarrow 2 </tex> — формула [[Участник:Yulya3102/Матан#Формула Ньютона-Лейбница для кусочно-непрерывных функций|Ньютона-Лейбница]]
 +
 
 +
<tex> 2 \Rightarrow 3 </tex> — очевидно
 +
 
 +
<tex> \gamma </tex> — петля; <tex> \gamma_1(t) \equiv \gamma(a) </tex>
 +
 
 +
<tex> \int_{\gamma_1} \sum V_i dx_i = 0 = \int_{\gamma} \sum V_i dx_i </tex>
 +
 
 +
<tex> 3 \Rightarrow 2 </tex> — очевидно
 +
 
 +
<tex> \gamma := \gamma_{2-} \cdot \gamma_1; \ 0 = \int_{\gamma} \sum V_i dx_i = \int_{\gamma_{2-}} + \int_{\gamma_1} = \int_{\gamma_1} - \int_{\gamma_2} </tex>
 +
 
 +
<tex> 2 \Rightarrow 1 </tex>
 +
 
 +
Фиксируем точку <tex> x_0 \in O; \ \forall x \in O </tex>
 +
 
 +
Возьмём как-нибудь путь <tex> \gamma_x </tex> из <tex> x_0 </tex> в <tex> x </tex>
 +
 
 +
<tex> f(x) := \int_{\gamma_x} \sum V_i dx_i; f </tex> — потенциал?
 +
 
 +
Докажем, что <tex> \frac{\partial f}{\partial x_1} = V_1 </tex> (аналогично <tex> \frac{\partial f}{\partial x_i} = V_i; \ i = 2...m </tex>)
 +
 
 +
Выберем <tex> B(x, r) \subset O </tex>
 +
 
 +
<tex> |h| < r; \ t \mapsto (x_1 + th, x_2 ... x_m); \ \gamma'_h(t) = (h, 0, ..., 0) </tex>
 +
 
 +
<tex> f(x_1 + h, x_2 ... x_m) - f(x) = \int_{\gamma_h \gamma_x} \sum V_i dx_i - \int_{\gamma_x} \sum V_i dx_i = </tex>
  
=== Необходимое условие потенциальности гладкого поля. Лемма Пуанкаре ===
+
<tex>= \int_{\gamma_h} \sum V_i dx_i = \int_0^1 V_1(x_1 + th, x_2 ... x_m)h dt  = </tex> [[Участник:Yulya3102/Матан#Теорема о среднем. Следствия|теорема о среднем]] <tex> = V_1(x_1 + \Theta h, x_2 ... x_m)h; \ \Theta \in [0, 1] </tex>
{{Теорема
 
|statement=
 
Пусть <tex> V </tex> — гладкое потенциальное векторное поле в <tex> O </tex>. Тогда <tex> \forall x \in O \ \frac{\partial V_i}{\partial x_j} = \frac{\partial V_j}{\partial x_i}, \ i, j \in [1 : m] </tex>
 
}}
 
  
{{Лемма
+
<tex> \frac{f(x_1 + h, ... x_m) - f(x)}{h} = V_1(x_1 + \Theta h, ...) \to V_1(x) </tex>
|statement=
 
Пусть <tex> O \subset \mathbb{R}^m </tex> — выпуклая, <tex> V </tex> — векторное поле в <tex> O </tex>, гладкое и <tex> \forall x \forall i, j \ \frac{\partial V_i}{\partial x_j} = \frac{\partial V_j}{\partial x_i} </tex>. Тогда <tex> V </tex> — потенциальное.
 
 
}}
 
}}
  
=== Лемма о гусенице ===
+
=== Лемма о дифференцировании интеграла по параметру ===
 
{{Лемма
 
{{Лемма
 
|statement=
 
|statement=
Пусть <tex> \gamma: [a, b] \to O </tex>. Тогда существуют дробление <tex> a = t_0 < t1 < ... < t_n = b </tex> и шары <tex> B_1, ..., B_n \subset O </tex>, что <tex> \gamma [t_{k - 1}, t_k] \subset B_k, \ k \in [1 : n] </tex>.
+
Пусть <tex> f: [a; b] \times [c; d] \to \mathbb{R}, \ f(x, y) </tex> — непрерывна, дифференцируема по <tex> y </tex> при любых <tex> x </tex> и <tex> f'_y </tex> непрерывна на промежутке. Пусть <tex> \Phi(y) = \int\limits_a^b f(x, y) dx, \ y \in [c, d] </tex>. Тогда <tex> \Phi(y) </tex> дифференцируема и <tex> \Phi'(y) = \int\limits_a^b f'_y(x, y) dx </tex>.
}}
+
|proof=
 +
<tex> \frac{\Phi(y + h) - \Phi(y)}{h} = \int_a^b \frac{f(x, y + h) - f(x, y)}{h} dx = \int_a^b f'_y (x, y + \Theta h) dx; \ \Theta \in [0, 1] </tex> зависит от <tex> x, y </tex>
  
=== Лемма о равенстве интегралов по похожим путям ===
+
<tex> f'_y </tex> — непрерывна на <tex> [a, b] \times [c, d] </tex>
{{Лемма
 
|statement=
 
Пусть <tex> \gamma, \tilde{\gamma}: [a; b] \to O \subset \mathbb{R}^m </tex> — кусочно-гладкие, похожие, <tex> V </tex> — локально-потенциальное векторное поле, <tex> \gamma(a) = \tilde{\gamma} (a), \ \gamma(b) = \tilde{\gamma} (b) </tex>. Тогда <tex> \int\limits_{\gamma} \sum V_i dx_i = \int\limits_{\tilde{\gamma}} \sum V_i dx_i </tex>.
 
}}
 
  
=== Лемма о похожести путей, близких к данному ===
+
<tex> \forall \epsilon > 0 \ \exists \delta > 0 \ \forall x, y : |x - y| < \delta; \ |f'_y(x) - f'_y(y)| < \epsilon </tex> — равномерная непрерывность
{{Лемма
 
|statement=
 
Пусть <tex> \gamma: [a, b] \to O </tex>. Тогда [любые два пути, мало отличающиеся от данного — похожие] <tex> \exists \delta > 0 </tex> такое, что если пути <tex> \gamma_1, \gamma_2: [a, b] \to O </tex> — «близкие» к <tex> \gamma </tex>, то есть <tex> | \gamma(t) - \gamma_1(t) | < \delta, \ | \gamma(t) - \gamma_2(t) | < \delta </tex>, то <tex> \gamma_1, \gamma_2 </tex> похожи.
 
}}
 
  
=== Равенство интегралов по гомотопным путям ===
+
<tex> | \frac{\Phi(y + h) - \Phi(y)}{h} - \int_a^b f'_y(x, y)dx | = | \int_a^b f'_y(x, y + \Theta h) - f'_y(x, y)dx | \le </tex>
{{Теорема
 
|statement=
 
Пусть <tex> V </tex> — локально-потенциальное векторное поле в <tex> O </tex>, <tex> \gamma_0, \gamma_1: [a; b] \to O </tex> — связанно (петельно) гомотопны. Тогда <tex> \int\limits_{\gamma_0} \sum V_i dx_i = \int\limits_{\gamma_1} \sum V_i dx_i </tex>.
 
}}
 
  
=== Потенциальность локально потенциального поля. Следствие о лемме Пуанкаре ===
+
<tex> \le \int_a^b | f'_y(x, y + \Theta h) - f'_y(x, y) |dx \le^* \int_a^b \epsilon dx = \epsilon(b - a) </tex>
{{Теорема
 
|statement=
 
Пусть <tex> O </tex> — односвязная область, <tex> V </tex> — локально потенциальное поле в <tex> O </tex>. Тогда <tex> V </tex> потенциально.
 
}}
 
  
Следствие: если <tex> O </tex> — односвязная, <tex> V \in V'(O), \  \forall i, j \ \forall x \in \Omega \ \frac{\partial V_i}{\partial x_j} = \frac{\partial V_j}{\partial x_i} </tex>, то <tex> V </tex> — потенциально.
+
<tex> \le^* : \forall \epsilon > 0 \ \exists \delta > 0 \ \forall h : |h| < \delta </tex>
  
{{Лемма
+
<tex> | \frac{\Phi(y + h) - \Phi(y)}{h} - \int_a^b f'_y | < \epsilon (b - a) </tex> — определение предела.
|statement=
 
Пусть <tex> O </tex> — дополнение круга <tex> \overline{B(0; 1/2)} </tex>. Тогда <tex> O </tex> неодносвязна.
 
 
}}
 
}}
  
=== Асимптотика интеграла $\int_0^{\pi/2}\cos^nx\,dx$, $n\no+\infty$ ===
+
=== Необходимое условие потенциальности гладкого поля. Лемма Пуанкаре ===
 
{{Теорема
 
{{Теорема
 
|statement=
 
|statement=
<tex> \int\limits_0^{\pi/2} \cos^n x dx \underset{n \to + \infty}{\sim} \int\limits_0^{1/\pi^{4/3}} \cos^n x dx </tex>
+
Пусть <tex> V </tex> — гладкое потенциальное векторное поле в <tex> O </tex>. Тогда <tex> \forall x \in O \ \frac{\partial V_i}{\partial x_j} = \frac{\partial V_j}{\partial x_i} \ (*), \ i, j \in [1 : m] </tex>
 +
|proof=
 +
<tex> f </tex> — потенциал, обе части <tex> (*) = \frac{\partial^2 f}{\partial x_i \partial x_j} </tex> (— непр., т.к. <tex> V </tex> — гладкое)
 
}}
 
}}
  
=== Лемма о локализации (в методе Лапласа) ===
 
 
{{Лемма
 
{{Лемма
 
|statement=
 
|statement=
Пусть <tex> f(x) </tex> непрерывна, <tex> f(x) > 0 </tex> на <tex> (a; b), \ \int\limits_a^b f(x) dx = M, \ \varphi(x) </tex> строго монотонно убывает, непрерывна. Тогда <tex> \forall c \in (a, b) \ \int\limits_a^b f(x) e^{A \varphi(x)} \underset{A \to + \infty}{\sim} \int\limits_a^c f(x) e^{A \varphi(x)} </tex>.
+
Пусть <tex> O \subset \mathbb{R}^m </tex> — выпуклое, <tex> V </tex> — векторное поле в <tex> O </tex>, гладкое и <tex> \forall x \forall i, j \ \frac{\partial V_i}{\partial x_j} = \frac{\partial V_j}{\partial x_i} </tex>. Тогда <tex> V </tex> — потенциальное.
}}
+
|proof=
 +
фиксируем <tex> A \in O; \ \gamma [0, 1] \to O; \ t \mapsto A + t * (x - A); \ \gamma' = x - A </tex>
 +
 
 +
<tex> f(x) := \int_{\gamma} \sum V_i dx_i = </tex><tex> \int_0^1 V_1(A + t(x - A))\cdot(x_1 - A_1) + ... + V_m(A + t(x - A)) \cdot (x_m - A_m)dt </tex>
  
=== Метод Лапласа вычисления асимптотики интегралов ===
+
<tex> \frac{\partial f}{\partial x_i} = \int_0^1 V_i(A + t(x - A)) + \sum_{j = 1}^{m} \overbrace{\frac{\partial V_j}{\partial x_i}}^{\frac{\partial V_i}{\partial x_j}} (A + t(x - A))t(x_j - A_j)dt = </tex>
{{Теорема
 
|statement=
 
Пусть <tex> f > 0 </tex> на <tex> (a; b) </tex>, непрерывна, <tex> \int\limits_a^b f = M, \ f(t) \sim L(t - a)^q, \ t \to a, \ q > -1, \ L > 0, \ \varphi </tex> строго непрерывно убывает, <tex> \varphi(a) - \varphi(t) \sim c(t - a)^p, \ p > 0, \ (c > 0) </tex>. Тогда <tex> \int\limits_a^b f(x) e^{A \varphi(t)} dt \underset{A \to + \infty}{\sim} L \cdot e^{A \varphi(x)} \cdot \frac{1}{p} \cdot \frac{1}{cA^{\frac{q + 1}{r}}} \cdot \Gamma(\frac{q + 1}{r}) </tex>.
 
}}
 
  
=== Теорема Вейерштрасса о приближении функций многочленами ===
+
<tex> = \int_0^1 (t V_i (A + t(x - A)))'_t  dt = t V_i (A + t(x - A))|_{t = 0}^{t = 1} = V_i (x) </tex>
{{Теорема
 
|statement=
 
Пусть <tex> f </tex> непрерывна на <tex> [a; b] </tex>. Тогда существует многочлен <tex> P_n(x), \ n = 1, 2 ... </tex>, что <tex> \forall x \in [a; b] \ P_n(x) \to f(x) </tex>.
 
 
}}
 
}}
  
=== Формула Стирлинга для Гамма-функции ===
+
=== Лемма о гусенице ===
{{Теорема
+
{{Лемма
 
|statement=
 
|statement=
<tex> \Gamma (x + 1) \underset{x \to + \infty}{\sim} x^x e^{-x} \sqrt{2 \pi x} </tex>
+
Пусть <tex> \gamma: [a, b] \to O </tex>. Тогда существуют дробление <tex> a = t_0 < t_1 < ... < t_n = b </tex> и шары <tex> B_1, ..., B_n \subset O </tex>, что <tex> \gamma [t_{k - 1}, t_k] \subset B_k, \ k \in [1 : n] </tex>.
}}
+
|proof=
 +
<tex> \forall c \in [a, b] </tex> — выберем шар <tex> B(\gamma(c), V_c) \subset O </tex>
  
== Определения и факты ==
+
<tex> \tilde \alpha_c := \inf \{ \alpha \in [a, b]; \ \gamma([\alpha, c]) \subset B (\gamma(c), V_c) \} </tex>
=== Список ненаписанных определений ===
 
Комплексная производная
 
  
Односвязная область — проверить символьное пояснение
+
<tex> \tilde \beta_c := \sup \{ \beta \in [a, b]; \ \gamma([c, \beta]) \subset B (\gamma(c), V_c) \} </tex>
  
=== Равномерно сходящийся ряд ===
+
Пусть <tex> \tilde \alpha_c < \alpha_c < c < \beta_c < \tilde \beta_c </tex>
{{Определение
 
|definition=
 
Последовательность функций <tex> f_1(x), f_2(x), ... , f_n(x) </tex> называется равномерно сходящейся на множестве <tex> X </tex>, если существует предельная функция <tex> f(x) = \lim_{n \to \infty} f_n(x) \ (x \in X ) </tex> и для любого числа <tex> \varepsilon > 0 </tex> можно указать число <tex> N = N(\varepsilon) </tex> такое, что <tex> |f(x) - f_n(x) | < \varepsilon </tex> при <tex> n > N </tex> и <tex> x \in X </tex>. В этом случае пишут <tex> f_n(x) \rightrightarrows f(x) </tex>.
 
  
Функциональный ряд называется равномерно сходящимся на множестве <tex> X </tex>, если равномерно сходится на этом множестве последовательность его частичных сумм.
+
<tex> \forall c </tex> мы имеем <tex> (\alpha_c, \beta_c) </tex> — открытое покрытие <tex> [a, b] </tex> и <tex> \exists </tex> конечное подпокрытие
}}
 
  
=== Признак Абеля равномерной сходимости ===
+
Можно считать <tex> \forall i \ \exists s_i </tex> — которое лежит в <tex> (\alpha_{c_i}, \beta_{c_i}) </tex>, но не лежит в <tex> (\alpha_{c_j}, \beta_{c_j}); \ i \ne j </tex>
{{Теорема
 
|statement=
 
Рассмотрим ряд <tex> \sum a_n(x) b_n(x) </tex>, <tex> x \in X </tex>:
 
  
1) <tex> \sum a_n(x) </tex> равномерно сходится, <tex> x \in X </tex>
+
<tex> s_1 < s_2 ... < s_n </tex>
 
 
2) <tex> b_n(x) </tex> равномерно ограничена и монотонна по <tex> n </tex>
 
 
 
Тогда <tex> \sum a_n(x) b_n(x) </tex> равномерно сходится на <tex> X </tex>.
 
 
}}
 
}}
  
=== Радиус сходимости степенного ряда ===
+
=== Лемма о равенстве интегралов по похожим путям ===
см. [[Участник:Yulya3102/Матан3сем#Теорема о круге сходимости степенного ряда|Теорема о круге сходимости степенного ряда]] пункт 3.
+
{{Лемма
 +
|statement=
 +
Пусть <tex> \gamma, \tilde{\gamma}: [a; b] \to O \subset \mathbb{R}^m </tex> — кусочно-гладкие, похожие, <tex> V </tex> — локально-потенциальное векторное поле, <tex> \gamma(a) = \tilde{\gamma} (a), \ \gamma(b) = \tilde{\gamma} (b) </tex>. Тогда <tex> \int\limits_{\gamma} \sum V_i dx_i = \int\limits_{\tilde{\gamma}} \sum V_i dx_i </tex>.
 +
|proof=
 +
Cуществуют дробление <tex> a = t_0 < t_1 < ... < t_n = b </tex> и шары <tex> B_1, ..., B_n \subset O </tex>
  
=== Формула Адамара ===
+
<tex> \forall k </tex> в <tex> B_k </tex> существует потенциал векторного поля <tex> V </tex>
{{Определение
 
|definition=
 
Число <tex> R </tex> — радиус сходимости.
 
<tex> R = \frac{1}{\overline{lim}\sqrt[n]{a_n}} </tex>
 
}}
 
  
=== Комплексная производная ===
+
<tex> \gamma|_{[t_{k - 1}, t_k]} \subset B_k; \ \tilde \gamma|_{[t_{k - 1}, t_k]} \subset B_k </tex>
http://clubmt.ru/lec3/lec34.htm (тут первое определение)
 
  
=== Экспонента, синус и косинус комплексной переменной ===
+
Пусть <tex> f_1 </tex> — потенциал <tex> V </tex> в <tex> B_1 </tex>, в <tex> B_2 </tex> выберем потенциал <tex> f_2. \ f_1(\gamma(t_1)) = f_2(\gamma(t_1)) </tex>
{{Определение
 
|definition=
 
<tex> \mathrm{exp}(z) := \sum_{n=0}^{+ \infty} \frac{z^n}{n!} </tex>
 
  
<tex> \sin(z) := \mathrm{Im}(\mathrm{exp}(z)) </tex>
+
в <tex> B_3 </tex> выберем <tex> f_3. \ f_2(\gamma(t_2)) = f_3(\gamma(t_2))) </tex> и т.д.
  
<tex> \cos(z) := \mathrm{Re}(\mathrm{exp}(z)) </tex>
+
<tex> \int_{\gamma} \sum V_i dx_i = \int_a^b \langle V(\gamma(t)), \gamma(t)dt = \sum_{i = 1}^{n} \int_{t_{i - 1}}^{t_i} = \sum_{i = 1}^{n} f_i (x(t_i)) - f_{i - 1}(\gamma(t_{i - 1})) </tex>
}}
 
  
=== Отображение, бесконечно малое в точке ===
+
<tex> \int_{\tilde \gamma} \sum V_i dx_i = f_n(\tilde \gamma(t_n)) - f_1(\tilde \gamma(t_0)) </tex>
{{Определение
 
|definition=
 
Пусть <tex> \varphi: \ E \in \mathbb{R}^m \to \mathbb{R}^l </tex>, <tex> a \in E </tex>. <tex> \varphi </tex> — бесконечно малое при <tex> x \to a </tex>, если <tex> \lim \varphi(x) = \mathbb{O}_l </tex>. (<tex> \mathbb{O}_l </tex> — <tex> l </tex>-мерный ноль)
 
 
}}
 
}}
 +
* Замечание
  
=== o(h) при h->0 ===
+
<tex> \gamma(a) = \tilde \gamma(a), \ \gamma(b) = \tilde \gamma(b) \\ \gamma(a) = \gamma(b), \ \tilde \gamma(a) = \tilde \gamma(b) </tex>
{{Определение
 
|definition=
 
Пусть <tex> \varphi: \ \mathbb{R}^m \to \mathbb{R}^l </tex>. <tex> \varphi(h) = o(h) </tex> при <tex> h \to 0 </tex>, если <tex> \frac{\varphi(h)}{||h||} </tex> — бесконечно малая при <tex> h \to 0 </tex>.
 
}}
 
  
=== Дифференцируемое отображение ===
+
=== Лемма о похожести путей, близких к данному ===
{{Определение
+
{{Лемма
|definition=
+
|statement=
Пусть <tex>f:D\subset\mathbb{R}^n\to\mathbb{R}^m,x\in\operatorname{Int}D</tex> (<tex>\operatorname{Int} D</tex> — множество внутренних точек (внутренность) множества D). Если существует такой линейный оператор <tex>A\in\mathcal{L}(\mathbb{R}^n\to\mathbb{R}^m)</tex> (<tex>\mathcal{L}(X\to Y)</tex> — множество линейных ограниченных операторов из <tex>X</tex> в <tex>Y</tex>), что
+
Пусть <tex> \gamma: [a, b] \to O </tex>. Тогда [любые два пути, мало отличающиеся от данного — похожие] <tex> \exists \delta > 0 </tex> такое, что если пути <tex> \gamma_1, \gamma_2: [a, b] \to O </tex> — «близкие» к <tex> \gamma; * </tex>, то есть <tex> \forall t \in [a, b] \ \ | \gamma(t) - \gamma_1(t) | < \delta, \ | \gamma(t) - \gamma_2(t) | < \delta </tex>, то <tex> \gamma_1, \gamma_2 </tex> похожи.
 +
|proof=
 +
Cуществуют дробление <tex> a = t_0 < t_1 < ... < t_n = b </tex> и шары <tex> B_1, ..., B_n \subset O </tex> для <tex> \gamma </tex>
  
<tex>f(x+h)=f(x)+Ah+o(h), h\to\mathbb{O}_n</tex>,
+
<tex> \gamma[t_{k - 1}, t_{k}] </tex> — компакт в <tex> B_k </tex>
  
то отображение <tex>f</tex> называется '''дифференцируемым''' в точке <tex>x</tex>. При этом оператор <tex>A</tex> называется '''производным оператором''', '''производным отображением''' или, короче, '''производной''' отображения <tex>f</tex> в точке <tex>x</tex> и обозначается <tex>f'(x)</tex>.
+
<tex> \exists \delta_k > 0 : \delta_k = dist(\gamma[t_{k - 1}, t_k], \partial B_k); g(t) = dist(\gamma(t), \partial B_k) </tex>
}}
 
  
=== Производный оператор ===
+
<tex> \delta := \min_{1 \le k \le n} \delta_k </tex>
{{Определение
 
|definition=
 
Оператор <tex> A </tex> из определения производной называется производным оператором отображения <tex> f </tex> в точке <tex> x </tex>.
 
}}
 
  
=== Дифференциал отображения ===
+
<tex> A_k = \{ x \in \mathbb{R}^n : \exists t \in [t_{k - 1}, t_{k}] \ \ \rho(\gamma(t), x) < \delta \} \subset B_k </tex>
{{Определение
 
|definition=
 
Величина <tex>f'(x)h</tex> называется '''дифференциалом''' отображения <tex>f</tex> в точке <tex>x</tex>, соответствующим приращению <tex>h</tex>, и обозначается <tex>df(x,h)</tex> или <tex>d_x f(h)</tex>.
 
}}
 
  
=== Матрица Якоби ===
+
<tex> \forall \gamma_1, \gamma_2 </tex> — удовл. <tex> * : \gamma_1 [a, b] \subset \cup_{k = 1}^{n} A_k, \gamma_2 \subset \cup_{k = 1}^{n} A_k </tex> и <tex> (\{B_k\}, \{t_i\}) </tex> — гусеница реал. похож. путей
{{Определение
 
|definition=Пусть отображение <tex>f:D\subset\mathbb{R}^n\to\mathbb{R}^m</tex> дифференцируемо в точке <tex>x\in\operatorname{Int} D</tex>. Матрица оператора <tex>f'(x)</tex> называется '''матрицей Якоби''' отображения <tex>f</tex> в точке <tex>x</tex>.
 
 
}}
 
}}
  
=== Частные производные ===
+
=== Равенство интегралов по гомотопным путям ===
{{Определение
+
{{Теорема
|definition=
+
|statement=
Пусть <tex> f: D \subset \mathbb{R}^n \to \mathbb{R}, \ x \in \operatorname{Int} D, \ k \in [1 : n] </tex>. Производная <tex> \frac{\partial f}{\partial e^k} (x) </tex> (где <tex> e^k </tex> — это орт (т.е. единичный вектор — вектор, норма которого равна 1)) называется частной производной функции <tex> f </tex> по <tex> k </tex>-ой переменной в точке <tex> x </tex> и обозначается ещё <tex> D_k f(x), \ D_{x_k} f(x), \ f'_{x_k} (x), \ \frac{\partial f}{\partial x_k} (x) </tex>.
+
Пусть <tex> V </tex> — локально-потенциальное векторное поле в <tex> O </tex>, <tex> \gamma_0, \gamma_1: [a; b] \to O </tex> — связанно гомотопны. Тогда <tex> \int\limits_{\gamma_0} \sum V_i dx_i = \int\limits_{\gamma_1} \sum V_i dx_i </tex>. Тоже верно для петельной гомотопии.
}}
+
|proof=
 +
<tex> \Gamma </tex> — гомотопия. <tex> \gamma_u(t) = \Gamma(t, u), \ u \in [0, 1]  </tex>
  
=== Производная по вектору, по направлению ===
+
<tex> \Phi(u) = \int_{\gamma_u} \sum V_i dx_i </tex>. Проверим, что <tex> \Phi </tex> — локальная постоянная
{{Определение
 
|definition=
 
Пусть <tex> f: D \subset \mathbb{R}^n \to \mathbb{R} </tex>, <tex> x \in Int(D) </tex>, <tex> h \in \mathbb{R}^n </tex>. Предел <tex> \lim_{t \to 0} \frac{f(x + th) - f(x)}{t} </tex> называется производной функции <tex> f </tex> по вектору <tex> h </tex> в точке <tex> x </tex> и обозначается <tex> D_h f(x) </tex> или <tex> \frac{\partial f}{\partial h}(x) </tex>. Если <tex> |h| = 1 </tex>, то вектор <tex> h </tex> называется направлением, а производная по нему производной по направлению <tex> h </tex>.
 
}}
 
  
=== Градиент ===
+
<tex> (\forall u_0 \ \exists W(u_0) </tex> при <tex> u \in W(u_0) : \Phi </tex> — постоянна)
{{Определение
 
|definition=
 
Пусть <tex>f:D\subset\mathbb{R}^n\to\mathbb{R},x\in\operatorname{Int}D</tex>. Если существует такой вектор <tex>a\in\mathbb{R}^n</tex>, что <tex>f(x+h)=f(x)+\langle a,h\rangle+o(h),h\to\mathbb{O}_n</tex>, то функция <tex>f</tex> называется '''дифференцируемой''' в точке <tex>x</tex>.
 
  
Вектор-строка <tex>a</tex> называется '''градиентом''' функции <tex>f</tex> в точке <tex>x</tex> и обозначается <tex>\operatorname{grad} f(x)</tex> или <tex>\nabla f(x)</tex>. Символ <tex>\nabla</tex> называется '''символом''' или '''оператором Гамильтона'''.
+
<tex> \Gamma : \overbrace{[a, b] \times [0, 1]}^{copmact} \to O </tex> — равномерно непрерывна.
}}
 
  
=== Частная производная второго порядка, k-го порядка ===
+
<tex> \forall \delta > 0 \ \exists \zeta > 0 \ \forall (t_1, u_1), (t_2, u_2) \in [a, b] \times [0, 1] \ \ </tex><tex>\ \ \begin{matrix} |t_1 - t_2| < \zeta \\ |u_1 - u_2| < \zeta \end{matrix} </tex> верно <tex> |\Gamma(t_1, u_1) - \Gamma(t_2, u_2)| < \frac{\delta}{2} </tex>
{{Определение
 
|definition=
 
Предположим, что <tex> r - a \in \mathbb{R} </tex> и частные производные порядка <tex> r - 1 </tex> уже определены. Пусть <tex> i_1, ... , i_r \in [1 : n], \ f : D \subset \mathbb{R}^n \to \mathbb{R}, \ x \in D </tex>. Частная производная функции <tex> f </tex> порядка <tex> r </tex> по переменным с номерами <tex> i_1, ..., i_r </tex> в точке <tex> x </tex> определяется равенством <tex> D_{i_1, ..., i_r}^r f(x) = D_{i_r} (D_{i_1, ..., i_{r - 1}}^{r-1} f)(x) </tex>, если правая часть существует.
 
 
}}
 
}}
  
=== Классы функций $C^k(E)$ ===
+
=== Потенциальность локально потенциального поля. Следствие о лемме Пуанкаре ===
{{Определение
+
{{Теорема
|definition=
+
|statement=
Множество функций, <tex> r </tex> раз непрерывно дифференцируемых на открытом подмножестве <tex> D </tex> пространства <tex> \mathbb{R}^n </tex>, обозначается <tex> C^{(r)} (D) </tex> или <tex> C^r (D) </tex>. По определению <tex> C^0 (D) = C(D) </tex> — класс непрерывных на <tex> D </tex> функций. Через <tex> C^{(\infty)} (D) </tex> обозначается класс бесконечно дифференцируемых на <tex> D </tex> функций.
+
Пусть <tex> O </tex> — односвязная область, <tex> V </tex> — локально потенциальное поле в <tex> O </tex>. Тогда <tex> V </tex> потенциально.
}}
+
|proof=
 +
<tex> V </tex> — потенциально <tex> \Leftrightarrow \forall \gamma : [a, b] \to \mathbb{R}, \ \gamma(a) = \gamma(b) : \ \int_{\gamma} \sum V_i dx_i = 0 </tex>
  
=== Мультииндекс и обозначения с ним ===
+
По предыдущей теореме: <tex> \int_{\gamma} \sum V_i dx_i = \int_{\gamma_1} \sum V_i dx_i </tex> — гомотопия пост. пути <tex> \gamma_1 </tex>
{{Определение
 
|definition=
 
Вектор <tex> k \in \mathbb{Z}_+^n </tex> называют мультииндексом. Величину <tex> (k) = k_1 + ... + k_n </tex> называют высотой мультииндекса <tex> k </tex>.
 
 
}}
 
}}
Если <tex> k = (k_1, .., k_n) </tex> — мультииндекс, <tex> (k) \leqslant r </tex>, то частную производную порядка <tex> k </tex> (порядком частной производной называют как сам мультииндекс, так и его высоту) функций класса <tex> C^{(r)} </tex> обозначают <tex> D^k f, \ f^{(k_1, ..., k_n)}, \ f^{(k)} </tex>. Также полагают <tex> k! = k_1 ! \cdot ... \cdot k_n ! </tex>, <tex> h^k = h_1^{k_1} \cdot ... \cdot h_n^{k_n} </tex>, где <tex> h \in \mathbb{R}^n </tex>.
 
  
=== Формула Тейлора (различные виды записи) ===
+
Следствие: если <tex> O </tex> — односвязная, <tex> V \in C^1(O), \  \forall i, j \ \forall x \in \Omega \ \frac{\partial V_i}{\partial x_j} = \frac{\partial V_j}{\partial x_i} </tex>, то <tex> V </tex> — потенциально.
Из теорем:
 
  
<tex dpi="150"> f(x) = \sum_{(k) \leqslant r} \frac{f^{(k)} (a) }{k!} (x - a)^k + \sum_{(k) = r + 1} \frac{f^{(k)} (a + \theta(x - a))}{k!} (x - a)^k </tex>
+
=== Асимптотика интеграла $\int_0^{\pi/2}\cos^nx\,dx$, $n\no+\infty$ ===
 +
{{Теорема
 +
|statement=
 +
<tex> \int\limits_0^{\pi/2} \cos^n x dx \underset{n \to + \infty}{\sim} \sqrt{\frac{2}{n}}  \int\limits_0^{+\inf} e^{-t^2} dt </tex>
 +
|proof=
  
<tex dpi="150"> f(x + h) = \sum_{(k) \leqslant r} \frac{f^{(k)} (x)}{k!} h^k + \sum_{(k) = r + 1} \frac{f^{(k)} (x + \theta h)}{k!} h^k </tex>
+
Доказательство в три шага, полностью выписывать много, поэтому здесь только идеи:
  
<tex dpi="150"> f(x + h) = \sum_{(k) \leqslant r} \frac{f^{(k)} (x)}{k!} h^k + o(|h|^r), \ h \to \mathbb{O}_n </tex>
+
1) <tex>\int\limits_0^{\pi/2} \cos^n x dx \underset{n \to + \infty}{\sim} \int\limits_0^{n^{-\frac{1}{3}}} \cos^{n}x dx</tex>
  
С остатком в интегральной форме:
+
Доказывается заменой <tex>\cos^n{x} = e^{n\ln{\cos{x}}}</tex> и каким-то подбором нового предела интегрирования, зависящего от n (конспект, стр.143)
  
<tex dpi="150"> f(x + h) = \sum_{(k) \leqslant r} \frac{f^{(k)} (x)}{k!} h^k + \int\limits_0^1 \sum_{(k) = r + 1} \frac{r + 1}{k!} f^{(k)} (x + th) h^k (1 - t)^r dt </tex>
+
2) Доказываем, что x — точка максимума для <tex>\ln{\cos{x}}</tex>, вместе с этим заменяем по формуле Тейлора <tex>n\ln{\cos{x}}</tex> на <tex>-\frac{nx^2}{2}+o(x^2)</tex> и показываем, что это <tex>o(x^2)</tex> не мешает подставить замену в интеграл.
  
Формула в дифференциалах:
+
3) Делаем замену <tex>t=\sqrt{\frac{n}{2}}x, dx = \sqrt{\frac{2}{n}}dt</tex>, получаем интеграл из условия.
  
<tex dpi="150"> f(x + h) = \sum_{l=0}^{r} \frac{1}{l!} d^l f(x, h) + \frac{1}{(r+1)!} d^{r + 1} f(x + \theta h, h) </tex>
+
}}
  
Формула в координатах:
+
=== Лемма о локализации (в методе Лапласа) ===
 +
{{Лемма
 +
|statement=
 +
Пусть <tex> f(x) </tex> непрерывна, <tex> f(x) > 0 </tex> на <tex> (a; b), \ \int\limits_a^b f(x) dx = M, \ \varphi(x) </tex> строго монотонно убывает, непрерывна. Тогда <tex> \forall c \in (a, b) \ \int\limits_a^b f(x) e^{A \varphi(x)} \underset{A \to + \infty}{\sim} \int\limits_a^c f(x) e^{A \varphi(x)} </tex>.
 +
|proof=
 +
<tex> \int_{c}^{b} f(x) e^{A \varphi(x)} \le \max_{x \in [c, b]} e^{A \varphi(x)} \int_c^b f(x)dx \le e^{A \varphi(c)}M </tex>
  
<tex dpi="150"> f(x, y) = \sum_{l=0}^r \frac{1}{l!} \sum_{\nu = 0}^{l} C_l^{\nu} \frac{\partial^l f(x^0, y^0)}{\partial x^{\nu} \partial y^{l - \nu}} (x - x^0)^{\nu} (y - y^0)^{l - \nu} + o((\sqrt{(x - x^0)^2 + (y - y^0)^2} )^r), \ (x , y) \to (x^0, y^0) </tex>
+
<tex> \int_a^c f(x) e^{A \varphi(x)} dx \ge \int_a^{\frac{c}{2}} f(x)e^{A \varphi(x)} \ge \min e^{A \varphi(x)} \int_a^{\frac{c}{2}} f(x)dx = e^{A \varphi(\frac{c}{2})} \int_a^{\frac{c}{2}} f(x)dx </tex> // последняя экспонента с большим показателем
 +
}}
  
=== $n$-й дифференциал ===
+
=== Метод Лапласа вычисления асимптотики интегралов ===
{{Определение
 
|definition=
 
Пусть <tex> f: \mathbb{R}^m \to \mathbb{R}, \ f \in C^r(\mathbb{R}^m) </tex>. Тогда:
 
  
<tex> df(a) = f'_{x_1}(a) dx_1 + ... + f'_{x_m}(a)dx_m </tex>
+
{{Теорема
 +
|statement=
 +
Пусть <tex> f > 0 </tex> на <tex> (a; b) </tex>, непрерывна, <tex> \int\limits_a^b f = M, \ f(t) \sim L(t - a)^q, \ t \to a, \ q > -1, \ L > 0, \ \varphi </tex> непрерывна, строго убывает, <tex> \varphi(a) - \varphi(t) \sim c(t - a)^p, \ p > 0 </tex>. Тогда <tex> \int\limits_a^b f(t) e^{A \varphi(t)} dt \underset{A \to + \infty}{\sim} e^{A \varphi(a)} \cdot \frac{1}{p} \cdot \frac{1}{(cA)^{\frac{q + 1}{p}}} \cdot \Gamma(\frac{q + 1}{p}) </tex>.
  
<tex> d^2f(a) = d(df(a)) = f''_{x_1, x_1} dx_1 dx_1 + f''_{x_1, x_2} dx_1 dx_2 + f''_{x_2, x_1} dx_2 dx_1 + ... </tex>
+
|proof=
 
 
<tex> d^3f(a) = d(d^2f(a)) = ... </tex>
 
  
<tex> d^r f(a) = \sum c_{i_1, ..., i_r} \frac{\partial^r f(a)}{\partial x_{i_1} \cdot ... \cdot x_{i_r}} dx_{i_1} \cdot ... \cdot dx_{i_r} </tex>, где <tex> c_{i_1, ..., i_r} </tex> — количество способов получить дифференциал, выбирая разный порядок.
+
* В доказательстве используется прием: при <tex>q > 1, p > 0, A > 0, s > 0</tex> в интеграле <tex>\int\limits_0^s t^q e^{-At^p} dt</tex>  
}}
 
  
=== Норма линейного оператора ===
+
* вводим замену <tex>u = At^p, t = (\frac{u}{A})^{1/p}, dt = \frac{u^{1/p-1}}{pA^{1/p}}</tex>.
Напомним, что норма в векторном пространстве <tex> X </tex> над <tex> \mathbb{R} </tex> — функция <tex> p: X \to \mathbb{R}_+ </tex>, удовлетворяющая аксиомам нормы: положительная определённость (<tex> p(x) = 0 </tex> тогда и только тогда, когда <tex> x = 0 </tex>), положительная однородность (<tex> p(\lambda x) = |\lambda| p(x) </tex>, где <tex> \lambda </tex> — скаляр), неравенство треугольника (<tex> p(x + y) \leqslant p(x) + p(y)</tex>). Аналогично для матриц (там <tex> \lambda \in \mathbb{R} </tex>).
 
{{Определение
 
|definition=
 
Пусть <tex> X, Y </tex> — нормированные пространства (оба вещественные или оба комплексные), <tex> A: X \to Y </tex> — линейный оператор. Нормой оператора <tex> A </tex> называется величина <tex> || A || = \underset{||x||_X \leqslant 1}{\sup} ||Ax||_Y </tex>.
 
}}
 
  
=== Локальный максимум, минимум, экстремум ===
+
* Тогда он превращается в <tex>\frac{1}{pA^{\frac{q+1}{p}}} \int\limits_0^{As^p} u^{\frac{q+1}{p} - 1}e^{-u}du</tex>, который при <tex>A\to{+\infty}</tex> стремится к <tex>\frac{1}{pA^{\frac{q+1}{p}}}\Gamma({\frac{q+1}{p}})</tex>
{{Определение
 
|definition=
 
Пусть <tex> f: D \subset \mathbb{R}^n \to \mathbb{R}, \ x_0 \in D </tex>. Если существует такая окрестность <tex> V_{x_0} </tex> точки <tex> x_0 </tex>, что для любого <tex> x \in V_{x_0} \cap D </tex> выполняется неравенство:
 
  
<tex> f(x) \leqslant f(x_0) </tex>, то <tex> x_0 </tex> называется точкой максимума функции <tex> f </tex>;
+
'''Утверждения:'''
  
<tex> f(x) < f(x_0) </tex>, то <tex> x_0 </tex> называется точкой строгого максимума функции <tex> f </tex>.
+
1) <tex>\forall{c\in(a, b)}\ \forall{\varepsilon > 0}\ \exists{A_0}\ \forall{A > A_0}\ \int\limits_a^c{fe^{A\varphi}} \le \int\limits_a^b{fe^{A\varphi}} \le (1 + \varepsilon)\int\limits_a^c{fe^{A\varphi}}</tex> (следствие из теоремы о локализации)
  
Аналогично определяются точки минимума и строгого минимума. Если <tex> x_0 </tex> является точкой (строгого) максимума или минимума функции <tex> f </tex>, то <tex> x_0 </tex> называется точкой (строгого) экстремума <tex> f </tex>.
+
2) <tex>\forall{\varepsilon > 0}\ \exists{A_0}\ \forall{A > A_0}</tex>
}}
 
  
=== Положительно-, отрицательно-, незнако- определенная квадратичная форма ===
+
<tex>(1-\varepsilon)\frac{1}{pA^{\frac{q+1}{p}}}\Gamma(\frac{q+1}{p}) \le \int\limits_0^s t^q e^{-At^p} dt \le \frac{1}{pA^{\frac{q+1}{p}}}\Gamma(\frac{q+1}{p})</tex> (следствие из приема выше. Да, читается ужасно)
{{Определение
 
|definition=
 
Пусть <tex> K </tex> — квадратичная форма от <tex> n </tex> переменных. <br>
 
  
1) Если <tex> K(h) > 0 </tex> для всех <tex> h \in \mathbb{R}^n \backslash \{ \mathbb{O}_n \} </tex>, то форма <tex> K </tex> называется положительно определённой. <br>
+
'''Доказательство'''
  
2) Если <tex> K(h) < 0 </tex> для всех <tex> h \in \mathbb{R}^n \backslash \{ \mathbb{O}_n \} </tex>, то форма <tex> K </tex> называется отрицательно определённой. <br>
+
Выбираем окрестность точки <tex>a: [a; a+s]</tex> и <tex>\varepsilon</tex> такое, что
  
3) Если форма <tex> K </tex> принимает значения разных знаков, то <tex> K </tex> называется неопределённой. <br>
+
<tex>1-\varepsilon < \frac{f(t)}{L(t-a)^q} < 1+\varepsilon</tex>
  
4) Если <tex> K(h) \geqslant 0 \ (K(h) \leqslant 0) </tex> для всех <tex> h \in \mathbb{R}^n </tex> и существует такое <tex> h \neq \mathbb{O}_n </tex>, что <tex> K(h) = 0 </tex>, то форма <tex> K </tex> называется положительно (отрицательно) полуопределённой.
+
<tex>1-\varepsilon < \frac{\varphi(a) - \varphi(t)}{c(t-a)^p} < 1+\varepsilon</tex>
}}
 
  
=== Диффеоморфизм ===
+
Для <tex>A > A_0</tex>, удовлетворяющих двум утверждениям выше, выполняется:
{{Определение
 
|definition=
 
Отображение <tex> F: O \subset \mathbb{R}^m \to \mathbb{R}^m </tex>, где <tex> O </tex> открыто, называется диффеоморфизмом, если оно дифференцируемо в нуле, обратимо, и обратное к нему тоже дифференцируемо.
 
}}
 
  
=== Формулировка теоремы о неявном отображении в терминах систем уравнений ===
+
<tex>\int\limits_a^b f(t)e^{A\varphi(t)} dt \le (1+\varepsilon)\int\limits_a^{a+s}L(t-a)^q \cdot e^{A\varphi(a)} \cdot e^{-A(\varphi(a)-\varphi(t)} dt \le</tex>
{{Теорема
 
|statement=
 
Дана система из <tex> n </tex> уравнений для функций от <tex> m + n </tex> переменных. Функции дифференцируемы <tex> n </tex> раз.
 
  
<tex> \begin{cases}
+
<tex>\le (1+\varepsilon)Le^{A\varphi(a)}\int\limits_0^s{\tau^q}e^{-Ae^{c(1-\varepsilon)\tau^p}}d\tau</tex>
f_1(x_1, ..., x_m, y_1, ..., y_n) = 0 \\  
 
... \\
 
f_n(x_1, ..., x_m, y_1, ..., y_n) = 0
 
\end{cases} </tex>
 
  
<tex dpi="150"> \frac{\partial F}{\partial y} :=
+
По утверждению 2 это меньше или равно <tex>\frac{1+\varepsilon}{(1-\varepsilon)^{\frac{q+1}{p}}}\cdot L\cdot [e^{A \varphi(a)} \frac{1}{p(cA)^{\frac{q + 1}{p}}} \Gamma(\frac{q + 1}{p})]</tex>. В квадратных скобках то, что нам нужно.
\begin{pmatrix}
 
\frac{\partial f_1}{\partial y_1} & ... & \frac{\partial f_1}{\partial y_n} \\
 
\ & ... & \ \\
 
\frac{\partial f_n}{\partial y_1} & ... & \frac{\partial f_n}{\partial y_n}
 
\end{pmatrix} </tex>
 
  
Пусть <tex> (a, b) = (a_1, ..., a_m, b_1, ..., b_n) </tex> удовлетворяет системе, <tex> \det (\frac{\partial F}{\partial y} (a, b)) \neq 0 </tex>. Тогда существует <tex> u(a) \subset \mathbb{R}^m </tex> и существует единственное отображение <tex> \Phi: \mathbb{R}^m \to \mathbb{R}^n, \ \Phi(a) = b, \ \Phi \in C^n </tex> такие, что <tex> \forall x \in u(a) \ (x, \Phi(x)) </tex> удовлетворяет системе.
+
Используя другие части неравенства, находим, что <tex>\int\limits_a^b f(t)e^{A\varphi(t)} dt \ge \frac{1-\varepsilon}{(1+\varepsilon)^{\frac{q+1}{p}}}\cdot L\cdot [e^{A \varphi(a)} \frac{1}{p(cA)^{\frac{q + 1}{p}}} \Gamma(\frac{q + 1}{p})]</tex>.
}}
 
  
=== Гладкое простое $k$-мерное многообразие в {\mathbb R}^m ===
+
Вроде доказали.
{{Определение
 
|definition=
 
<tex> M \subset \mathbb{R}^m </tex> — простое <tex> k </tex>-мерное многообразие, если <tex> \exists \Omega \subset \mathbb{R}^k \ \exists \Phi: \Omega \to M </tex>. <tex> \Phi </tex> называется параметризацией. Если <tex> \Phi: \Omega \to \mathbb{R}^m, \ \Phi \in C^r(\Omega, \mathbb{R}^m), \ \forall a \in \Omega \ \operatorname{rg} \Phi'(a) = k </tex> (<tex> \operatorname{rg} </tex> — ранг), то <tex> M </tex> — простое гладкое (класса <tex> C^r </tex>) <tex> k </tex>-мерное многообразие.
 
}}
 
  
=== Относительный локальный максимум, минимум, экстремум ===
 
{{Определение
 
|definition=
 
Пусть <tex> f: \mathbb{R}^{m+n} \to \mathbb{R}, \ \Phi: \mathbb{R}^{m+n} \to \mathbb{R}^n, \ H_{\Phi} = \{x \in \mathbb{R}^{m+n}: \ \Phi(x) = \mathbb{O}_n\} </tex> (<tex> \Phi(x) = \mathbb{O}_n </tex> — уравнение связи). Тогда <tex> p \in H_{\Phi} </tex> — локальный относительный (условный) экстремум <tex> f </tex> при условии <tex> \Phi = \mathbb{O}_n </tex>. Это значит, что <tex> p </tex> — локальный экстремум <tex> f | _{H_\Phi} </tex>. Если <tex> \exists U(p) \subset \mathbb{R}^{m+n} \ \forall x \in U(p) \cap H_{\Phi} \ f(x) > f(p) </tex>, то <tex> p </tex> — локальный минимум (строгий), если <tex> f(x) \geqslant f(p) </tex>, то <tex> p </tex> — локальный минимум (строгий). Аналогично задаются локальные максимумы.
 
 
}}
 
}}
  
Или в стиле определения обычного экстремума:
+
=== Теорема Вейерштрасса о приближении функций многочленами ===
{{Определение
+
{{Теорема
|definition=
 
Пусть <tex> f: D \subset \mathbb{R}^{n+m} \to \mathbb{R}, \ \Phi: D \to \mathbb{R}^m, \ x_0 \in D </tex>. Если <tex> \Phi (x_0) = \mathbb{O}_m </tex> и существует такая окрестность <tex> V_{x_0} </tex> точки <tex> x_0 </tex>, что для любого <tex> x \in V_{x_0} \cap D </tex>, удовлетворяющего условию <tex> \Phi(x) = \mathbb{O}_m </tex>, выполняется равенство <tex> f(x) \leqslant f(x_0) </tex>, то <tex> x_0 </tex> называется точкой условного или относительного максимума функции <tex> f </tex> при условии связи <tex> \Phi (x) = \mathbb{O}_m </tex>.
 
}}
 
 
 
=== Формулировка достаточного условия относительного экстремума ===
 
{{Утверждение
 
 
|statement=
 
|statement=
Пусть для точки <tex> a </tex> выполняются условия теоремы о необходимом условии относительного экстремума. Пусть <tex> h = (h_1, ..., h_{m+n}) </tex> — решение уравнения <tex> \Phi'(a) h = 0 </tex>. Рассмотрим квадратичную форму <tex> Q(h_1, ..., h_m) = d^2 G_a </tex>, где <tex> G </tex> — функция Лагранжа (<tex> G(x) = f(x) + \sum_{i=1}^m \lambda_i \varphi_i(x) </tex>, <tex> \varphi_i </tex> — условия), где <tex> \lambda_1, ... \lambda_n </tex> взяты из условия «подозрительности» точек. Тогда если <tex> Q </tex>:
+
Пусть <tex> f </tex> непрерывна на <tex> [a; b] </tex>. Тогда существует многочлен (последовательность многочленов?) <tex> P_n(x), \ n = 1, 2 ... </tex>, что <tex> \forall x \in [a; b] \ P_n(x) \to f(x) </tex>.
 +
|proof=
 +
<tex> [a, b] \subset [a - 1, b + 1] = [a_1, b_1] </tex> // Можно считать <tex> \begin{matrix} [a, b] = [\frac{1}{3}, \frac{2}{3}] \\ [a_1, b_1] = [0, 1] \end{matrix} </tex>
  
1) положительно определена, то <tex> a </tex> — точка локального относительного минимума;
+
<tex> \tilde f(x) = \begin{cases} f(x), x \in [a, b] \\ f(a), x \in [a_1, a] \\ f(b) x \in [b, b_1] \end{cases} </tex>
  
2) отрицательно определена, то <tex> a </tex> — точка локального относительного максимума;
+
Заметим, что: <tex> \int_{a_1}^{b_1} \tilde f(t)(1 - (x - t)^2)^n dt \sim_{n \to +\infty} \sqrt{\frac{\pi}{n}} f(x); \ x \in [a, b] </tex>
  
3) незнакоопределена, то <tex> a </tex> — не точка локального относительного экстремума;
+
<tex> \varphi (t) = ln(1 - (x - t)^2); \ max \varphi </tex> — достигается при <tex> t = x </tex>
  
4) знакоопределена, но вырождена, то неизвестно, является ли <tex> a </tex> точкой локального относительного экстремума.
+
<tex> \varphi(t) \sim -(x - t)^2, t \to x </tex>
}}
 
  
=== Кусочно-гладкий путь ===
+
<tex> \varphi''(x) = -2, \ \varphi(x) = 0 </tex>
{{Определение
 
|definition=
 
Путь — <tex> \varphi: [a; b] \to \mathbb{R}^M </tex>, непрерывное
 
  
<tex> L = \varphi([a; b]) </tex> — носитель пути («кривая»)
+
<tex> Q_n(x) \sim \sqrt{\frac{\pi}{n}} f(x), \ n \to +\infty </tex>
  
<tex> \varphi </tex> — кусочно-гладкий путь, если существует дробление <tex> t_0 = a < t_1 < ... < t_n = b </tex> такое, что <tex> \varphi|_{[t_{k - 1}, t_k]} </tex> — гладкий путь.
+
<tex> \sqrt{\frac{n}{\pi}} Q_n (x) \to f(x)_{x \in [a_1, b]}, \ n \to +\infty </tex>
 
}}
 
}}
 +
* Замечание
  
=== Интеграл векторного поля по кусочно-гладкому пути ===
+
<tex> \forall f </tex> — непр. на <tex> [a, b] \ \ \exists f_n(x) </tex> — многочлен : <tex> P_n(x) \rightrightarrows f </tex> на <tex> [a, b] </tex>
{{Определение
 
|definition=
 
<tex> V: E \subset \mathbb{R}^m \to \mathbb{R}^m </tex>, где <tex> E </tex> открыто — векторное поле. Рассматриваем только непрерывные векторные поля
 
  
<tex> V </tex> — гладкое векторное поле, если <tex> V \in C^r (E, \mathbb{R}^m) </tex>
+
=== Формула Стирлинга для Гамма-функции ===
 +
{{Теорема
 +
|statement=
 +
<tex> \Gamma (x + 1) \underset{x \to + \infty}{\sim} x^x e^{-x} \sqrt{2 \pi x} </tex>
 +
|proof=
 +
<tex> \Gamma(x + 1) = \int_0^{+\infty} t^x e^{-t} dt =_{t = ux; \ dt = xdu} \ </tex><tex>\ x^{x + 1} \int_0^{+\infty} u^x e^{-ux} du = x^{x + 1} \int_0^{+\infty} e^{-x(u - \ln u)} du \sim </tex>
  
Пусть <tex> V </tex> — непрерывное векторное поле в <tex> E </tex>, <tex> \gamma </tex> — кусочно-гладкий путь в <tex> E </tex>: <tex> \gamma: [a; b] \to E </tex>. Тогда интеграл векторного поля по пути <tex> \gamma </tex> равен <tex> I(V, \gamma) = \int\limits_a^b \left \langle V(\gamma(t)), \gamma'(t) \right \rangle dt = \int\limits_a^b (V_1 dx_1 + ... + V_m dx_m) </tex>, где <tex> x_i = \gamma_i(t) </tex>.
+
// <tex> \varphi(u) = -(u - \ln u) </tex>
}}
 
  
=== Потенциальное векторное поле ===
+
// <tex> \varphi' = -(1 - \frac{1}{u}); u = 1; \varphi'(u) = 0 - (\cdot) max </tex>
{{Определение
 
|definition=
 
Пусть <tex> O \subset \mathbb{R}^m </tex> (<tex> O </tex> — область). <tex> V: O \to \mathbb{R}^m </tex> потенциально в <tex> O </tex>, если существует потенциал <tex> F: O \to \mathbb{R}^m </tex>, где <tex> F </tex> дифференцируемо в <tex> O </tex>, такой, что <tex> \frac{\partial F}{\partial x_k} = V_k, \ k \in [1 : m] </tex>.
 
}}
 
  
=== Потенциал векторного поля ===
+
// <tex> \varphi'' = -\frac{1}{u^2}; \ \varphi''(1) = -1 </tex>
{{Определение
 
|definition=
 
<tex> F </tex> из предыдущего определения — потенциал.
 
}}
 
  
=== Похожие пути ===
+
<tex> \sim x^{x + 1} e^{-x} \sqrt{\frac{2\pi}{x}} \cdot \frac{1}{\sqrt{1}} \cdot 1 </tex>
{{Определение
 
|definition=
 
Пути <tex> \gamma, \tilde{\gamma} : [a; b] \to \mathbb{R}^m </tex> — похожие, если у них существует общая «гусеница» («гусеница» — это сооружение из леммы о гусенице. Линия, а на ней пересекающиеся шарики).
 
 
}}
 
}}
 +
<tex> \int_{\gamma} \sum V_i dx_i = \int_{\gamma_1} \sum V_i dx_i</tex>
  
=== Локально-потенциальное векторное поле ===
+
== Определения и факты ==
{{Определение
+
[[Участник:Yulya3102/Матан3сем/Определения|Перемещено, а то из-за большого размера страница не грузится на некоторых телефонах]]
|definition=
 
<tex> V: O \to \mathbb{R}^m </tex> — локально-потенциальное, если <tex> \forall x \in O \ \exists U(x) \subset O </tex> такое, что <tex> V </tex> — потенциальное в <tex> U(x) </tex>.
 
}}
 
 
 
=== Интеграл локально-потенциального векторного поля по произвольному пути ===
 
{{Определение
 
|definition=
 
Интеграл локально-потенциального векторного поля по произвольному пути равен его интегралу по кусочно-гладкому пути, близкому к данному.
 
}}
 
 
 
=== Гомотопия путей, связанная, петельная гомотопия ===
 
{{Определение
 
|definition=
 
Пусть <tex> \gamma_0, \gamma_1: [a; b] \to O </tex>. <tex> \Gamma: [a; b] \times [0; 1] \to O </tex> — гомотопия этих путей, если она непрерывна и <tex> \forall t \ \Gamma(t, 0) = \gamma_0 (t), \ \Gamma(t, 1) = \gamma_1(t) </tex>. Связанная гомотопия — <tex> \gamma_0 (a) = \gamma_1(a), \ \gamma_0 (b) = \gamma_1(b), \ \forall s \ \Gamma (a, s) = \gamma_0 (a), \ \Gamma (b, s) = \gamma_0 (b) </tex>. Петельная гомотопия — <tex> \gamma_0 (a) = \gamma_0(b), \ \gamma_1 (a) = \gamma_1(b), \ \forall s \in [0, 1] \ \Gamma (a, s) = \Gamma (b, s) </tex>.
 
}}
 
 
 
=== Односвязная область ===
 
???
 
{{Определение
 
|definition=
 
Область <tex> O </tex> — односвязная, если любая петля в <tex> O </tex> стягиваема: <tex> \forall \gamma: [a; b] \to O, \ \gamma(a) = \gamma(b), \ \gamma, \gamma_2 </tex> — петельно гомотопные пути, <tex> \gamma_2: [a; b] \to O, \gamma(t) \equiv \gamma(a) </tex>.
 
}}
 

Текущая версия на 14:50, 29 января 2015

Содержание

Основные вопросы

Признак Вейерштрасса

Теорема:
Рассмотрим ряд [math] \sum u_n(x) [/math], где [math] u_n : E \rightarrow \mathbb{R} [/math] ([math] E [/math]— метрическое пространство). Пусть есть ряд [math] \sum c_n [/math] — сходящийся, такой, что [math] \forall x \in E \ |u_n(x)| \leqslant c_n [/math]. Тогда [math] \sum u_n(x) [/math] равномерно сходится на [math] E [/math].
Доказательство:
[math]\triangleright[/math]
[math] M_n = sup_{x \in E}|S_n(x) - S(x)| = sup|\sum_{n = N + 1}^{+ \infty} u_n(x)| \le sup\sum_{n = N + 1}^{+ \infty}|u_n(x)| \le sup_{x \in E}\sum_{n = N + 1}^{+ \infty}|u_n(x)| \le sup_{x \in E}\sum c_n = \sum_{n = N + 1}^{+ \infty}c_n \xrightarrow[N \rightarrow + \infty]{} 0 [/math]
[math]\triangleleft[/math]

Теорема Стокса--Зайдля для рядов

Теорема:
Пусть ряд [math] \sum u_n(x) [/math], где [math] u_n: X \rightarrow \mathbb{R} [/math] ( [math]X[/math] — метрическое пространство), равномерно сходится на [math] X [/math]. Пусть есть точка [math] x_0 \in X [/math], такая, что все [math] u_n [/math] непрерывны в [math] (\cdot) x_0 [/math]. Тогда [math] S(x) = \sum u_n(x) [/math] непрерывна в точке [math] (\cdot) x_0 [/math].
Доказательство:
[math]\triangleright[/math]

1) [math] S_n(x) = \sum_{n = 1}^{N}u_n(x) [/math] — непрерывна в [math] (\cdot) x_0 [/math]

2) [math] S_n \rightrightarrows_{n \rightarrow + \infty. x \in X} S [/math]

из 1) и 2) [math] \Rightarrow S(x) [/math] непрерывна в [math] (\cdot) x_0 [/math]

Где вы вообще такое доказательство нашли? Тут фигня какая-та. Нормальное доказательство есть в Фихтенгольце.
[math]\triangleleft[/math]

Теорема об интегрировании функционального ряда

Теорема:
Пусть [math] u_n \in C[a, b] [/math] ([math] C [/math] — множество непрерывных функций), [math] \sum u_n(x) [/math] равномерно сходится на [math] [a; b] [/math], [math] S(x) = \sum u_n(x) [/math].

Тогда[math] * [/math] [math] \int\limits_{a}^{b} S(x) dx = \sum_{n=1}^{+\infty} \int\limits_{a}^{b} u_n(x) dx [/math]

[math] * [/math] 1) [math] S(x) [/math] — непрерывно [math] \rightarrow [/math] интеграл имеет смысл.

2) Правая часть имеет смысл — это следует из доказательства.
Доказательство:
[math]\triangleright[/math]

[math] S_n(x) \in C[a, b] \ \ \int\limits_{a}^{b} S_n(x)dx = \sum_{n = 1}^{N}\int\limits_{a}^{b}u_n(x)dx [/math]

Сделаем предельный переход по [math]N[/math]

[math] S_n \rightrightarrows S \ \ \int\limits_{a}^{b} S(x)dx = \sum_{n = 1}^{+ \infty}\int\limits_{a}^{b}u_n(x)dx [/math]
[math]\triangleleft[/math]

Теорема о дифференцировании функционального ряда

Теорема:
Пусть [math] u_n \in C'[a; b] [/math] ([math] C' [/math] — множество непрерывно дифференцируемых функций).

1) [math] \sum_{n = 1}^{+ \infty} u_n(x) = S(x) [/math] поточечно сходится на [math] [a; b] [/math]

2) [math] \sum_{n = 1}^{+ \infty} u'_n(x) = \varphi(x)[/math] равномерно сходится при [math] x \in [a, b] [/math]

Тогда [math] S(x) \in C'[a, b] [/math] и [math] S'(x) = \varphi(x) [/math].
Доказательство:
[math]\triangleright[/math]

Следует из т. о предельном переходе под знаком производной (прошлый семестр).

  • [math] (\lim_{n \to +\infty} f_n) = \lim_{n \to +\infty}(f{'}_n); \ f_n \in C^1[a, b] [/math]
  • [math] f_n \to f [/math] — поточечно на [math] [a, b]. \ f{'}_n \rightrightarrows \varphi [/math] при [math] n \to +\infty, x \in [a, b] [/math]
  • Тогда [math] f [/math] — дифф. на [math] [a, b] \ \forall x \in [a, b] : f{'}(x) = \varphi(x) [/math].
[math] \begin{matrix} S_n \rightarrow S \\ S_{n}' \rightrightarrows \Phi \end{matrix} [/math] Тогда [math] S' = \Phi [/math]
[math]\triangleleft[/math]

Теорема о почленном предельном переходе в суммах

Теорема:
Пусть [math] u_n(x): \left \langle a, b \right \rangle \rightarrow \mathbb{R} [/math], [math] x_0 \in \left \langle a; b \right \rangle [/math].

1) [math] \exists \lim_{x \to x_0} u_n(x) = a_n [/math]

2) [math] \sum u_n(x) [/math] равномерно сходится на [math] \left \langle a, b \right \rangle [/math]

Тогда

1) [math] \sum a_n [/math] — сходится

2) [math] \sum a_n = \lim_{x \to x_0} (\sum_{n=1}^{+\infty} u_n(x) ) [/math]
Доказательство:
[math]\triangleright[/math]

1) [math] S_N = \sum_{n = 1}^{N} u_n(x); S_N^{(a)} = \sum_{n = 1}^{N} a_n ? S_N^{(a)} [/math] — имеет предел

  • Критерий Больцано-Коши [math] \lim S_n^{(a)} = S^{(a)} [/math]
  • [math] \forall \epsilon \gt 0 \ \exists N \ \forall n \gt N \ \forall p : |S_n^{(a)} - S_{n + p}^{(a)}| \lt \epsilon [/math]

[math] |S_n^{(a)} - S_{n + p}^{(a)}| \le |S_n^{(a)} - S_n(x)| + |S_n(x) - S_{n + p}(x)| + |S_{n + p}(x) - S_{n + p}^{(a)}| [/math]

Берём [math] \forall \epsilon \gt 0 [/math] из р. сх-ти

[math] \exists N \ \forall n \gt N \ \forall p \ \forall x : |S_n(x) - S_{n + p}(x)| \lt \frac{\epsilon}{3} [/math]

[math] |S_n(x) - S(x)| \lt \frac{\epsilon}{6} [/math]

[math] |S_{n + p}(x) - S(x)| \lt \frac{\epsilon}{6} [/math]

При данном [math]n : S_n(x) = u_1(x) + \ldots + u_n(x) \xrightarrow[x \rightarrow x_0]{} a_1 + \ldots + a_n = S_n^{(a)} [/math]

Выберем [math] x [/math] так близко к [math] x_0 [/math], чтобы [math] \begin{matrix} |S_n^{(a)} - S_n(x)| \lt \frac{\epsilon}{3} \\ |S_{n + p}(x) - S_{n + p}^{(a)}| \lt \frac{\epsilon}{3} \end{matrix} [/math]

[math]u_n(x); \hat{u}_n(x) := \begin{Bmatrix} u_n(x) & x \ne x_0 \\ a_n & x = x_0 \end{Bmatrix}[/math] — непр. равномерно в [math] (\cdot) x_0 [/math]

[math] \sum \hat{u}_n(x) [/math] — р. сх. на [math] \langle a, b \rangle [/math]

Утв. 2 следует из т. 1. Стокса-Зайдля для рядов

[math] M_n = \sup |\sum_{n = N + 1}^{+ \infty} \hat{u}_n(x)| \le \sup |\sum_{n = n + 1}^{+ \infty} u_n(x)| + |\sum_{n = N + 1}^{+ \infty} a_n| \xrightarrow[N \rightarrow +\infty]{} 0 [/math]
[math]\triangleleft[/math]

Теорема о перестановке пределов

([math] \lim_{n \to + \infty} \ \lim_{x \to 0} = \lim_{x \to 0} \ \lim_{n \to + \infty} [/math])

Теорема:
Пусть [math] f_n: X \rightarrow \mathbb{R} [/math], [math] x_0 \in X [/math] [или даже [math] x_0 [/math] — предельная точка [math] X [/math]]

1) [math] f_n(x) [/math] сходится равномерно к [math] S(x) [/math] при [math] n \to + \infty, \ x \in X [/math]

2) [math] f_n(x) \underset{x \to x_0}{\rightarrow} A_n [/math]

Тогда

1) [math] \exists lim_{n \to + \infty} A_n = A \in \mathbb{R} [/math]

2) [math] S(x) \underset{x \to x_0}{\rightarrow} A [/math]
Доказательство:
[math]\triangleright[/math]

[math] u_1 = f_1; \ u_2 = f_2 - f_1; \ u_3 = f_3 - f_2; [/math]

Тогда: [math] f_N(x) = \sum_{n = 1}^{N}u_n(x) [/math]

Условие 1: [math] \sum u_n [/math] р. сх. к сумме [math] S(x) [/math]

[math] u_n = f_n - f_{n - 1} [/math]

Условие 2: [math] lim_{x \rightarrow x_0}u_n(x) = a_n = A_n - A_{n - 1} [/math] (при [math] n = 1[/math] проявить сообразительность)

[math] A_n = \sum_{k = 1}^{n}a_k [/math]

по теореме о почл. пр. переходе в суммах:

1) [math] \sum a_k [/math] — сх., т.е. [math]\exists lim_{n \rightarrow + \infty} A_n = A[/math]

2) [math] \sum a_n = lim_{x \rightarrow x_0}(\sum u_n(x)) [/math]

[math] S(x) \xrightarrow[x \rightarrow x_0]{} A [/math]
[math]\triangleleft[/math]

Замечание: верна теорема [math] f(x, y) [/math]

[math] lim_{x \rightarrow x_0}(lim_{y \rightarrow y_0}f(x, y)) = lim_{y \rightarrow y_0}(lim_{x \rightarrow x_0}f(x, y)) [/math]

при условии 1: [math] \exists lim_{y \rightarrow y_0} f(x, y) = g(x) [/math] — и этот предел равномерный

[math]\exists lim_{x \rightarrow x_0}f(x, y) = h(y)[/math]

Признак Дирихле равномерной сходимости функционального ряда

Теорема:
Пусть есть ряд [math] \sum a_n(x) b_n(x) [/math], [math] x \in X [/math]

1) частичные суммы ряда [math]a_n(x)[/math] равномерно ограничены, т.е. [math] \exists c_a \ \forall x | \sum_{k = 1}^{n} a_k(x) | \leqslant c_a [/math]

2) [math] b_n(x) [/math] монотонна по [math] n [/math] и равномерно сходится к [math] 0 [/math]

Тогда [math] \sum a_n(x) b_n(x) [/math] равномерно сходится на [math] X [/math].
Доказательство:
[math]\triangleright[/math]

Применяя преобразование Абеля

[math]\sum_{k=n+1}^{n+p}b_k(x)a_k(x) = b_{n+p}(x)\sum_{k = 1}^{n + p}a_k(x)-\sum_{k=n+1}^{n+p-1}(b_{k+1}(x)-b_k(x))\sum_{j=1}^{k}a_j(x)[/math]

В силу равномерной ограниченности частичных сумм ряда [math]\sum a_k(x)[/math] при некотором [math]M[/math]

[math]|\sum_{k = 1}^{n}a_k(x)| \le M \ \forall n \in N, \forall x \in X[/math]

Тогда, используя монотонность [math]b_k(x)[/math] (по [math]k[/math]), имеем

[math]|\sum_{k=n+1}^{n+p}b_k(x)a_k(x)| \le M|b_{n+p}(x)|+M \sum_{k = n + 1}^{n+p-1}|b_{k+1}(x)-b_k(x)|= 2M|b_{n+p}(x)|+M|b_{n+1}(x)|[/math]

Из этого неравенства в силу [math]b_k \rightrightarrows 0[/math] получаем, что

[math]\forall \varepsilon \gt 0 \ \exists n(\varepsilon ) : |\sum_{k=n+1}^{n+p}b_k(x)a_k(x)| \lt \varepsilon \ \forall n \ge n(\varepsilon), \forall p \in N, \forall x \in X[/math]

Применяя критерий Коши, получаем, что ряд сходится равномерно на [math]X[/math].
[math]\triangleleft[/math]

Метод суммирования Абеля

Теорема:
Пусть [math] \sum a_n [/math] сходится. Рассмотрим функцию [math] f(x) = \sum a_n x^n [/math]. Тогда [math] \sum a_n = \lim_{x \to 1 - 0} f(x) [/math].
Доказательство:
[math]\triangleright[/math]

[math]a_n, b_n = x^n; \ X = [0, 1][/math]

[math] \sum a_n b_n [/math]по признаку Абеля равномерно сх-ся [math][0, 1][/math]

[math]lim \ a_n x^n \xrightarrow[x \rightarrow 1 - 0]{} a_n [/math]
[math]\triangleleft[/math]

Теорема о круге сходимости степенного ряда

Теорема:
Пусть [math] (A) [/math] [math] \sum_{k=0}^{+ \infty} a_k(z-z_0)^k [/math] — произвольный степенной ряд [math] [ a_k \in \mathbb{C}, z [/math] — комплексная переменная [math] ] [/math] или [math] [ a_k \in \mathbb{R}; z, z_0 \in \mathbb{R} ] [/math]

Возможны три случая:

1) [math] \forall z \in \mathbb{C} [/math] ряд [math] (A) [/math] сходится

2) [math] (A) [/math] сходится только при [math] z = z_0 [/math]

3) [math] \exists R [/math] [math] 0 \lt R \lt + \infty [/math] при

[math] |z - z_0| \lt R [/math] сходится

[math] |z - z_0| \gt R [/math] расходится

[math] R [/math] — радиус сходимости
Доказательство:
[math]\triangleright[/math]

Нужно доказать абсолютную сходимость

[math] \sum |a_k| \cdot |z - z_0|^k [/math]

  • Признак Коши: [math] \overline{lim}_{n \to + \infty} \sqrt[n]{|a_n| \cdot |z - z_0|^n} = \overline{lim}_{n \to + \infty} \sqrt[n]{|a_n|} \cdot |z - z_0| = |z - z_0| \cdot\overline{lim}_{n \to + \infty} \sqrt[n]{|a_n|} [/math]

1) [math] \overline{lim} = 0 [/math] при всех [math] z [/math] ряд [math] (A) [/math] сходится абсолютно

2) [math] \overline{lim} = + \infty [/math] при [math] z = z_0 \text{ } lim_{n \to + \infty} \sqrt[n]{|a_n| \cdot |z - z_0|^n} = 0 [/math], т.е. ряд сходится

при [math] z \ne z_0 \text{ } lim \sqrt[n]{...} = + \infty [/math] расходится (слагаемые [math] \nrightarrow 0 [/math])

3) [math] \overline{lim} \sqrt[n]{a_n} [/math] — конечен [math] = \frac{1}{R} [/math]

[math] |z - z_0| \lt R [/math] ряд [math] (A) [/math] сходится абсолютно

[math] |z - z_0| \gt R [/math] расходится (слагаемые [math] \nrightarrow 0 [/math])
[math]\triangleleft[/math]

Теорема о равномерной сходимости и непрерывности степенного ряда

Теорема:
Пусть ряд [math] (A) = \sum a_n(z - z_0)^n, 0 \lt R \le + \infty [/math] — радиус сходимости. Тогда:

1) Для [math] r : 0 \lt r \lt R [/math] ряд [math] (A) [/math] равномерно сходится в круге [math] \overline{B(z_0, r)} [/math]

2) В круге [math] B(z_0, R) [/math] сумма ряда [math] (A) [/math] — непрерывна.
Доказательство:
[math]\triangleright[/math]

(1) Признак Вейерштрасса

[math] z \in \overline{B(z_0, r)} [/math]

[math] |a_n(z - z_0)^n| = |a_n| \cdot r^n [/math]

[math] \sum |a_n| \cdot r^n [/math] — сходится! т.к. [math] \sum a_n \cdot r^n [/math] — абс. сх.

[math] (z := z_0 + r \in B(z_0, R)) [/math]

(2) фиксируем [math] z \in B(z_0, R) [/math]; Возьмём [math] r : |z - z_0| \lt r \lt R [/math]

В [math] B(z_0, r) [/math] ряд р. сх. и слагаемые непр. [math] \Rightarrow [/math] сумма непрерывна.
[math]\triangleleft[/math]

Линейные и комплексно линейные отображения. Уравнения Коши--Римана

Лемма:
Пусть [math] f: E \subset \mathbb{C} \to \mathbb{C}, \ z_0 \in \operatorname{Int} E, \ f [/math] — комплексно дифференцируема в точке [math] z_0 [/math]. Тогда, если [math] f \leftrightarrow F: \mathbb{R}^2 \to \mathbb{R}^2, \ (x, y) \mapsto (\operatorname{Re}{f(x + iy)}, \operatorname{Im}{f(x + iy)} ) [/math], отображение [math] F [/math] дифференцируемо в [math] (x_0, y_0) [/math] и выполнены соотношения:

[math] \frac{\partial F_1}{\partial x} (x_0, y_0) = \frac{\partial F_2}{\partial y} (x_0, y_0) [/math]

[math] \frac{\partial F_1}{\partial y} (x_0, y_0) = - \frac{\partial F_2}{\partial x} (x_0, y_0) [/math]

(уравнения Коши-Римана)
Доказательство:
[math]\triangleright[/math]
Википедия [1]
[math]\triangleleft[/math]

Теорема о почленном дифференцировании степенного ряда

Теорема:
Ряд [math] (A) = \sum a_n(z - z_0)^n = f(z), R \in [0, + \infty], |z - z_0| \lt R [/math]

Ряд [math] (A)' = \sum_{n = 1}^{+ \infty} n a_n (z - z_0)^{n - 1} [/math]

Тогда: 1) радиус сх-ти [math] (A') = R [/math]. 2) при [math] |z - z_0| \lt R; f'(z) = \sum n a_n (z - z_0)^{n - 1} [/math]

[Тогда [math]f[/math] — дифф. при [math] |z - z_0| \lt r [/math] и [math] f'(z) = \sum n a_n (z - z_0)^{n - 1} [/math] ]
Доказательство:
[math]\triangleright[/math]

[math]R = \frac{1}{\overline{\lim}\sqrt[n]{|a_n|}}; R_A = \frac{1}{\overline{\lim}\sqrt[n]{(n + 1)|a_{n + 1}|}} = R[/math]

[math] \frac{f(z + h) - f(z)}{h} = \sum \frac{a_n (z + h - z_0)^n - a_n (z - z0)^n }{h} = \sum a_n \frac{(z + h - z_0) - (z - z_0)^n}{h} [/math]

Проверим р. сх. [math] z \in B(z_0, r), r \lt R [/math]; [math] ]h : |h| \le r - |z - z_0| [/math]

Тогда: [math] z + h \in \overline{B(z_0, r)}; |z + h - z_0| \le r; |z - z_0| \le r [/math]

[math] |a_n \frac{(z + h - z_0)^n - (z - z_0)^n}{h}| \le \frac{|a_n|}{|h|} n r^{n - 1} |h| = |a_n| n r^{n - 1} [/math]

[math] \sum h|a_n|r^{n - 1} [/math] — сх. [math]\Rightarrow[/math] по признаку Вейерштрасса р. сх. при [math] |h| \lt r - |z - z_0| [/math]

[math] f(z) = \lim_{h \rightarrow 0} \frac{f(z + h) - f(z)}{h} = \sum \lim a_n \frac{(z + h - z_0)^n - (z - z_0)^n}{h} = \sum n(z - z_0)^{n - 1} a_n [/math]
[math]\triangleleft[/math]

Экспонента, синус, косинус. Свойства.

1.1) [math] \mathrm{exp}(0) = 1 [/math]

1.2) [math] \mathrm{exp}(\overline{z}) = \overline{\mathrm{exp}(z)}; \ /S_n(\overline{z}) = \overline{S_n(x)})/[/math]

1.3) [math] (\mathrm{exp}(z))' = \mathrm{exp}(z); \ /\sum_{n = 1}^{+ \infty} (\frac{z^n}{n!})' = \sum_{n = 1}^{+ \infty} \frac{z^{n - 1}}{(n - 1)!} = \sum_{n = 0}^{+ \infty} \frac{z^n}{n!}/ [/math]

1.4) [math] (\mathrm{exp}(x))'|_{x = 0} = 1 [/math]

Теорема:
[math] \forall z, w \in \mathbb{C} : \mathrm{exp}(z + w) = \mathrm{exp}(z) ⋅ \mathrm{exp}(w) [/math]
Доказательство:
[math]\triangleright[/math]

[math] \sum \frac{z^n}{n!} \cdot \sum \frac{w^k}{k!} [/math]

[math] \sum_{k = 0}^{+ \infty} \frac{(z + w)^k}{k!} = \sum_{k = 0}^{+ \infty} \sum_{l = 0}^{k} \frac{z^l}{l!} \cdot \frac{w^{k - l}}{(k - l)!} = \sum_{l = 0}^{+ \infty} \sum_{k = l}^{+ \infty} \frac{z^l}{l!} \cdot \frac{w^{k - l}}{(k - l)!} = [/math]

[math] = \sum_{l = 0}^{+ \infty} \sum_{n = 0}^{+ \infty} \frac{z^l}{l!} \cdot \frac{w^n}{n!} = \sum_{l = 0}^{+ \infty}(\frac{z^l}{l!} \cdot \sum_{n = 0}^{+ \infty} \frac{w^n}{n!}) = (\sum \frac{w^n}{n!})(\sum \frac{z^l}{l!}) [/math]
[math]\triangleleft[/math]
  • Следствие: [math] \mathrm{exp}(z) \ne 0 [/math] — ни при каких [math] z [/math]

2.1) [math] \sin x = \frac{\mathrm{exp}(ix) - \mathrm{exp}(-ix)}{2i} [/math]

2.2) [math] \cos x = \frac{\mathrm{exp}(ix) + \mathrm{exp}(-ix)}{2} [/math]

2.3) [math] \cos(z) = \sum_{n=0}^{+\infty} (-1)^n \frac{z^{2n}}{(2n)!} [/math]

2.4) [math] \sin(z) = \sum_{n=0}^{+\infty} (-1)^n \frac{z^{2n - 1}}{(2n - 1)!} [/math]

2.5) Пусть [math] T(x) = \mathrm{exp}(ix) [/math]

[math] T(x+y) = T(x)T(y) [/math]

[math] \cos(x + y) = \cos(x)\cos(y) - \sin(x)\sin(y) [/math]

[math] \sin(x + y) = \cos(x)\sin(y) + \cos(y)\sin(x) [/math]

2.6) [math] |T(x)| = 1; \ \cos^2(x) + \sin^2(x) = 1 [/math]

[math] (\frac{T(x) + T(-x)}{2})^2 + (\frac{T(x) - T(-x)}{2i})^2 = T(x)T(-x) = T(0) = \mathrm{exp}(i0) = 1 [/math]

2.7) [math] \lim_{x \to 0} \frac{\sin(x)}{x} = 1; \ \lim_{x \to 0} \frac{1 - \cos(x)}{x^2} = \frac{1}{2}[/math]

[math] \lim_{x \to 0} (\frac{\mathrm{exp}(ix) - 1}{ix}) = \lim_{x \to 0} (\frac{\cos(x) - 1}{ix} + \frac{i \sin(x)}{ix}) [/math]


[math] x \in \mathbb{C} \begin{cases} e^x = 1 + x + \frac{x^2}{2} + \ldots \\ \sin(x) = x + \frac{x^3}{3} + \ldots \\ \cos(x) = 1 - \frac{x^2}{2} + \ldots \end{cases} [/math]

[math] |x| \lt 1 \begin{cases} (1 + x)^\alpha = 1 + \alpha x + \frac{\alpha (\alpha - 1)}{2} x^2 + \ldots \\ \frac{1}{1-x} = 1 + x + x^2 + \ldots \\ \ln(1 + x) = x - \frac{x^2}{2} + \frac{x^3}{3} - \ldots \end{cases}[/math]

[math] \sum a_k \to [/math] Абель [math] \to \sum a_k \cdot x^k = f(x); \lim_{x \to 1- 0}f(x) = S [/math]

Единственность производной

Теорема:
Производный оператор единственный.
Доказательство:
[math]\triangleright[/math]

Покажем, что значение производного оператора [math]A[/math] на каждом векторе [math]h\in\mathbb{R}^n[/math] определяется однозначно. По линейности оператора [math]A\mathbb{O}_n=\mathbb{O}_m[/math]. Зафиксируем [math]h\ne\mathbb{O}_n[/math]. Возьмём достаточно малое по модулю [math]t\in\mathbb{R}\backslash\{0\}[/math] (достаточно взять [math]|t|\in\mathbb{R}\left(0, {r\over |h|}\right)[/math], где [math]B(x, r)\subset D[/math]) и подставим [math]th[/math] вместо [math]h[/math] в равенство из определения. По линейности [math]A[/math] имеем:

[math]f(x+th)=f(x)+tAh+o(t), t\to0[/math].

Перенеся [math]f(x)[/math] в левую часть и разделив на [math]t[/math], получим:

[math]{f(x+th)-f(x)\over t}=Ah+{o(t)\over t}\underset{t\to0}\to Ah[/math],

то есть

[math]Ah=\underset{t\to0}\lim{{f(x+th)-f(x)}\over{t}}[/math].
[math]\triangleleft[/math]

Лемма о покоординатной дифференцируемости

Лемма:
Дифференцируемость отображения [math]f[/math] в точке [math]x[/math] равносильна одновременной дифференцируемости всех его координатных функций [math]f_i[/math] в точке [math]x[/math].
Доказательство:
[math]\triangleright[/math]

Пусть [math]f[/math] дифференцируемо в точке [math]x[/math]. Запишем равенство из определения производного оператора покоординатно:

[math]f_i(x+h)=f_i(x)+A_i h+\alpha_i(h)|h|, i\in[1:m][/math].

Координатные функции [math]A_i[/math] линейного оператора [math]A[/math] являются линейными, а непрерывность и равенство нулю в нуле отображения [math]\alpha[/math] равносильно такому же свойству его координатных функций [math]\alpha_i[/math]. Поэтому для [math]f_i[/math] выполнено определение дифференцируемости.

Обратно, пусть [math]f_i[/math] дифференцируемы в точке [math]x[/math]. Тогда для каждого [math]i\in[1:m][/math] существует линейная функция [math]A_i[/math] и функция [math]\alpha_i[/math], непрерывная и равная нулю в нуле, для которых выполняется равенство. Следовательно, для [math]f[/math] выполняется равенство из определения производного оператора, где [math]A[/math] — оператор с координатными функциями [math]A_i[/math].
[math]\triangleleft[/math]

Необходимое условие дифференцируемости.

Теорема:
Пусть [math] f : E \subset \mathbb{R}^m \rightarrow \mathbb{R} [/math] — дифференцируемо в точке [math] a \in \operatorname{Int}(E) [/math]

Тогда [math] \forall x \ \exists {\partial f\over\partial x_k}(a) [/math] и матрица Якоби [math] f'(a) = ({\partial f\over\partial x_1}(a), \ldots, {\partial f\over\partial x_m}(a)) [/math]

Замечание: Для [math] F : E \rightarrow \mathbb{R}^l [/math] — дифференцируемо в точке [math] a [/math]; [math]F'(a) = ({\partial f_i\over\partial x_j})_{i = 1 \ldots l; j = 1 \ldots m} [/math]
Доказательство:
[math]\triangleright[/math]

[math]f(a + h) = f(a) + f'(a) \cdot h + o(h)[/math]

[math] h := (0, \ldots, 0, t, 0, \ldots, 0) [/math]

[math] f(a_1, \ldots, a_k + t, \ldots, a_m) = f(a_1 \ldots a_m) + (f'(a))_k \cdot t + o(t) [/math] — это св-во дифф-ти [math] \varphi_k [/math] в [math] \cdot (a) [/math] из опр. частн. производных.

[math] {o(h)\over ||L||} \rightarrow 0 [/math]
[math]\triangleleft[/math]

Достаточное условие дифференцируемости

Теорема:
Пусть [math] f : E \subset \mathbb{R}^m \rightarrow \mathbb{R}; \ \exists r \ B(a, r) \subset E [/math], в шаре [math]B(a, r) [/math] существуют все [math] f'x_k, k = {1..m} [/math] и все производные непрерывны в точке [math] a[/math]. Тогда [math] f [/math] дифференцируема в точке [math] a[/math]
Доказательство:
[math]\triangleright[/math]

[math] m = 2 [/math]

[math] f(x_1, x_2) - f(a_1, a_2) = (f(x_1, x_2) - f(x_1, a_2)) + (f(x_1, a_2) - f(a_1, a_2)) =^* [/math] // [math] =^* [/math] — По теореме Лагранжа

// [math] \varphi_2(t) = f(x, t); \varphi_2(x_2) - \varphi_2(a_2) = \varphi'_2(t) \cdot (x_2 - a_2) [/math] // [math] t [/math] — средняя точка

[math] =^* \frac{\partial f}{\partial x_2}(x_1, \bar x_2)(x_2 - a_2) + \frac{\partial f}{\partial x_1}(\bar x_1, a_2)(x_1 - a_1) = [/math][math] \frac{\partial f}{\partial x_2}(a_1, a_2)(x_2 - a_2) + \frac{\partial f}{\partial x_1}(a_1, a_2)(x_1 - a_1) + [/math]

[math] o(\begin{bmatrix} x_1 - a_1 \\ x_2 - a_2 \end{bmatrix}) \to ||\ldots|| = \sqrt{(x_1 - a_1)^2 + (x_2 - a_2)^2} \begin{cases} + [\frac{\partial f}{\partial x_2}(x_1, \bar x_2) - \frac{\partial f}{\partial x_2}(a_1, a_2)](x_2 - a_2) + \\ [\frac{\partial f}{\partial x_1}(\bar x_1, a_2) - \frac{\partial f}{\partial x_1}(a_1, a_2)](x_1 - a_1) \end{cases}[/math]

[math][\ldots] \cdot \frac{x - a}{\sqrt{(x_1 - a_1)^2 + (x_2 - a_2)^2}} \ \[/math] где: [math] \frac{x - a}{\sqrt{(x_1 - a_1)^2 + (x_2 - a_2)^2}} \le 1 [/math] по модулю; [math] [\ldots] \to 0 [/math] при [math] (x_1, x_2) \to (a_1, a_2) [/math]
[math]\triangleleft[/math]

Лемма об оценке нормы линейного оператора

Лемма:
Пусть [math] A: \mathbb{R}^m \to \mathbb{R}^l [/math] — линейный оператор. Тогда [math] ||Ax|| \le C_A||x|| [/math], где [math] C_A = \sqrt{\sum_{i, j} a_{i, j}^2} [/math] ([math] a_{i, j} [/math] — элементы его матрицы)
Доказательство:
[math]\triangleright[/math]

[math] ||x|| = 0 [/math], т.е. если [math] x = 0 [/math], то тривиально

[math] ||Ax||^2 = \sum_{i = 1}^{l}(\sum_{j = 1}^{m}a_{i, j}x_{j})^2 \le [/math] (КБШ) [math] \sum_{i = 1}^{l}((\sum_{j = 1}^{m}a_{i, j}^{2})(\sum_{j = 1}^{m}x_{j}^{2})) = (\sum_{i = 1}^{l}\sum_{j = 1}^{m}a_{i, j}^{2})(\sum_{j = 1}^{m}x_{j}^{2}) [/math]

[math] x^{(k)} \rightarrow x [/math]

[math]||x^{(k)} - x|| \rightarrow 0 [/math]

[math] Ax^{(k)} \xrightarrow{?} Ax [/math]

[math] ||A(x^{(k)} - x)|| \le C_A||x_k - x|| [/math]
[math]\triangleleft[/math]

Дифференцирование композиции

Теорема:
[math] F : E \subset \mathbb{R}^m \to \mathbb{R}^l; \ a \in IntE, F(E) \subset I [/math]

[math] G : I \subset \mathbb{R}^l \to \mathbb{R}^n; \ b = F(a) \in IntI [/math]

[math] F [/math] — дифф. в [math] (\cdot) a, G [/math] — дифф. в [math] (\cdot) b [/math];

[math] H = G \circ F \ // H(x) = G(F(x)) [/math]

Тогда: [math] H [/math] — дифф. в [math] (\cdot) a; H'(a) = G'(F(a)) \cdot F'(a) [/math]
Доказательство:
[math]\triangleright[/math]

[math] F(a + h) = F(a) + F'(a)h + \alpha(h)||h||; \ // \alpha(h) \xrightarrow[h \to 0]{} 0 [/math]

[math] G(b + k) = G(b) + G'(b)k + \beta(k)||k||; \ // \beta(k) \xrightarrow[k \to 0]{} 0 [/math]

[math] H(a + h) = G(F(a + h)) = G(\overbrace{F(a)}^{b} + \overbrace{F'(a)h + \alpha(h)||h||}^{k}) = [/math][math] G(b) + G'(b)(F'(a)h + \alpha(h)||h||) + \beta(k)||k|| = [/math]

[math] = \overbrace{G(F(a)) + G'(F(a) \cdot F'(a)h)}^{H(a)} + \overbrace{G'(b)\alpha(h)||h|| + \beta(k)||k||}^{? o(h) \leftarrow \text{proverim}} [/math]

1. [math] ||\ G'(b)\alpha(h)\|h\| \ || = \|h\| \cdot ||G'(b)\alpha(h)|| \le \|h\|\cdot C_{G(b)} \cdot ||\alpha(h)|| = o(h) [/math]

2. [math] \beta(k)||k|| [/math]

[math] \|k\| = || \ F'(a)h + \alpha(h)\|h\| \ || \le \overbrace{||F'(a)h||}^{C_{F'(a)} \cdot \|h\|} + \|\alpha(h)\|\cdot\|h\| \le (C_{F'(a)} + \|\alpha(h)\|\cdot \|h\|) [/math]

[math] ||\ \beta(k)\cdot \|k\| \ || \le \overbrace{||\beta{k}||}^{\to 0, h \to 0} \cdot \overbrace{(C_{F'(a)} + ||\alpha(h)||)}^{ogr. pri: \ h \to 0} \cdot \|h\| = o(h)[/math]

[math] F = (f_1(x_1 \ldots x_m), f_2(x_1 \ldots x_m), \ldots, f_l(x_1 \ldots x_m)) [/math]

[math] G = (g_1(y_1 \ldots y_l), \ldots, g_n(y_1 \ldots y_l)) [/math] [math] H = \overbrace{g_1}^{h_1}(f_1(x_1 \ldots x_n), \ldots, f_l(x_1 \ldots x_n)), \ldots, \overbrace{g_n}^{h_n}(f \ldots)) [/math]

[math] \frac{\partial h_i}{\partial x_j}(a) = \frac{\partial g_i}{\partial y_1}(b) \cdot \frac{\partial f_1}{\partial x_j}(a) + \frac{\partial g_i}{\partial y_2}(b) \cdot \frac{\partial f_2}{\partial x_j}(a) + \ldots + \frac{\partial g_i}{\partial y_l}(b) \cdot \frac{\partial f_l}{\partial x_j}(a) [/math]
[math]\triangleleft[/math]

Дифференцирование «произведений»

Лемма:
Пусть [math] F, G: \ E \subset \mathbb{R}^m \to \mathbb{R}^l [/math], [math] \lambda: E \to \mathbb{R} [/math], [math] a \in \operatorname{Int} E [/math]; [math] F, G, \lambda [/math] — дифференцируемые в [math] a [/math]. тогда:

1) [math] (\lambda F)' (a) h = ( \lambda'(a) h ) F(a) + \lambda(a) (F'(a) h) [/math]

2) [math] \left \langle F, G \right \rangle ' (a) h = \left \langle F'(a) h, G(a) \right \rangle + \left \langle F(a), G'(a) h \right \rangle [/math]

(здесь [math] \left \langle a, b \right \rangle [/math] — скалярное произведение [math] a [/math] и [math] b [/math])
Доказательство:
[math]\triangleright[/math]

1. Введём координатную ф-ю [math] F = (f_1 \ldots f_l) [/math]

[math] (\lambda f_i)'(a)h = (\lambda'(a)(h))f_i(a) + \lambda(a)(f'_i(a)h) [/math][math]i[/math]-ая коорд. док. ф-лы; [math] ]f_i \leftrightarrow f [/math]

[math] \lambda(a + h)f(a + h) - \lambda(a)f(a) = (\lambda(a + h) - \lambda(a))f(a + h) + \lambda(a)(f(a + h) - f(a)) = (\lambda'(a)h + o(h))f(a + h) + \lambda(a)(f'(a)h + o(h)) = [/math]

[math] = (\lambda'(a)h) \cdot f(a) + \lambda(a)f'(a)h + (\lambda'(a)h)(f(a + h) - f(a)) + o(h)f(a + h) + \lambda(a) \cdot o(h) [/math]

[math] || \frac{1 slag.}{||h||} || = \frac{|\lambda'(a)h|\cdot||f(a + h) - f(a)||}{||h||} \le \frac{||\lambda'(a)||\cdot||h||\cdot||f(a + h) - f(a)||}{||h||} \rightarrow 0 [/math]

[math] ||2 slag.|| = |o(h)| \cdot ||f(a + h)|| = o(h); \ \ ||f(a + h)|| [/math] — ограничена.

[math] ||3 slag.|| = ||\lambda(a) \cdot o(h)|| = |\lambda(a)| \cdot ||o(h)|| = o(h) [/math]

2. [math] \left \langle F, G \right \rangle ' (a)h = (\sum_{i = 1}^{l}f_i g_i)'(a)h = [/math] лин. дифф. [math] \sum(f_i g_i)'(a)h = \sum(f'_i(a)h)g_i(a) [/math][math] + f_i(a)(g'_i(a)h) = \left \langle F'(a)h, G(a) \right \rangle + \left \langle F(a), G'(a)h \right \rangle [/math]

Замечание: [math]m = 1; \ F, G : \mathbb{R} \rightarrow \mathbb{R}^l [/math]

[math] \left \langle F, G \right \rangle ' (a) = \left \langle F'(a), G(a) \right \rangle + \left \langle F(a), G'(a) \right \rangle [/math]
[math]\triangleleft[/math]

Теорема Лагранжа для векторнозначных функций

Теорема:
[math] F : [a, b] \rightarrow \mathbb{R}^l; F [/math] — непр. на [math] [a, b] [/math] и дифф. на [math] [a, b] [/math] Тогда: [math] \exists c_{G(a, b)} : ||F(b) - F(a)|| \le ||F'(c)|| \cdot |b - a| [/math]
Доказательство:
[math]\triangleright[/math]

[math]\varphi (t) := \langle F(b) - F(a), F(t) \rangle; t \in [a, b]; (\varphi : [a, b] \rightarrow \mathbb{R}) [/math]

[math] \varphi(b) - \varphi(a) = \langle F(b) - F(a), F(b) - F(a) \rangle = ||F(b) - F(a)||^2 [/math]

[math] \begin{matrix} \varphi'(t) = \langle F(b) - F(a), F'(t) \rangle \\ \varphi(b) - \varphi(a) = \varphi'(c)(b - a) \end{matrix} [/math]

[math] ||F(b) - F(a)|| \le ||F'(c)||(b - a) [/math]

// Если ехать быстро и криво

[math] F : \mathbb{R} \rightarrow \mathbb{R}^2; t \rightarrow (\cos t, \sin t) [/math]

[math] F' = (-\sin t, \cos t); ||F'(t)|| = 1 [/math] при [math] \forall t [/math]

[math] ||F(b) - F(a)|| \ne ||F'(c)|| \cdot (b - a) [/math]

// [math]||F'(x)|| = 1; (b - a) [/math] — длина дуги; [math] ||F(b) - F(a)|| [/math] — длина хорды
[math]\triangleleft[/math]

Экстремальное свойство градиента

Теорема:
[math] f : E \subset \mathbb{R}^m \rightarrow \mathbb{R}; f [/math] — дифф. в [math] (\cdot) a, \nabla f(a) \ne 0 [/math]

[math] l = \frac{\nabla f(a)}{||\nabla f(a)||} [/math] — направление

Тогда [math] l [/math] указывает напр-е наискорейшего возр. ф-и, а [math] -l [/math] самого быстрого убывания.

Более того: [math] \forall [/math] напр. [math] u : -||\nabla f(a)|| \le \frac{\partial f}{\partial u}(a) \le ||\nabla f(a)|| [/math] равенство достижимо для [math] u = \pm l [/math]
Доказательство:
[math]\triangleright[/math]

[math] -||\nabla f(a)|| \cdot ||u|| \le \frac{\partial f}{\partial u}(a) \le ||\nabla f(a)|| \cdot ||u|| [/math] // [math] u = 1 [/math]

// [math] \frac{\partial f}{\partial u}(a) = \langle \nabla f(a), u \rangle [/math]
[math]\triangleleft[/math]

Независимость частных производных от порядка дифференцирования

Теорема:
[math] f : E \subset \mathbb{R}^2 \to \mathbb{R}; \ a \in IntE [/math]

[math] \frac{\partial f}{\partial x_1}, \frac{\partial f}{\partial x_2} [/math] — опр. в окр. [math] (\cdot) a [/math], дифф. в окр. [math] (\cdot) a [/math]

[math] \frac{\partial^2 f}{\partial x_1 \partial x_2} [/math] и [math] \frac{\partial^2 f}{\partial x_2 \partial x_1} [/math] — непр. в [math] (\cdot) a [/math]

Тогда эти две частные производные равны.
Доказательство:
[math]\triangleright[/math]

[math] \vartriangle^2 f(h, k) = f(a_1 + h, a_2 + k) - f(a_1 + h, a_2) - f(a_1, a_2 + k) + f(a_1, a_2) [/math] — задано при [math] |h|, |k| \lt r; V(a) = B(a, 2r) [/math]

фикс. [math]k: \varphi(h) = f(a_1 + h, a_2 + k) - f(a_1 + h, a_2) [/math]

[math] \vartriangle^2 f(h, k) = \varphi(h) - \varphi(0) \overbrace{=}^{t. Lagrange} \varphi'(\bar h)h = [/math][math] (f'_{x_1}(a_1 + \bar h, a_2 + k) - f'_{x_1}(a + \bar h, a_2) )h \overbrace{=}^{t. Lagrange} f''_{x_1 x_2}(a_1 + \bar h, a_2 + \bar k)hk [/math]

[math] \bar h, \bar k [/math] — средние точки

[math] \psi(k) = f(a_1 + h, a_2 + k) - f(a_1, a_2 + k) [/math]

[math] \vartriangle^2 f(h, k) = f''_{x_2 x_1}(a_1 + \hat h, a_2 + \hat k)hk [/math]

[math] f''_{x_2 x_1}(a_1 + \hat h, a_2 + \hat k) = f''_{x_1 x_2}(a_1 + \bar h, a_2 + \bar k) \Rightarrow f''_{x_2 x_1} = f''_{x_1 x_2} [/math]
[math]\triangleleft[/math]
  • Замечание 1:

Аналогично: [math] i, j : 1 \le i, j \le m; i \ne j [/math]

[math] \frac{\partial f}{\partial x_i}, \frac{\partial f}{\partial x_j} [/math] — опр. в окр. [math] (\cdot) a; \frac{\partial^2 f}{\partial x_i \partial x_j}, \frac{\partial^2 f}{\partial x_j \partial x_i} [/math] — непр. в [math] (\cdot) a [/math]

  • Замечание 2:

Если [math] f [/math] сущ. част. пр. [math]k[/math]-того порядка в окр. [math](\cdot)a[/math] и все они непр. в [math](\cdot)a[/math]

Для [math] \forall i_1 \ldots i_k [/math] — индексы [math] \in \{ 1 \ldots m \} [/math]

и [math] \forall j_1 \ldots \j_k [/math] — которые получаются из набора [math] i_1 \ldots i_k [/math] перестановка

Верно: [math] \frac{\partial^k f}{\partial x_{i_1} \ldots \partial x_{i_k}}(a) = \frac{\partial^k f}{\partial x_{j_1} \ldots \partial x_{j_k}}(a) [/math]

Полиномиальная формула

Лемма:
Если [math] r \in \mathbb{Z}_+ [/math], [math] k [/math] — мультииндекс, [math] a [/math] - вектор, то [math] (a_1 + ... + a_m)^r = \sum_{k: (k) = r} \frac{r!}{k!} a^{k} [/math]
Доказательство:
[math]\triangleright[/math]

Индукция по [math]r[/math]

[math] r = 1 [/math]

[math] k = (0, 0, \ldots, \overbrace{1}^{k}, 0, \ldots); a_k \cdot \frac{1!}{0!0! \ldots 1!0! ...} = 1 [/math]

[math] r = r + 1 [/math]

[math] (a_1 + ... + a_m)^{r + 1} = (a_1 + ... + a_m) \cdot \sum \frac{r!}{k_1! ... k_m!} \cdot a_1^{k_{1}} ... a_m^{k_{m}} = [/math]

[math] = \sum \frac{r!}{k_1! ... k_m!} \cdot a_1^{k_{1}+1} ... a_m^{k_{m}} + \sum \frac{r!}{k_1! ... k_m!} \cdot a_1^{k_{1}} a_2^{k_2 + 1} ... a_m^{k_{m}} + [/math][math] \sum \frac{r!}{k_1! ... k_m!} \cdot a_1^{k_{1}} ... a_{m-1}^{k_{m - 1}} a_m^{k_{m} + 1} = [/math]

[math] = \sum_{\beta : |\beta| = r + 1; \beta_1 \ge 1} \frac{r! \beta_1}{\beta_1!\beta_2!...\beta_m!} \cdot a_1^{\beta_1}...a_m^{\beta_m} + \sum_{\beta : |\beta| = r + 1; \beta_2 \ge 1} \frac{r! \beta_2}{\beta_1!\beta_2!...\beta_m!} \cdot a_1^{\beta_1}...a_m^{\beta_m} + [/math] <ещё [math] m - k [/math] суммы> = [math] \sum_{|b| = r + 1} \frac{r! (b_1 + ... + b_m)}{b_1! ... b_m!} \cdot a_1^{\beta_1}...a_m^{\beta_m} [/math];

[math] \beta_1 \ge 1 .. [/math] — это ограничение можно убрать, т.к. все слагаемые с [math] \beta_1 = 0 [/math] имеют нулевой индекс

[math] (k_1 + 1, k_2 ... k_m) \to (\beta_1 ... \beta_m) [/math]
[math]\triangleleft[/math]
  • Замечание 1

[math] \sum_{(k_1...k_m); k_i \ge 0; k_1 + ... + k_m = r} \frac{r!}{k_1! ... k_m!} \cdot a_1^{k_{1}} ... a_m^{k_{m}} = [/math][math] \sum_{i_1 = 1}^m \sum_{i_2 = 1}^m ... \sum_{i_r = 1}^m a_{i_1} a_{i_2} ... a_{i_r} [/math]

  • Замечание 2

[math] m = 2; k_1, k_2 = r - k_1 [/math]

[math] \sum_{k_1 = 0}^{r} \frac{r!}{k_1!(r - k_1)!} \cdot a_1^{k_1} a_2^{r - k_1} = (a_1 + a_2)^r [/math]

Лемма о дифференцировании «сдвига»

Лемма:
Пусть [math] f: E \subset \mathbb{R}^m \to \mathbb{R} [/math], [math] E [/math] открыто в [math] \mathbb{R}^m [/math], [math] \ a \in E, \ h \in \mathbb{R}^m [/math], так, что [math] \forall t \in [-1; 1] \ a + th \in E [/math]. Также [math] f \in C^r(E) [/math]. Пусть [math] \varphi (t) = f(a + th) [/math]. Тогда [math] \forall t_0 \in (-1; 1) [/math] верно [math] \varphi^{r} (t_0) = \sum_{\alpha: (\alpha) = r} \frac{r!}{\alpha!} f^{(\alpha)} (a + t_0 h) h^{\alpha} [/math].
Доказательство:
[math]\triangleright[/math]
Доказательства нет, есть пример, из которого можно придумать доказательство по индукции, наверное.
[math]\triangleleft[/math]

Многомерная формула Тейлора (с остатком в форме Лагранжа и Пеано)

Лагранж:

Теорема:
Пусть [math] r \in \mathbb{R}_+ [/math], [math] D [/math] открыто в [math] \mathbb{R}^n [/math], [math] f \in C^{(r + 1)} (D), \ a, x \in \mathbb{R}^n, \ \overline{a, x} \subset D [/math]. Тогда существует такое [math] \theta \in (0, 1) [/math], что [math] f(x) = \sum_{(k) \leqslant r} \frac{f^{(k)} (a) }{k!} (x - a)^k + \sum_{(k) = r + 1} \frac{f^{(k)} (a + \theta(x - a))}{k!} (x - a)^k [/math].
Доказательство:
[math]\triangleright[/math]

[math]\phi(t)=f(a+th), t\in{[-1;1]}[/math]

[math]f(a+h) = \phi(1)[/math]

Разложили [math]\phi(1)[/math] по одномерной формуле Тейлора в точке 0, используя лемму о дифференцировании сдвига, — получили то, что нужно.
[math]\triangleleft[/math]

Также можно обозначить точки через [math] x [/math] и [math] x + h [/math], тогда формула запишется в виде [math] f(x + h) = \sum_{(k) \leqslant r} \frac{f^{(k)} (x)}{k!} h^k + \sum_{(k) = r + 1} \frac{f^{(k)} (x + \theta h)}{k!} h^k [/math].

Пеано:

Теорема:
Пусть [math] r \in \mathbb{N} [/math], [math] D [/math] открыто в [math] \mathbb{R}^n [/math], [math] f \in C^{(r + 1)} (D), \ x \in D [/math]. Тогда [math] f(x + h) = \sum_{(k) \leqslant r} \frac{f^{(k)} (x)}{k!} h^k + o(|h|^r), \ h \to \mathbb{O}_n [/math].

Теорема о пространстве линейных отображений

Теорема:
[math](1) ||\ldots||_{m, n} [/math] — норма в пр-ве [math] \mathcal{L}_{m, n} [/math], то есть

[math] 1. ||A|| \ge 0, ||A|| = 0 \Leftrightarrow A = \mathbb{O}_{m, n} [/math]

[math] 2. \forall \lambda \in \mathbb{R} : ||\lambda A|| = |\lambda|\cdot||A|| [/math]

[math] 3. ||A + B|| \leqslant ||A|| + ||B|| [/math]

[math] (2) A \in \mathcal{L}_{m, n}, B \in \mathcal{L}_{n, k}: ||BA||_{m, k} \leqslant ||B||_{n, k} \cdot ||A||_{m, n} [/math]
Доказательство:
[math]\triangleright[/math]

[math](1)[/math]

1. очевидно [math]||A|| = 0; sup_{|x| \le 1}|Ax| = 0 \Rightarrow Ax \equiv 0 \Rightarrow A = \mathbb{O} [/math] // для [math] x \in B(0, 1) [/math]

2. очевидно, св-ва [math] sup [/math]. Википедия[2]

3. [math] \forall x : |(A + B)x| = |Ax + Bx| \le |Ax| + |Bx| \le ||A||\cdot|x| + ||B||\cdot|x| [/math][math] = (||A|| + ||B||)|x| \Rightarrow ||A + B|| \le C [/math] \\ [math] ||A|| + ||B|| = C [/math]

[math](2)[/math]

[math] |B(Ax)| \le ||B||\cdot|Ax| \le ||B||\cdot||A||\cdot|x| \Rightarrow ||BA|| \le C [/math] \\ [math] ||B|| \cdot ||A|| = C [/math]
[math]\triangleleft[/math]

Теорема Лагранжа для отображений

Теорема:
[math] F : E [/math] откр. [math] \subset \mathbb{R}^m \rightarrow \mathbb{R}^n; [/math] дифф. [math] E; a, b \in E [/math]

[math] [a, b] = \{ c = a + t(b - a), t \in [0, 1] \} \subset E [/math]

Тогда: [math] \exists c \in [a, b] : |F(b) - F(a)| \le ||F'(c)||\cdot|b - a| [/math]
Доказательство:
[math]\triangleright[/math]

[math] g(t) = F(a + t(b - a)), t \in [0, 1] \\ g'(t) = F'(a + t(b - a))\cdot(b - a) [/math] // [math] |g(b) - g(a)| \le |g'(c)|\cdot|b - a| [/math]

[math] ||F(b) - F(a)|| = |g(1) - g(0)| \le |F'(c)(b - a)| \le ||F'(c)||\cdot|b - a| [/math]
[math]\triangleleft[/math]

Теорема об обратимости линейного отображения, близкого к обратимому

Теорема:
Пусть [math] A \in \Omega(\mathbb{R}^n) [/math] ([math] \Omega(\mathbb{R}^n) [/math] — множество обратимых линейных операторов в [math] \mathbb{R}^n [/math]), [math] B \in \mathcal{L}(\mathbb{R}^n), \ || B - A || \lt \frac{1}{||A^{-1}||} [/math]. Тогда:

1) [math] B \in \Omega (\mathbb{R}^n) [/math];

2) [math] ||B^{-1}|| \leqslant \frac{1}{||A^{-1}||^{-1} - ||B - A||} [/math];

3) [math] ||B^{-1} - A^{-1}|| \leqslant \frac{||A^{-1}||}{||A^{-1}||^{-1} - ||B - A||} ||B - A|| [/math].
Доказательство:
[math]\triangleright[/math]

Лемма: пусть [math]\exists{c \gt 0} : \forall{x} |Bx| \ge c|x|[/math]

Тогда [math]B[/math] — обратим, [math]||B^{-1}|| \le \frac{1}{c}[/math]

Это правда, потому что [math]\operatorname{Ker}{B} = \{0\}[/math], значит, [math]B[/math] — биекция(пусть [math]B(x_1)=B(x_2): B(x_1)-B(x_2)=0 \Leftrightarrow B(x_1 - x_2) = 0 \Rightarrow x_1 = x_2[/math])

Неравенство получается из [math]|Bx| \ge c|x|[/math] заменой [math]Bx=y, x = B^{-1}y[/math]

Само доказательство:

[math]|Bx| = |Ax + (B-A)x| \ge |Ax| - |(B-A)x| \ge \frac{1}{||A^{-1}||}|x| - ||B-A|| \cdot |x| = (\frac{1}{||A^{-1}||} - ||B-A||) \cdot |x|[/math]

По условию теоремы множитель в последней части больше нуля, поэтому по лемме [math]B[/math] обратим, по этой же лемме выполнено 2).


[math]||B^{-1} - A^{-1}|| = ||B^{-1}\cdot (A-B) \cdot A^{-1}|| \le ||B^{-1}||\cdot ||A-B|| \cdot ||A^{-1}|| \le \frac{||A^{-1}||}{||A^{-1}||^{-1} - ||B - A||} ||B - A||[/math]
[math]\triangleleft[/math]

Теорема о непрерывно дифференцируемых отображениях

Теорема:
Пусть [math] F : E \subset \mathbb{R}^m \rightarrow \mathbb{R}^n [/math], где [math] E [/math] открыто, дифференцируемо на [math] E [/math]. Тогда эквивалентны утверждения:

[math] I) F \in C^{1}(E) [/math]

[math] II) F' : E \rightarrow \mathcal{L}_{m, n} [/math] — непрерывна.
Доказательство:
[math]\triangleright[/math]

[math] I \Rightarrow II [/math]

[math] ||A|| \le \sqrt{\sum a_i^2}; A = (a_{ij}); [/math]

? [math] F' [/math] непр. в [math] (\cdot) \overline{X} [/math]

[math] \forall \epsilon \gt 0 \exists \delta \gt 0 : \forall x : |x - \overline{x}| \lt \delta [/math]

[math] ||F'(x) - F'(\overline{x})|| \lt \epsilon [/math]

[math] ||F'(x) - F'(\overline{x})|| \le \sqrt{\sum(\frac{\partial f_i}{\partial x_j}(x) - \frac{\partial f_i}{\partial x_j}(\overline{x}))^2} [/math]

[math] \forall \epsilon \gt 0 [/math] выберем [math] \delta : |\frac{\partial f_i}{\partial x_j}(x) - \frac{\partial f_i}{\partial x_j}(\overline{x})| \lt \frac{\epsilon}{\sqrt{mn}}[/math]; при [math] |x - \overline{x}| \lt \delta; i = 1 \ldots n; j = 1 \ldots m [/math]

[math] II \Rightarrow I [/math]

[math] F' [/math] — непрерывна. [math] e_1 \ldots e_m [/math] — нормированный базис [math]\mathbb{R}^m[/math]

[math] F'(x)e_i = \begin{pmatrix} \frac{\partial f_i}{\partial x_1}(x) \\ \ldots \\ \frac{\partial f_i}{\partial x_n}(x) \end{pmatrix}; [/math]

[math] \begin{matrix} |F'(x)e_i| \le ||F'(x)|| \cdot 1 \\ |\frac{\partial f_i}{\partial x_j}(x)| \le |F'(x)e_i| \le ||F'(x)|| \end{matrix} [/math]

Точно также: [math] |\frac{\partial f_i}{\partial x_j}(x) - \frac{\partial f_i}{\partial x_j}(\overline{x})| \le ||F'(x) - F'(\overline{x})|| [/math]
[math]\triangleleft[/math]

Необходимое условие экстремума. Теорема Ролля

Необходимое условие экстремума:

Теорема:
Пусть [math] f: E [/math] открыто [math] \subset \mathbb{R}^m \to \mathbb{R}; \ \ a [/math] — точка лок. экстремума. [math] f [/math] — дифф. на [math] E [/math]. Тогда [math] \nabla_a f = 0 [/math] (т.е. [math] f'_{x_1}(a) = 0, \ldots, f'_{x_m}(a) = 0 [/math])
Доказательство:
[math]\triangleright[/math]
Меняем [math]f(a+l)[/math] на [math]g(t)=f(a+tl)[/math], по теореме Ферма из первого семестра [math]g'(0)=0[/math]. Из этого следует, что все частные производные в точке a равны нулю, что нам и было нужно.
[math]\triangleleft[/math]

Теорема Ролля:

Теорема:
Пусть [math] f: K [/math] компакт [math] \subset \mathbb{R}^m \to \mathbb{R} [/math], дифференцируемо на [math] \operatorname{Int} K \ne 0 [/math], [math] f \equiv \operatorname{const} [/math] на [math] \partial K [/math] (граница [math] K [/math]), [math] f [/math] — непр. на [math] K [/math]. Тогда существует [math] a \in \operatorname{Int} K: \ \nabla f(a) = 0 [/math].
Доказательство:
[math]\triangleright[/math]

Если [math]f[/math] постоянна на [math]K[/math], то утверждение очевидно.

Если нет, то по теореме Вейерштрасса [math]f[/math] на компакте достигает наибольшего или наименьшего значения в какой-то точке, а по необходимому условию экстремума в этой точке градиент равен нулю.
[math]\triangleleft[/math]

Лемма об оценке квадратичной формы и об эквивалентных нормах

Утверждение:
1) Если квадратичная форма [math] h [/math] положительно определена, то существует такое [math] \gamma_h [/math], что [math] h(x) \ge \gamma_h |x|^2 [/math] для всех [math] x \in \mathbb{R}^m [/math]
2) Пусть [math] p : \mathbb{R}^m \to \mathbb{R}_+ [/math] — норма. Тогда [math] \exists c_1, c_2 \gt 0 \ \forall x \ c_1 |x| \leqslant p(x) \leqslant c_2 |x| [/math].
[math]\triangleright[/math]

1) [math] \gamma_h = min_{|x| = 1}h(x) [/math]

(Сфера [math] \{ x : |x| = 1 \} [/math] — компакт по теореме Вейерштрасса [math] \exists min [/math])

[math] x = 0 : \text{ok} [/math]

[math] x \ne 0 : h(x) = h(|x| \cdot \frac{x}{|x|}) = |x|^2 \cdot h(\frac{x}{|x|}) \ge \gamma_h |x|^2 [/math]

[math] h(tx) = t^2 h(x) [/math]

2) [math] c_1 := min_{|x| = 1} p(x); c_2 := max_{|x| = 1} p(x); [/math] — по т. Вейерштрасса (т.к. [math]p(x)[/math] — непр.)

[math] x = 0 : \text{triv} [/math]

[math] x \ne 0 : p(x) = p(|x| \cdot \frac{x}{|x|}) = |x| \cdot p(\frac{x}{|x|}) \begin{matrix} \le c_2|x| \\ \ge c_1|x| \end{matrix} [/math]
[math]\triangleleft[/math]

Достаточное условие экстремума

Теорема:
Пусть [math] f = Е [/math] открыто в [math] \mathbb{R}^m \to \mathbb{R} [/math], дифф. на [math] Е, a \in E [/math] — стационарная точка [math] f [/math] (то есть [math] \nabla f(a) = \mathbb{O}_m [/math]). [math] d^2 f(a, h) = Q(h) [/math] — кв. форма.

Тогда справедливы следующие утверждения:

1) Если [math] Q(h) [/math] положительно определённая, то [math] a [/math] — точка минимума (локального).

2) Если [math] Q(h) [/math] отрицательно определённая, то [math] a [/math] — точка максимума (локального).

3) Если [math] Q(h) [/math] не знакоопределённая, то [math] a [/math] — не точка экстремума.

4) Если [math] Q(h) [/math] положительно/отрицально опр. вырожденное, то (?) может быть макс., мин. требуется исследование
Доказательство:
[math]\triangleright[/math]

[math](1) : f(a + h) = f(a) + \sum_{i = 1}^{m} f'_{x_i}(a) \cdot h_i + \frac{1}{2} \sum f''_{x_i x_j}(a + \theta h)h_i h_j [/math]

[math] 2(f(a + h) - f(a)) = \sum_{i, j = 1}^{m}f''_{x_i x_j}(a)h_i h_j + \sum_{i, j = 1}^{m}(f''_{x_i x_j}(a + \theta h) - f''_{x_i x_j}(a))h_i h_j [/math] // [math] |h_i| \lt |h| [/math]

Выберем [math] U(a) [/math] так, чтобы при [math] a + h \in U(a) [/math]

[math] \sum |f''_{x_i x_j}(a + \theta h) - f(a)| \le \frac{\gamma}{2} [/math]

[math] 2(f(a + h) - f(a)) \ge \gamma_Q |h|^2 - \frac{\gamma_Q}{2} |h|^2 \gt 0 [/math]

Таким образом [math]a[/math] точка локального минимума

[math](3) : Q(h) [/math] — не знакоопределён. [math] \begin{matrix} h \ne 0 & Q(h) \ge 0 \\ \bar h \ne 0 & Q(\bar h) \lt 0 \end{matrix} [/math]

[math] 2(f(a + th) - f(a)) = Q(th) + \sum(f''_{x_i x_j}(a + \theta th) - f''_{x_i x_j}(a))th_i th_j = [/math]

[math] = t^2 Q(h) + t^2 \sum(f''_{x_i x_j}(a + \theta th) - f''_{x_i x_j}(a))h_i h_j [/math]

[math]Q(h) \gt 0; t^2 \sum(f''_{x_i x_j}(a + \theta th) - f''_{x_i x_j}(a))h_i h_j [/math] — при [math] t \to 0 [/math] эта сумма из '?' б.м по модулю [math] \le Q(h) [/math] при малых [math] t [/math]
[math]\triangleleft[/math]

Лемма о почти локальной инъективности

Лемма:
Пусть [math] F: O \subset \mathbb{R}^m \to \mathbb{R}^m [/math] — диффеоморфизм, [math] x_0 \in \mathbb{R}^m , \ \det F'(x_0) \neq 0 [/math]. Тогда [math] \exists c, \delta \gt 0 \ \forall h: |h| \lt \delta \ | F(x_0 + h) - F(x_0) | \geqslant c|h| [/math]
Доказательство:
[math]\triangleright[/math]

1) [math] F [/math] — линейное. [math] \exists (F'(x_0))^{-1} [/math]

[math] F(x_0 + h) - F(x_0) = F(h); F'(x_0) \equiv F [/math]

[math] |h| = |F^{-1} Fh| \le ||F^{-1}|| \cdot |Fh| [/math]

[math] |Fh| \ge \frac{1}{||F^{-1}||} \cdot |h|; c := \frac{1}{||F^{-1}||} [/math]

2) [math] F(x_0 + h) - F(x_0) = F'(x_0)h + \alpha(h)\cdot|h|; c = \frac{1}{||F'(x_0)^{-1}||} [/math]

[math] |F(x_0 + h) - F(x_0)| \ge |F'(x_0)h| - |\alpha(h)|\cdot|h| \ge c|h| - |\alpha(h)|\cdot|h| [/math][math] = (c - (\alpha(h))) \cdot |h| \ge^* \frac{c}{2}\cdot|h| [/math]

// [math] \ge^*: \exists \delta \gt 0: [/math] при [math] |h| \lt \delta: |\alpha(h)| \lt \frac{c}{2} [/math]
[math]\triangleleft[/math]

Теорема о сохранении области

Теорема:
Пусть [math] F: O \subset \mathbb{R}^m \to \mathbb{R}^m [/math], где [math] O [/math] открыто — диффеоморфизм в [math] O [/math], [math] \forall x \in O \ \det(F'(x)) \neq 0 [/math]. Тогда [math] F(O) [/math] открыто.
  • Замечание

1. Если [math] O [/math] — лин. связное и [math] F [/math] — непр. [math] \Rightarrow F(O) [/math] — лин. связное

2. Непрерывность [math] F : \forall A \subset \mathbb{R}^m : F^{-1}(A) [/math] — откр. [в [math] O [/math]]
Доказательство:
[math]\triangleright[/math]

[math] x_0 \in O; y_0 = F(x_0) [/math] — внутрення точка [math] F(O) [/math]?

[math] \exists c, \delta : \forall |h| \le \delta \ |F(x_0 + h) - F(x_0)| \ge c|h| [/math]

при [math] |h| = \delta \ F(x_0 + h) \ne F(x_0) = y_0 [/math]

[math] dist(y_0, A) = inf_{a \in A} \rho (y_0, c)[/math]

Возьмем [math] r = \frac{1}{2} dist(y_0, F(S(x_0, \delta))) [/math](S — сфера, т. е. граница шара)

Утверждение: [math] B(y_0, r) \subset F(O) [/math]

Т.е.: [math] \forall y \in B(y_0, r) \ \exists x \in B(x_0, \delta) \ F(x) = y [/math]

[math] \varphi(x) = |F(x) - y|^2 = (F_1(x_1...x_m) - y_1)^2 + (F_2 - y_2)^2 + \ldots + (F_m - y_m)^2; [/math] [math] x \in B(x_0, \delta[/math]

[math] min \varphi [/math] — внутри [math] B(x_0, \delta) [/math]

В точке [math]x_0: \varphi(x_0) = |y_0 - y|^2 \lt r^2 [/math].

На сфере [math] S(x_0, \delta) [/math]: [math] \varphi(x) = |F(x) - y|^2 \ge (\overbrace{|F(x) - y_0|}^{ \ge 2r} - \overbrace{|y - y_0|}^{ \lt r })^2 \ge r^2 [/math]

[math] \varphi [/math] — имеет [math] (\cdot) min [/math] внутри шара [math] B(x_0, \delta) [/math] по теореме Вейерштрасса

[math] \begin{cases} 2(F_1(x_1...x_m) - y_1)\frac{\partial F_1}{\partial x_1} + 2(F_2(x_1...x_m) - y_2)\frac{\partial F_2}{\partial x_1} + \ldots + 2(F_m() - y_m)\frac{\partial F_m}{\partial x_1} = 0 \\ \ldots \\ 2(F_1(x_1...x_m) - y_1)\frac{\partial F_1}{\partial x_m} + \ldots + 2(F_m() - y_m)\frac{\partial F_m}{\partial x_m} = 0 \end{cases} [/math]

[math] det(\frac{\partial F_i}{\partial x_j}) \ne 0 \Rightarrow [/math] в точке минимума [math] \begin{matrix} F_1(x_1...x_m) = y_1 \\ \ldots \\F_m(x_1..x_m) = y_m \end{matrix} [/math](у системы есть только тривиальное решение)
[math]\triangleleft[/math]

Теорема о диффеоморфизме

Теорема:
Пусть [math] F: O \subset \mathbb{R}^m \to \mathbb{R}^m, \ F \in C^r(O) [/math], [math] F [/math] — обратима и её производная невырождена, [math] (\forall x \in O \ \det(F'(x))) \neq 0 [/math].

Тогда:

1) [math] F^{-1} \in C^r [/math]

2) [math] y_0 = F(x_0), \ (F^{-1})' (y_0) = (F'(x_0))^{-1} [/math]
Доказательство:
[math]\triangleright[/math]

1) [math] r = 1 [/math]

[math]F(O) = O' [/math] — открытое

Пусть [math] S = F^{-1}, S : O' \to O[/math]

Пусть [math] U \subset O[/math] — открытое, тогда [math] S^{-1}(U) [/math] — открытое.

  • [math] T : X \to Y[/math] — непрерывное отображение [math] \Leftrightarrow \forall U \subset Y : T^{-1}(U) [/math] — открыто. // Мне кажется, из определения диффеоморфизма и предыдущей теоремы следует, что обратное отображение тоже диффеоморфизм и предыдущие строчки и так очевидны.

[math] y_0 = F(x_0); x_0 = S(y_0) [/math]

[math] y - y_0 = F(x) - F(x_0) = A(x - x_0) + o(x - x_0) [/math]

[math] S(y) - S(y_0) = x - x_0 = A^{-1}(y - y_0) - A^{-1} o(x - x_0) [/math]

  • [math] T [/math] — диффеоморфизм, матрица [math]T'(x_0)[/math] невырождена [math]\Rightarrow[/math] [math] \exists c, \delta \ \forall x \in B(x_0, \delta) \ |T(x) - T(x_0)| \gt c|x - x_0| [/math] // По лемме о почти локальной инъективности

Возьмём [math] c, \delta [/math] из леммы.

Пусть [math] T = F'(x_0) [/math]

[math] y - y_0 = T(x - x_0) + \alpha(x)|x - x_0| [/math]

[math] S(y) - S(y_0) = T^{-1}(y - y_0) - \overbrace{T^{-1} \alpha(x) |S(y) - S(y_0)|}^{? o(y - y_0)} [/math]

Можно считать, что [math] y [/math] близко к [math] y_0 [/math], так что [math] |x - x_0| = |S(y) - S(y_0)| \lt \delta [/math]

[math] | \ T^{-1} \alpha(x) \cdot |x - x_0| \ | = |T^{-1}(\alpha(x))|\cdot|x - x_0| \le [/math][math] \| T^{-1} \| \cdot |\alpha(x)| \cdot \frac{1}{c} |F(x) - F(x_0)| \le \frac{\| T^{-1} \|}{c}|y - y_0|\cdot|\alpha(x)| [/math]

[math]// y \to y_0; x \to x_0; \alpha(x) \to 0 [/math]

[math] y \mapsto S(y) = x \mapsto F'(x) = T \mapsto T^{-1} = S'(y) [/math]

2) [math] r [/math] — любое. (без доказательства)
[math]\triangleleft[/math]

Теорема о локальной обратимости

Теорема:
Пусть [math] F: O \subset \mathbb{R}^m \to \mathbb{R}^m [/math], где [math] O [/math] открыто; [math] F \in C^1(O, \mathbb{R}^m); x_0 \in O; \det F'(x_0) \ne 0 [/math] Тогда [math] \exists U(x_0): \ F |_U [/math] — диффеоморфизм ([math] F |_U [/math] или [math] F|U [/math] — сужение отображения [math] F [/math] на множество [math] U [/math]).
Доказательство:
[math]\triangleright[/math]

Нужно проверить лишь: [math] \exists U(x_0) : F|_U [/math] — обратима

[так как можно считать что [math] \det F'(x) \ne 0 [/math] на [math] U(x_0) \Rightarrow F(U(x_0)) [/math] открыто и [math] F^{-1} [/math] определено на открытом множестве и дифференцируемо по предыдущим теоремам]

[math] |F(x) - F(y)| \ge^{?} |x - y| [/math] // Это какая-то хрень, к тому же она в конце не доказана. Надо проверить, что [math]\forall{x \neq y} |F(x) - F(y)| \gt 0[/math], тогда отображение будет биекцией.

[math] \exists c \ \forall h \in \mathbb{R}^m : |F'(x_0)h| \ge c|h|; \ U = B(x_0, r) \subset O [/math]

[math] \begin{matrix} 1: \forall x \in U & \det F'(x) \ne 0 \\ 2: \forall x \in U & \| F'(x) - F'(x_0) \| \lt \frac{c}{4} \end{matrix} [/math]

[math] x, y \in B(x_0, r); y = x + h [/math]

[math] F(y) - F(x) = ( F(x + h) - F(x) - F'(x)h ) + ( F'(x) - F'(x_0) )h + F'(x_0)h [/math]

[math] |F(y) - F(x)| \ge |F'(x_0)h| - |F(x + h) - F(x) - F'(x)h| - |(F'(x) - F'(x_0))h| \ge [/math]

[math] \ge c|h| - sup_{t \in [x, x + h]} \| F'(t) - F'(x) \| \cdot |h| - \| F'(x) - F'(x_0) \| \cdot |h| \ge c|h| - \frac{c}{4}|h| - \frac{c}{4}|h| = \frac{c}{2}|h| \gt 0[/math]
[math]\triangleleft[/math]
  • Замечание

[math] \det F' \ne 0 [/math] — нужно для дифференцируемости.

[math] F : \mathbb{R} \to \mathbb{R}; x \mapsto x^3; F^{-1} [/math] — не дифференцируемо в нуле

Теорема о неявном отображении

Теорема:
Пусть [math] F: E \subset \mathbb{R}^{m + n} \to \mathbb{R}^n [/math], где [math] E [/math] открыто, [math] F \in C^r (E, \mathbb{R}^n), \ (a, b) \in E, \ F(a, b) = 0 [/math]. Пусть известно, что [math] F'_y (a, b) [/math] невырождено ([math] \det F'_y (a, b) \neq 0 [/math]). Тогда:

1) существуют открытые [math] P \subset \mathbb{R}^m, \ Q \subset \mathbb{R}^n, \ a \in P, \ b \in Q [/math], и существует единственное [math] \varphi: P \to Q, \varphi \in C^r [/math], что [math] \forall x \in P \ F(x, \varphi(x) ) = 0 [/math]

Раньше тут был забыт минус!

2) [math] \varphi'(x) = -[F'_y (x, \varphi(x) ) ]^{-1} \cdot F'_x(x, \varphi(x)) [/math]
Доказательство:
[math]\triangleright[/math]

Пусть [math]\Phi(x, y) = (x, F(x,y))[/math].

[math]\Phi(a, b) = (a, 0)[/math]

[math]\Phi{'} = \begin{pmatrix} E_n & O \\ F'_x & F'_y \end{pmatrix}[/math].

[math]\det{\Phi'} = \det{F'_y} \neq 0[/math]

По теореме о локальной обратимости [math]\exists{U(a,b)}[/math] — такая, что [math]\Phi[/math] — диффеоморфизм в данной окрестности.

Тогда существует обратное отображение [math]\Psi(u, v) = (u, H(u, v))[/math].

Почти очевидно, что [math]\varphi(x) = H(x, 0)[/math].

Берем производную — получаем 2): [math]F'(x, \varphi(x)) = F'_x + F'_{y}\varphi{'} = 0[/math]
[math]\triangleleft[/math]

Теорема о задании гладкого многообразия системой уравнений

Теорема:
Пусть [math] M \subset \mathbb{R}^m, \ 1 \leqslant k \lt m, \ 1 \leqslant r \leqslant + \infty [/math] (гладкое многообразие), [math] p \in M [/math].

Эквивалентные утверждения:

1) [math] \exists U(p) \subset \mathbb{R}^m: \ M \cap U(p) [/math] — простое [math] k [/math]-мерное многообразие

2) [math] \exists \tilde{U}(p) [/math] и существуют функции [math] f_1, ..., f_{m - k}: \tilde{U}(p) \to \mathbb{R} [/math] класса [math] C^r [/math], для которых выполняются условия:

2.1) [math] x \in M \cap \tilde{U}(p) \leftrightarrow f_1(x) = 0, ... , f_{m - k}(x) = 0 [/math]

2.2) [math] \nabla f_1, ... , \nabla f_{m - k} [/math] — линейно независимые
Доказательство:
[math]\triangleright[/math]

[math] 1 \Rightarrow 2 [/math]

[math] \Phi : \Omega \to \mathbb{R}^m [/math] — параметризация [math] C^r; \ p = \Phi(t_0); \ \Phi'(t_0) [/math] — матрица [math] m \times k [/math]

[math] Rg \Phi'(t_0) = k [/math] — реализуется на первых [math] k [/math] степенях

[math] \det( \frac{\partial \Phi_i}{\partial U_j} (t_0) ) \ne 0; \ L : \mathbb{R}^m \mapsto \mathbb{R}^k; \ (x_1 ... x_m) \mapsto (x_1 ... x_k) [/math]

[math] 2 \Rightarrow 1 [/math]

Очевидно: [math] (L \circ \Phi)'(p) [/math] — невырожденно.

[math] \Phi = (\Phi_1 ... \Phi_m); L \circ \Phi = (\Phi_1 ... \Phi_k) [/math]

[math] \exists W(t_0) : L \circ \Phi [/math] — диффеоморфизм на [math] W(t_0) [/math]

[math] V = (L \circ \Phi)(W) \Rightarrow L [/math] взаимно однозначное отображение [math] \Phi(W) [/math] на [math] V [/math]

[math] \Psi_1 = (L \circ \Phi)^{-1}; \ H : V \to \mathbb{R}^{m - k}; \ \Phi(\Psi(V)) = (V, H(V)) [/math]

[math] \Phi(W) [/math] — открыто в [math] M \Rightarrow \Phi(W) [/math] — реал. как [math] G \cap M, \ G [/math] — откр. в [math] \mathbb{R}^m [/math]

[math] G := V \times \mathbb{R}^{m - k}; \ \tilde{U} = G \cap G_1 [/math]

[math] \begin{cases} f_1 = H_1 - X_{k + 1} \\ \ldots \\ f_{m - k} = H_{m - k} - X_m \end{cases} [/math]

[math] \begin{matrix} \nabla f_1 = (\frac{\partial H_1}{\partial x_1} \cdots \frac{\partial H_1}{\partial x_k}, 1, 0, \ldots, 0 ) \\ \cdots \\ \nabla f_{m - k} = ( \frac{\partial H_{m - k}}{\partial x_1} \cdots \frac{\partial H_{m - k}}{\partial x_k}, 0, \ldots, 0, 1 ) \end{matrix} [/math]
[math]\triangleleft[/math]

Необходимое условие относительного локального экстремума

Теорема:
Пусть [math] f: E \subset \mathbb{R}^{m+n} \to \mathbb{R} [/math], где [math] E [/math] открыто, [math] \Phi : E \to \mathbb{R}^n, \ a \in E, \ \Phi(a) = 0, \ \operatorname{rg} \Phi'(a) = n [/math]. Пусть [math] f [/math] имеет в точке [math] a [/math] локальный относительный экстремум. Тогда [math] \exists \lambda = (\lambda_1 , ... , \lambda_m) \in \mathbb{R}^n [/math], что [math] \begin{cases} f'(a) + \lambda \Phi'(a) = \mathbb{O}_{m+n} \\ \Phi(a) = \mathbb{O}_n \end{cases} [/math]
Доказательство:
[math]\triangleright[/math]

Пусть ранг реализуется на столбцах [math] x_{m + 1}, \ldots, x_{m + n} [/math]. Переобозначим [math] y_1 = x_{m + 1}; \ldots; y_n = x_{m + n} [/math].

По теореме о неявном отображении: [math] \exists \Psi: U(a_x) \rightarrow W(a_0) \\ \forall x \in U(a_x) \ \Phi(x, \Psi(x)) = 0 [/math]

[math] x \mapsto (x, \Psi(x)) [/math] — гл. параметризация

[math] g(x) = f(x, \Psi(x)) [/math]; Точка [math] a_x [/math] — лок. экстремум [math] g' [/math].

[math] f'_x(a) + f'_y(a) \cdot \Psi'(a_x) = 0 [/math] — необходимое усл. экстремума в матр. форме.

[math] \Phi'_x(a) + \Phi'_y(a) \cdot \Psi'(a_x) = 0 [/math]

[math] \forall \lambda \in \mathbb{R}^n : \ \lambda \Phi'_x(a) + \lambda \Phi'_y(a) \cdot \Psi'(a_x) = 0 [/math]

[math] (f'_x(a) + \lambda \Phi'_x(a)) + (f'_y(a) + \lambda \Phi'_y(a)) \cdot \Psi'(a_x) = 0 [/math]

[math] \lambda := -(f'_y(a))(\Phi'_y(a))^{-1} [/math]

При таком [math] \lambda : [/math]

[math] \begin{cases} f'_x(a) + \lambda \Phi'_x(a) = 0 \\ f'_y(a) + \lambda \Phi'_y(a) = 0 \\ \Phi(a) = 0 \end{cases} [/math]
[math]\triangleleft[/math]

Вычисление нормы линейного оператора с помощью собственных чисел

Теорема:
Пусть [math] A \in \mathcal{L}_{m, n} [/math]. Тогда [math] || A || = \max \{\sqrt{\lambda}, \lambda [/math] — собственное число [math] A^T \cdot A \} [/math].
Доказательство:
[math]\triangleright[/math]
[math] ||A||^2 = max_{|x| = 1}|Ax|^2 = max_{|x| = 1} \langle Ax, Ax \rangle = max_{|x| = 1}\langle A^tAx, x \rangle [/math]
[math]\triangleleft[/math]

Простейшие свойства интеграла векторного поля по кусочно-гладкому пути

1) Линейность по векторному полю: [math] I(\alpha V_1 + \beta V_2, \gamma) = \alpha I(V_1, \gamma) + \beta I(V_2, \gamma) [/math].

[math] \int_{a}^{b} \langle (\alpha V_1 + \beta V_2), \gamma{'} \rangle dt [/math] — по линейному скалярному произведению

2) Аддитивность при дроблении пути:

[math] \gamma : [a, b] \to \mathbb{R}^m; \ c \in [a, b] [/math]

[math] \gamma_1 : [a, c] \to \mathbb{R}^m; \ t \mapsto \gamma(t); \ \gamma_2 : [c, b] \to \mathbb{R}^m [/math]

[math] I(V, \gamma) = I(V, \gamma_1) + I(V, \gamma_2) [/math].

[math] \int_{a}^{b} ... = \int_a^c + \int_c^b [/math]

3) Замена параметра: если [math] \varphi: [p; q] \to [a; b] [/math] — гладкая, [math] \varphi(p) = a, \ \varphi(q) = b [/math], [math] \gamma: [a; b] \to \mathbb{R}^m [/math], [math] \tilde{\gamma} = \gamma \circ \varphi: [p; q] \to \mathbb{R}^m [/math] [math] s \mapsto \gamma(\varphi(s)) [/math]

Тогда [math] I(V, \gamma) = I(V, \tilde{\gamma}) [/math].

[math] I(V, \gamma) = \int_a^b \langle V(\gamma(t)), \gamma{'}(t) \rangle dt =_{t = \varphi(s)} [/math][math] \int_a^b \langle V (\gamma(\varphi (s))), \gamma{'}(\varphi (s)) \varphi'(s) \rangle ds = \int_p^q \langle V(\tilde{\gamma}(s)), \tilde{\gamma}'(s) \rangle ds [/math]

4) Пусть [math] \gamma_1: [a; b] \to \mathbb{R}^m, \ \gamma_2: [c; d] \to \mathbb{R}^m, \ \gamma_1(b) = \gamma_2(c), \ \gamma = \gamma_2 \gamma_1 [/math] — произведение путей:

[math] \gamma: [a; b + d - c] \to \mathbb{R}^m = \begin{cases} \gamma_1(t), \ t \in [a; b] \\ \gamma_2(t - b + c), \ t \in [b; b + d - c] \end{cases} [/math]

то [math] I(V, \gamma_2 \gamma_1) = I(V, \gamma_1) + I(V, \gamma_2) [/math].

[math] \int_a^{b + d - c} \langle V(\gamma(t)), \gamma{'}t \rangle dt = \int_a^b + \int_b^{b + d - c} [/math] \\ заменить параметр [math] s = t - b + c; s \in [c, d] [/math]

[math] \gamma : [a, b] \to \mathbb{R}^m; \ \gamma_- [/math] — противоположный путь (в обратную сторону)

[math] \gamma_-(t) = \gamma(b + a - t), t \in [a, b] [/math]

[math] I(V, \gamma_-) = -I(V, \gamma) [/math]

[math] \int_a^b \langle V(\gamma(b - a - t)), \gamma_-(t) \rangle dt = \int \langle V (\gamma(s)), \gamma{'}(s) \rangle ds [/math]

5) Оценка интеграла:

Теорема:
[math] | \int\limits_{a}^{b} (V_1 dx_1 + ... + V_m dx_m) | \leqslant \max_{x \in t_{\gamma}} |V(x)| \cdot L(\gamma) [/math], где [math] L(\gamma) [/math] — длина пути. [math] \gamma : [a, b] \to \mathbb{R}^m; L_{\gamma} = \gamma [a, b] \subset \mathbb{R}^m [/math]
Доказательство:
[math]\triangleright[/math]
[math] | \int_a^b \sum V_i (\gamma(t)) \cdot \gamma{'}_i(t) dt | \le \int_a^b |...| dt \le \int_a^b \sqrt{\sum V_i^2(\gamma(t))} \sqrt{\sum \gamma_i^{'2}(t)} dt = \int_a^b |V(\gamma(t))| \cdot |\gamma{'}(t)| \le max_{x \in L_{\gamma}} (V(x)) \cdot \int_a^b |\gamma{'}(t) dt| [/math]
[math]\triangleleft[/math]

Обобщенная формула Ньютона--Лебница

Теорема:
Пусть [math] V: O \to \mathbb{R}^m [/math] потенциально, [math] f [/math] — потенциал [math] V [/math], [math] \gamma[a;b] \to O [/math] — кусочно гладкий. Тогда [math] \int\limits_{\gamma} (V_1 dx_1 + ... V_m dx_m) = f(\gamma(b)) - f(\gamma(a)) [/math].
Доказательство:
[math]\triangleright[/math]

1) [math] \int\limits_{\gamma} \sum V_k d x_k = \int\limits_{a}^{b} (V_1(\gamma(t))\cdot\gamma'_1 + \ldots + V_m(\gamma(t))\cdot\gamma'_m) = f(\gamma(t))|_a^b [/math] — доказано для гладкого пути

\\ [math] V_1(\gamma(t))\cdot\gamma'_1 + \ldots + V_m(\gamma(t))\cdot\gamma'_m = f(\gamma(t))' [/math] [math] = f(\gamma_1(t)\ldots\gamma_m(t))' = \frac{\partial f}{\partial x_1}\cdot\gamma'_1 + \ldots + \frac{\partial f}{\partial x_m}\cdot\gamma'_m [/math]

\\ [math] \frac{\partial f}{\partial x_1} = V_1; \ldots; \frac{\partial f}{\partial x_m} = V_m [/math]

2) [math] a = t_0 \lt t_1 \lt \ldots \lt t_n = b [/math]

[math] \gamma|_{[t_{k-1}, t_{k}]} [/math] — гладкий

[math] \int\limits_{\gamma}\sum_k V_k d x_k = \sum_k (\int\limits_{t_k-1}^{t_k} \sum_i V_i d \gamma_i) = [/math][math] \sum(f(\gamma(t_k)) - f(\gamma(t_{k-1}))) = f(\gamma(b)) - f(\gamma(a)) [/math]
[math]\triangleleft[/math]

Характеризация потенциальных векторных полей в терминах интегралов

Теорема:
Если [math] V : O \to \mathbb{R}^m [/math] тогда эквиваленты следующие утверждение:

1) V потенциально в [math] O [/math]

2) Интеграл [math] V [/math] не зависит от пути (в обл. [math] O [/math])

3) [math] \forall \gamma : [a, b] \to O, \ \gamma(a) = \gamma(b); \ \int_{\gamma} \sum V_i dx_i = 0 [/math]
Доказательство:
[math]\triangleright[/math]

[math] 1 \Rightarrow 2 [/math] — формула Ньютона-Лейбница

[math] 2 \Rightarrow 3 [/math] — очевидно

[math] \gamma [/math] — петля; [math] \gamma_1(t) \equiv \gamma(a) [/math]

[math] \int_{\gamma_1} \sum V_i dx_i = 0 = \int_{\gamma} \sum V_i dx_i [/math]

[math] 3 \Rightarrow 2 [/math] — очевидно

[math] \gamma := \gamma_{2-} \cdot \gamma_1; \ 0 = \int_{\gamma} \sum V_i dx_i = \int_{\gamma_{2-}} + \int_{\gamma_1} = \int_{\gamma_1} - \int_{\gamma_2} [/math]

[math] 2 \Rightarrow 1 [/math]

Фиксируем точку [math] x_0 \in O; \ \forall x \in O [/math]

Возьмём как-нибудь путь [math] \gamma_x [/math] из [math] x_0 [/math] в [math] x [/math]

[math] f(x) := \int_{\gamma_x} \sum V_i dx_i; f [/math] — потенциал?

Докажем, что [math] \frac{\partial f}{\partial x_1} = V_1 [/math] (аналогично [math] \frac{\partial f}{\partial x_i} = V_i; \ i = 2...m [/math])

Выберем [math] B(x, r) \subset O [/math]

[math] |h| \lt r; \ t \mapsto (x_1 + th, x_2 ... x_m); \ \gamma'_h(t) = (h, 0, ..., 0) [/math]

[math] f(x_1 + h, x_2 ... x_m) - f(x) = \int_{\gamma_h \gamma_x} \sum V_i dx_i - \int_{\gamma_x} \sum V_i dx_i = [/math]

[math]= \int_{\gamma_h} \sum V_i dx_i = \int_0^1 V_1(x_1 + th, x_2 ... x_m)h dt = [/math] теорема о среднем [math] = V_1(x_1 + \Theta h, x_2 ... x_m)h; \ \Theta \in [0, 1] [/math]

[math] \frac{f(x_1 + h, ... x_m) - f(x)}{h} = V_1(x_1 + \Theta h, ...) \to V_1(x) [/math]
[math]\triangleleft[/math]

Лемма о дифференцировании интеграла по параметру

Лемма:
Пусть [math] f: [a; b] \times [c; d] \to \mathbb{R}, \ f(x, y) [/math] — непрерывна, дифференцируема по [math] y [/math] при любых [math] x [/math] и [math] f'_y [/math] непрерывна на промежутке. Пусть [math] \Phi(y) = \int\limits_a^b f(x, y) dx, \ y \in [c, d] [/math]. Тогда [math] \Phi(y) [/math] дифференцируема и [math] \Phi'(y) = \int\limits_a^b f'_y(x, y) dx [/math].
Доказательство:
[math]\triangleright[/math]

[math] \frac{\Phi(y + h) - \Phi(y)}{h} = \int_a^b \frac{f(x, y + h) - f(x, y)}{h} dx = \int_a^b f'_y (x, y + \Theta h) dx; \ \Theta \in [0, 1] [/math] зависит от [math] x, y [/math]

[math] f'_y [/math] — непрерывна на [math] [a, b] \times [c, d] [/math]

[math] \forall \epsilon \gt 0 \ \exists \delta \gt 0 \ \forall x, y : |x - y| \lt \delta; \ |f'_y(x) - f'_y(y)| \lt \epsilon [/math] — равномерная непрерывность

[math] | \frac{\Phi(y + h) - \Phi(y)}{h} - \int_a^b f'_y(x, y)dx | = | \int_a^b f'_y(x, y + \Theta h) - f'_y(x, y)dx | \le [/math]

[math] \le \int_a^b | f'_y(x, y + \Theta h) - f'_y(x, y) |dx \le^* \int_a^b \epsilon dx = \epsilon(b - a) [/math]

[math] \le^* : \forall \epsilon \gt 0 \ \exists \delta \gt 0 \ \forall h : |h| \lt \delta [/math]

[math] | \frac{\Phi(y + h) - \Phi(y)}{h} - \int_a^b f'_y | \lt \epsilon (b - a) [/math] — определение предела.
[math]\triangleleft[/math]

Необходимое условие потенциальности гладкого поля. Лемма Пуанкаре

Теорема:
Пусть [math] V [/math] — гладкое потенциальное векторное поле в [math] O [/math]. Тогда [math] \forall x \in O \ \frac{\partial V_i}{\partial x_j} = \frac{\partial V_j}{\partial x_i} \ (*), \ i, j \in [1 : m] [/math]
Доказательство:
[math]\triangleright[/math]
[math] f [/math] — потенциал, обе части [math] (*) = \frac{\partial^2 f}{\partial x_i \partial x_j} [/math] (— непр., т.к. [math] V [/math] — гладкое)
[math]\triangleleft[/math]
Лемма:
Пусть [math] O \subset \mathbb{R}^m [/math] — выпуклое, [math] V [/math] — векторное поле в [math] O [/math], гладкое и [math] \forall x \forall i, j \ \frac{\partial V_i}{\partial x_j} = \frac{\partial V_j}{\partial x_i} [/math]. Тогда [math] V [/math] — потенциальное.
Доказательство:
[math]\triangleright[/math]

фиксируем [math] A \in O; \ \gamma [0, 1] \to O; \ t \mapsto A + t * (x - A); \ \gamma' = x - A [/math]

[math] f(x) := \int_{\gamma} \sum V_i dx_i = [/math][math] \int_0^1 V_1(A + t(x - A))\cdot(x_1 - A_1) + ... + V_m(A + t(x - A)) \cdot (x_m - A_m)dt [/math]

[math] \frac{\partial f}{\partial x_i} = \int_0^1 V_i(A + t(x - A)) + \sum_{j = 1}^{m} \overbrace{\frac{\partial V_j}{\partial x_i}}^{\frac{\partial V_i}{\partial x_j}} (A + t(x - A))t(x_j - A_j)dt = [/math]

[math] = \int_0^1 (t V_i (A + t(x - A)))'_t dt = t V_i (A + t(x - A))|_{t = 0}^{t = 1} = V_i (x) [/math]
[math]\triangleleft[/math]

Лемма о гусенице

Лемма:
Пусть [math] \gamma: [a, b] \to O [/math]. Тогда существуют дробление [math] a = t_0 \lt t_1 \lt ... \lt t_n = b [/math] и шары [math] B_1, ..., B_n \subset O [/math], что [math] \gamma [t_{k - 1}, t_k] \subset B_k, \ k \in [1 : n] [/math].
Доказательство:
[math]\triangleright[/math]

[math] \forall c \in [a, b] [/math] — выберем шар [math] B(\gamma(c), V_c) \subset O [/math]

[math] \tilde \alpha_c := \inf \{ \alpha \in [a, b]; \ \gamma([\alpha, c]) \subset B (\gamma(c), V_c) \} [/math]

[math] \tilde \beta_c := \sup \{ \beta \in [a, b]; \ \gamma([c, \beta]) \subset B (\gamma(c), V_c) \} [/math]

Пусть [math] \tilde \alpha_c \lt \alpha_c \lt c \lt \beta_c \lt \tilde \beta_c [/math]

[math] \forall c [/math] мы имеем [math] (\alpha_c, \beta_c) [/math] — открытое покрытие [math] [a, b] [/math] и [math] \exists [/math] конечное подпокрытие

Можно считать [math] \forall i \ \exists s_i [/math] — которое лежит в [math] (\alpha_{c_i}, \beta_{c_i}) [/math], но не лежит в [math] (\alpha_{c_j}, \beta_{c_j}); \ i \ne j [/math]

[math] s_1 \lt s_2 ... \lt s_n [/math]
[math]\triangleleft[/math]

Лемма о равенстве интегралов по похожим путям

Лемма:
Пусть [math] \gamma, \tilde{\gamma}: [a; b] \to O \subset \mathbb{R}^m [/math] — кусочно-гладкие, похожие, [math] V [/math] — локально-потенциальное векторное поле, [math] \gamma(a) = \tilde{\gamma} (a), \ \gamma(b) = \tilde{\gamma} (b) [/math]. Тогда [math] \int\limits_{\gamma} \sum V_i dx_i = \int\limits_{\tilde{\gamma}} \sum V_i dx_i [/math].
Доказательство:
[math]\triangleright[/math]

Cуществуют дробление [math] a = t_0 \lt t_1 \lt ... \lt t_n = b [/math] и шары [math] B_1, ..., B_n \subset O [/math]

[math] \forall k [/math] в [math] B_k [/math] существует потенциал векторного поля [math] V [/math]

[math] \gamma|_{[t_{k - 1}, t_k]} \subset B_k; \ \tilde \gamma|_{[t_{k - 1}, t_k]} \subset B_k [/math]

Пусть [math] f_1 [/math] — потенциал [math] V [/math] в [math] B_1 [/math], в [math] B_2 [/math] выберем потенциал [math] f_2. \ f_1(\gamma(t_1)) = f_2(\gamma(t_1)) [/math]

в [math] B_3 [/math] выберем [math] f_3. \ f_2(\gamma(t_2)) = f_3(\gamma(t_2))) [/math] и т.д.

[math] \int_{\gamma} \sum V_i dx_i = \int_a^b \langle V(\gamma(t)), \gamma(t)dt = \sum_{i = 1}^{n} \int_{t_{i - 1}}^{t_i} = \sum_{i = 1}^{n} f_i (x(t_i)) - f_{i - 1}(\gamma(t_{i - 1})) [/math]

[math] \int_{\tilde \gamma} \sum V_i dx_i = f_n(\tilde \gamma(t_n)) - f_1(\tilde \gamma(t_0)) [/math]
[math]\triangleleft[/math]
  • Замечание

[math] \gamma(a) = \tilde \gamma(a), \ \gamma(b) = \tilde \gamma(b) \\ \gamma(a) = \gamma(b), \ \tilde \gamma(a) = \tilde \gamma(b) [/math]

Лемма о похожести путей, близких к данному

Лемма:
Пусть [math] \gamma: [a, b] \to O [/math]. Тогда [любые два пути, мало отличающиеся от данного — похожие] [math] \exists \delta \gt 0 [/math] такое, что если пути [math] \gamma_1, \gamma_2: [a, b] \to O [/math] — «близкие» к [math] \gamma; * [/math], то есть [math] \forall t \in [a, b] \ \ | \gamma(t) - \gamma_1(t) | \lt \delta, \ | \gamma(t) - \gamma_2(t) | \lt \delta [/math], то [math] \gamma_1, \gamma_2 [/math] похожи.
Доказательство:
[math]\triangleright[/math]

Cуществуют дробление [math] a = t_0 \lt t_1 \lt ... \lt t_n = b [/math] и шары [math] B_1, ..., B_n \subset O [/math] для [math] \gamma [/math]

[math] \gamma[t_{k - 1}, t_{k}] [/math] — компакт в [math] B_k [/math]

[math] \exists \delta_k \gt 0 : \delta_k = dist(\gamma[t_{k - 1}, t_k], \partial B_k); g(t) = dist(\gamma(t), \partial B_k) [/math]

[math] \delta := \min_{1 \le k \le n} \delta_k [/math]

[math] A_k = \{ x \in \mathbb{R}^n : \exists t \in [t_{k - 1}, t_{k}] \ \ \rho(\gamma(t), x) \lt \delta \} \subset B_k [/math]

[math] \forall \gamma_1, \gamma_2 [/math] — удовл. [math] * : \gamma_1 [a, b] \subset \cup_{k = 1}^{n} A_k, \gamma_2 \subset \cup_{k = 1}^{n} A_k [/math] и [math] (\{B_k\}, \{t_i\}) [/math] — гусеница реал. похож. путей
[math]\triangleleft[/math]

Равенство интегралов по гомотопным путям

Теорема:
Пусть [math] V [/math] — локально-потенциальное векторное поле в [math] O [/math], [math] \gamma_0, \gamma_1: [a; b] \to O [/math] — связанно гомотопны. Тогда [math] \int\limits_{\gamma_0} \sum V_i dx_i = \int\limits_{\gamma_1} \sum V_i dx_i [/math]. Тоже верно для петельной гомотопии.
Доказательство:
[math]\triangleright[/math]

[math] \Gamma [/math] — гомотопия. [math] \gamma_u(t) = \Gamma(t, u), \ u \in [0, 1] [/math]

[math] \Phi(u) = \int_{\gamma_u} \sum V_i dx_i [/math]. Проверим, что [math] \Phi [/math] — локальная постоянная

[math] (\forall u_0 \ \exists W(u_0) [/math] при [math] u \in W(u_0) : \Phi [/math] — постоянна)

[math] \Gamma : \overbrace{[a, b] \times [0, 1]}^{copmact} \to O [/math] — равномерно непрерывна.

[math] \forall \delta \gt 0 \ \exists \zeta \gt 0 \ \forall (t_1, u_1), (t_2, u_2) \in [a, b] \times [0, 1] \ \ [/math][math]\ \ \begin{matrix} |t_1 - t_2| \lt \zeta \\ |u_1 - u_2| \lt \zeta \end{matrix} [/math] верно [math] |\Gamma(t_1, u_1) - \Gamma(t_2, u_2)| \lt \frac{\delta}{2} [/math]
[math]\triangleleft[/math]

Потенциальность локально потенциального поля. Следствие о лемме Пуанкаре

Теорема:
Пусть [math] O [/math] — односвязная область, [math] V [/math] — локально потенциальное поле в [math] O [/math]. Тогда [math] V [/math] потенциально.
Доказательство:
[math]\triangleright[/math]

[math] V [/math] — потенциально [math] \Leftrightarrow \forall \gamma : [a, b] \to \mathbb{R}, \ \gamma(a) = \gamma(b) : \ \int_{\gamma} \sum V_i dx_i = 0 [/math]

По предыдущей теореме: [math] \int_{\gamma} \sum V_i dx_i = \int_{\gamma_1} \sum V_i dx_i [/math] — гомотопия пост. пути [math] \gamma_1 [/math]
[math]\triangleleft[/math]

Следствие: если [math] O [/math] — односвязная, [math] V \in C^1(O), \ \forall i, j \ \forall x \in \Omega \ \frac{\partial V_i}{\partial x_j} = \frac{\partial V_j}{\partial x_i} [/math], то [math] V [/math] — потенциально.

Асимптотика интеграла $\int_0^{\pi/2}\cos^nx\,dx$, $n\no+\infty$

Теорема:
[math] \int\limits_0^{\pi/2} \cos^n x dx \underset{n \to + \infty}{\sim} \sqrt{\frac{2}{n}} \int\limits_0^{+\inf} e^{-t^2} dt [/math]
Доказательство:
[math]\triangleright[/math]

Доказательство в три шага, полностью выписывать много, поэтому здесь только идеи:

1) [math]\int\limits_0^{\pi/2} \cos^n x dx \underset{n \to + \infty}{\sim} \int\limits_0^{n^{-\frac{1}{3}}} \cos^{n}x dx[/math]

Доказывается заменой [math]\cos^n{x} = e^{n\ln{\cos{x}}}[/math] и каким-то подбором нового предела интегрирования, зависящего от n (конспект, стр.143)

2) Доказываем, что x — точка максимума для [math]\ln{\cos{x}}[/math], вместе с этим заменяем по формуле Тейлора [math]n\ln{\cos{x}}[/math] на [math]-\frac{nx^2}{2}+o(x^2)[/math] и показываем, что это [math]o(x^2)[/math] не мешает подставить замену в интеграл.

3) Делаем замену [math]t=\sqrt{\frac{n}{2}}x, dx = \sqrt{\frac{2}{n}}dt[/math], получаем интеграл из условия.
[math]\triangleleft[/math]

Лемма о локализации (в методе Лапласа)

Лемма:
Пусть [math] f(x) [/math] непрерывна, [math] f(x) \gt 0 [/math] на [math] (a; b), \ \int\limits_a^b f(x) dx = M, \ \varphi(x) [/math] строго монотонно убывает, непрерывна. Тогда [math] \forall c \in (a, b) \ \int\limits_a^b f(x) e^{A \varphi(x)} \underset{A \to + \infty}{\sim} \int\limits_a^c f(x) e^{A \varphi(x)} [/math].
Доказательство:
[math]\triangleright[/math]

[math] \int_{c}^{b} f(x) e^{A \varphi(x)} \le \max_{x \in [c, b]} e^{A \varphi(x)} \int_c^b f(x)dx \le e^{A \varphi(c)}M [/math]

[math] \int_a^c f(x) e^{A \varphi(x)} dx \ge \int_a^{\frac{c}{2}} f(x)e^{A \varphi(x)} \ge \min e^{A \varphi(x)} \int_a^{\frac{c}{2}} f(x)dx = e^{A \varphi(\frac{c}{2})} \int_a^{\frac{c}{2}} f(x)dx [/math] // последняя экспонента с большим показателем
[math]\triangleleft[/math]

Метод Лапласа вычисления асимптотики интегралов

Теорема:
Пусть [math] f \gt 0 [/math] на [math] (a; b) [/math], непрерывна, [math] \int\limits_a^b f = M, \ f(t) \sim L(t - a)^q, \ t \to a, \ q \gt -1, \ L \gt 0, \ \varphi [/math] непрерывна, строго убывает, [math] \varphi(a) - \varphi(t) \sim c(t - a)^p, \ p \gt 0 [/math]. Тогда [math] \int\limits_a^b f(t) e^{A \varphi(t)} dt \underset{A \to + \infty}{\sim} e^{A \varphi(a)} \cdot \frac{1}{p} \cdot \frac{1}{(cA)^{\frac{q + 1}{p}}} \cdot \Gamma(\frac{q + 1}{p}) [/math].
Доказательство:
[math]\triangleright[/math]
  • В доказательстве используется прием: при [math]q \gt 1, p \gt 0, A \gt 0, s \gt 0[/math] в интеграле [math]\int\limits_0^s t^q e^{-At^p} dt[/math]
  • вводим замену [math]u = At^p, t = (\frac{u}{A})^{1/p}, dt = \frac{u^{1/p-1}}{pA^{1/p}}[/math].
  • Тогда он превращается в [math]\frac{1}{pA^{\frac{q+1}{p}}} \int\limits_0^{As^p} u^{\frac{q+1}{p} - 1}e^{-u}du[/math], который при [math]A\to{+\infty}[/math] стремится к [math]\frac{1}{pA^{\frac{q+1}{p}}}\Gamma({\frac{q+1}{p}})[/math]

Утверждения:

1) [math]\forall{c\in(a, b)}\ \forall{\varepsilon \gt 0}\ \exists{A_0}\ \forall{A \gt A_0}\ \int\limits_a^c{fe^{A\varphi}} \le \int\limits_a^b{fe^{A\varphi}} \le (1 + \varepsilon)\int\limits_a^c{fe^{A\varphi}}[/math] (следствие из теоремы о локализации)

2) [math]\forall{\varepsilon \gt 0}\ \exists{A_0}\ \forall{A \gt A_0}[/math]

[math](1-\varepsilon)\frac{1}{pA^{\frac{q+1}{p}}}\Gamma(\frac{q+1}{p}) \le \int\limits_0^s t^q e^{-At^p} dt \le \frac{1}{pA^{\frac{q+1}{p}}}\Gamma(\frac{q+1}{p})[/math] (следствие из приема выше. Да, читается ужасно)

Доказательство

Выбираем окрестность точки [math]a: [a; a+s][/math] и [math]\varepsilon[/math] такое, что

[math]1-\varepsilon \lt \frac{f(t)}{L(t-a)^q} \lt 1+\varepsilon[/math]

[math]1-\varepsilon \lt \frac{\varphi(a) - \varphi(t)}{c(t-a)^p} \lt 1+\varepsilon[/math]

Для [math]A \gt A_0[/math], удовлетворяющих двум утверждениям выше, выполняется:

[math]\int\limits_a^b f(t)e^{A\varphi(t)} dt \le (1+\varepsilon)\int\limits_a^{a+s}L(t-a)^q \cdot e^{A\varphi(a)} \cdot e^{-A(\varphi(a)-\varphi(t)} dt \le[/math]

[math]\le (1+\varepsilon)Le^{A\varphi(a)}\int\limits_0^s{\tau^q}e^{-Ae^{c(1-\varepsilon)\tau^p}}d\tau[/math]

По утверждению 2 это меньше или равно [math]\frac{1+\varepsilon}{(1-\varepsilon)^{\frac{q+1}{p}}}\cdot L\cdot [e^{A \varphi(a)} \frac{1}{p(cA)^{\frac{q + 1}{p}}} \Gamma(\frac{q + 1}{p})][/math]. В квадратных скобках то, что нам нужно.

Используя другие части неравенства, находим, что [math]\int\limits_a^b f(t)e^{A\varphi(t)} dt \ge \frac{1-\varepsilon}{(1+\varepsilon)^{\frac{q+1}{p}}}\cdot L\cdot [e^{A \varphi(a)} \frac{1}{p(cA)^{\frac{q + 1}{p}}} \Gamma(\frac{q + 1}{p})][/math].

Вроде доказали.
[math]\triangleleft[/math]

Теорема Вейерштрасса о приближении функций многочленами

Теорема:
Пусть [math] f [/math] непрерывна на [math] [a; b] [/math]. Тогда существует многочлен (последовательность многочленов?) [math] P_n(x), \ n = 1, 2 ... [/math], что [math] \forall x \in [a; b] \ P_n(x) \to f(x) [/math].
Доказательство:
[math]\triangleright[/math]

[math] [a, b] \subset [a - 1, b + 1] = [a_1, b_1] [/math] // Можно считать [math] \begin{matrix} [a, b] = [\frac{1}{3}, \frac{2}{3}] \\ [a_1, b_1] = [0, 1] \end{matrix} [/math]

[math] \tilde f(x) = \begin{cases} f(x), x \in [a, b] \\ f(a), x \in [a_1, a] \\ f(b) x \in [b, b_1] \end{cases} [/math]

Заметим, что: [math] \int_{a_1}^{b_1} \tilde f(t)(1 - (x - t)^2)^n dt \sim_{n \to +\infty} \sqrt{\frac{\pi}{n}} f(x); \ x \in [a, b] [/math]

[math] \varphi (t) = ln(1 - (x - t)^2); \ max \varphi [/math] — достигается при [math] t = x [/math]

[math] \varphi(t) \sim -(x - t)^2, t \to x [/math]

[math] \varphi''(x) = -2, \ \varphi(x) = 0 [/math]

[math] Q_n(x) \sim \sqrt{\frac{\pi}{n}} f(x), \ n \to +\infty [/math]

[math] \sqrt{\frac{n}{\pi}} Q_n (x) \to f(x)_{x \in [a_1, b]}, \ n \to +\infty [/math]
[math]\triangleleft[/math]
  • Замечание

[math] \forall f [/math] — непр. на [math] [a, b] \ \ \exists f_n(x) [/math] — многочлен : [math] P_n(x) \rightrightarrows f [/math] на [math] [a, b] [/math]

Формула Стирлинга для Гамма-функции

Теорема:
[math] \Gamma (x + 1) \underset{x \to + \infty}{\sim} x^x e^{-x} \sqrt{2 \pi x} [/math]
Доказательство:
[math]\triangleright[/math]

[math] \Gamma(x + 1) = \int_0^{+\infty} t^x e^{-t} dt =_{t = ux; \ dt = xdu} \ [/math][math]\ x^{x + 1} \int_0^{+\infty} u^x e^{-ux} du = x^{x + 1} \int_0^{+\infty} e^{-x(u - \ln u)} du \sim [/math]

// [math] \varphi(u) = -(u - \ln u) [/math]

// [math] \varphi' = -(1 - \frac{1}{u}); u = 1; \varphi'(u) = 0 - (\cdot) max [/math]

// [math] \varphi'' = -\frac{1}{u^2}; \ \varphi''(1) = -1 [/math]

[math] \sim x^{x + 1} e^{-x} \sqrt{\frac{2\pi}{x}} \cdot \frac{1}{\sqrt{1}} \cdot 1 [/math]
[math]\triangleleft[/math]

[math] \int_{\gamma} \sum V_i dx_i = \int_{\gamma_1} \sum V_i dx_i[/math]

Определения и факты

Перемещено, а то из-за большого размера страница не грузится на некоторых телефонах